Các bài toán chứng minh ba điểm thẳng hàng – ba đường thẳng đồng quy

Tài liệu gồm 80 trang, được biên soạn bởi thầy giáo Nguyễn Công Lợi, hướng dẫn phương pháp và tuyển chọn các bài toán chứng minh ba điểm thẳng hàng – ba đường thẳng đồng quy, đây là dạng toán thường gặp trong các đề tuyển sinh vào lớp 10 môn Toán.

Nguyễn Công Lợi
TÀI LIU TOÁN HC
2
CÁC BÀI TOÁN CHỨNG MINH
BA ĐIỂM THẲNG HÀNG - BA ĐƯỜNG THẲNG ĐỒNG QUY
A. CÁC BÀI TOÁN VỀ BA ĐIỂM THẲNG HÀNG
I. Một số phương pháp chứng minh ba điểm thẳng hàng
Phương pháp 1: Sử dụng góc bù nhau
Nếu có
0
180ABx xBC
thì 3 điểm A, B, C thẳng hàng theo thứ tự đó.
Phương pháp 2: Sử dụng tiên đề về đường thẳng song song
Tiên đề Ơclít: Qua một điểm ở ngoài một đường thẳng chỉ kẻ được duy nhất một đường thẳng
song song với đường thẳng đã cho. Do đó, nếu qua điểm A ta kẻ được AB AC cùng song song
với một đường thẳng d nào đó thì A, B, C thẳng hàng.
Để chứng minh ba điểm A, B, C thẳng hàng ta chứng minh AB AC cùng song song với
một đườngthẳng d.
Phương pháp 3: Sử dụng tiên đề về đường thẳng vuông góc
Để chứng minh ba điểm A, B, C thẳng hàng ta đi chứng minh AB và AC cùng vuông góc với
một đường thẳng d.
Phương pháp 4: Sử dụng 2 tia trùng nhau hoặc đối nhau
Nếu hai tia MA, MB trùng nhau hoặc đối nhau thì 3 điểm M, A, B thẳng hàng.
Phương pháp 5: Thêm điểm
Để chứng minh 3 điểm A, B, C thẳng hàng thể xác định thêm điểm D khác A, B, C sau đó
chứng minh hai trong ba bộ ba điểm A, B, D; A, C, D; B, C, D thẳng hàng.
Phương pháp 6: Phương pháp sử dụng hình đuy nhất
Để chứng minh ba điểm A, B, C thẳng hàng với C thuộc hình H nào đó. Ta gọi C’ là giao điểm
của AB với hình H và tìm cánh chứng minh hai điểm C và C’ trùng nhau.
Phương pháp 7: Sử dụng định lý Menelaus
Cho tam giác ABC. Các điểm A’, B’, C’ lần lượt nằm trên các đường thẳng BC, CA, AB sao
cho trong chúng hoặc không điểm nào, hoặc đúng 2 điểm thuộc các cạnh của tam giác ABC.
Khi đó A’, B’, C’ thẳng hàng khi và chỉ khi
' ' '
. . 1
' ' '
A B B C C A
A C B A C B
Chứng minh
Nguyễn Công Lợi
TÀI LIU TOÁN HC
3
+ Trường hợp 1: Trong 3 điểm A’, B’, C’ có đúng 2 điểm thuộc cạnh tam gi{c ABC. Giả sử
l| B’, C’
- Điều kiện cần: Qua A kẻ đường thẳng song song với BC cắt đường thẳng B’C’ tại M.
Ta có
C'A AM B ' C A ' C
;
C ' B A ' B B' A AM

. Vậy
A'B B ' C C ' A AM A ' C A ' B
. . . . 1
A ' C B ' A C ' B A ' B AM A ' C

- Điều kiện đủ: Gọi A’’ l| giao của B’C’ với BC.
[p dụng định lý Menelaus (phần thuận) ta có
A''B B ' C C ' A
. . 1
A '' C B ' A C ' B
A'B B ' C C' A
. . 1
A ' C B ' A C' B
nên
. Do B’, C’ lần lượt thuộc cạnh CA, AB nên A’’ nằm ngo|i cạnh BC.
Vậy
v| A’, A’’ nằm ngo|i cạnh BC suy ra
A'' A'
. Do đó A’, B’, C’ thẳng
hàng
+ Trường hợp 2: Trong 3 điểm A’, B’, C’ không có điểm thuộc cạnh tam gi{c ABC được
chứng minh tương tự.
II. Một số ví dụ minh họa
Ví dụ 1. Cho hình thang ABCD có AB//CD. Gọi O l| giao điểm của hai đường chéo AC v|
BD. Gọi M, N, P lần lượt l| trung điểm của AB, BC, AD. Gọi E l| trung điểm của PN.
Chứng minh rằng ba điểm M, O, E thẳng h|ng.
Phân tích tìm lời giải
Trên sở hình vẽ v| c{c yếu tố trung điểm ta nhận thấy nếu gọi K l| trung điểm
của CD thì tứ gi{c MNKP l| nh bình h|nh, khi đó ba điểm M, O, E thẳng h|ng. Để
được M, O, E ta cần chỉ ta được M, K, O thẳng h|ng. Do O l| giao điểm của hai đường
chéo nên ta thấy c{c tam gi{c đồng dạng. Do đó rất tự nhiên ta nghĩ đến chứng minh
0
KOM 180
.
Lời giải
Gọi K l| trung điểm của CD. Khi đó trong tam gi{c ABD có M v| P l| trung điểm của AB
v| AD nên PM l| đường trung bình, do đó PM//BD và
1
PM BD
2
.
Từ đó suy ra tứ gi{c MNKP l| hình bình
h|nh, do đó hai đường chéo NP v| MK cắt
nhau tại E hay ba điểm M, K, E thẳng h|ng .
Dễ thấy hai tam gi{c OAB v| OCD đồng
dạng nên ta được
OA AB
OC CD
. M| lại có
E
O
P
K
N
M
D
C
B
A
Nguyễn Công Lợi
TÀI LIU TOÁN HC
4
11
AM AB,CK CD
22

nên ta được
OA AM
OC CK
.
Xét hai tam giác OAM và OCK có
OAM OCK
OA AM
OC CK
nên ta được
OAM OCK
.
Từ đó suy ra
AOM COK
.
Mà ta có
0
AOM MOC AOC 180
nên ta được
0
MOK COK MOC AOM MOC 180
Do đó ba điểm M, O, K thẳng h|ng. Từ đó dẫn đến ba điểm M, O, E thẳng h|ng.
Ví dụ 2. Cho tam gi{c ABC nội tiếp đường tròn (O). Gọi M l| một điểm tuỳ ý thuộc đường
tròn (O). Gọi
1 1 1
A ; B ;C
theo thứ tự l| hình chiếu của M trên BC, CA, AB. Chứng minh ba
điểm
1 1 1
A ; B ;C
thẳng h|ng.
Phân tích tìm lời giải
Trên cơ sở hình vẽ v| giả thiết của b|i to{n ta nhận thấy c{c tứ gi{c nội tiếp. Điều
n|y cho ta c{c góc nội tiếp bằng nhau. Do đó từ yêu cầu chứng minh ba điểm
1 1 1
A ; B ;C
thẳng h|ng ta nghĩ đến chứng minh
0
1 1 1 1
C A B BA B 180
. Muốn vậy ta cần chỉ ra được
1 1 1 1
C A B B A C
.
Lời giải
Không mất tính tổng qu{t giả sử điểm M thuộc cung
nhỏ
BC
.
Ta có
0
11
BC M BA M 90
nên tứ gi{c
11
MA C B
nội tiếp.
Do đó ta được
1 1 1
BA C BMC
. Lại có
0
11
MA C MB C 90
nên tứ gi{c
11
MA CB
nội tiếp. Do đó ta được
1 1 1
CA B CMB
Mặt kh{c ta lại có
0
11
BAC BMC BAC B MC 180
nên
11
BMC B MC
Từ đó ta được
11
B MC C MB
. Kết hợp c{c kết quả trên ta được
1 1 1 1
C A B B A C
Từ đó suy ta
0
1 1 1 1 1 1 1 1
C A B BA B B A C BA B 180
nên ba điểm
1 1 1
A ; B ;C
thẳng h|ng
Nhận xét: Đường thẳng chứa ba điểm
1 1 1
A ; B ;C
gọi là đường thẳng Simsơn của tam giác ABC ứng
với điểm M. Nếu M trùng với đỉnh của tam giác ABC thì đường thẳng Simsơn chính là đường cao
tương ứng.
M
C
1
B
1
A
1
O
C
B
A
Nguyễn Công Lợi
TÀI LIU TOÁN HC
5
dụ 3. Cho tam gi{c ABC nhọn nội tiếp đường tròn (O). Điểm M bất kỳ trên cung nhỏ
BC. Gọi E, F thứ tự l| c{c điểm đối xứng của M qua AB, AC. Gọi H l| trực t}m trực t}m
ABC. Chứng minh rằng E, H, F thẳng h|ng.
Phân tích tìm lời giải
Trên cơ sở hình vẽ, tính tính đối xứng v| c{c tứ gi{c nội tiếp ta suy ra được c{c cặp
góc bằng nhau như
BHA' BEA
,
EHB EAB MAB
hay
A ' HC ABC
CHF MAC
.
Do đó để chứng minh ba điểm E, H, F thẳng h|ng ta đi chứng minh
0
EHB BHA ' A' HC CHF 180
.
Lời giải
Gọi B’ l| giao điểm của BH v| AC, A’ l|
giao điểm của AH v| BC. Khi đó tứ gi{c
HA’CB’ nội tiếp nên
BHA ' A ' CB' BCA AMB BEA
.
Từ đó ta được tứ gi{c AHBE nội tiếp nên
suy ra
EHB EAB MAB
. Hoàn toàn
tương tự ta có
A ' HC ABC
CHF MAC
.
Từ đó ta được
0
EHB BHA ' A' HC CHF MAB ACB ABC MAC ABC BAC ACB 180
Suy ra
0
EHF 180
nên ba điểm E, H, F thẳng h|ng.
Nhận xét: Đường thẳng đi qua 3 điểm E, H, F nói trên có tên là đường thẳng Steiner ứng với
điểm M.
dụ 4. Cho tứ gi{c ABCD nội tiếp đường tròn (O; R). C{c tia AB, DC cắt nhau tại M, c{c
tia AD, BC cắt nha tại N. Đường tròn ngoại tiếp tam gi{c MBC cắt MN tại K kh{c M. Gọi T
l| giao điểm của AC v| BD. Chứng minh rằng ba điểm O, T, K thẳng h|ng.
Phân tích tìm lời giải
Quan s{t hình vẽ ta nhận thấy OK v| TK cùng vuông góc với MN. Do đó ta hướng
đến sử dụng quan hệ vuông góc để chứng minh ba điểm thẳng h|ng. Ta gọi S l| giao điểm
của đường tròn ngoại tiếp tam gi{c ACM với MT. C{c tứ gi{c AMCS v| ABTS nội tiếp nên
22
MT.TS R OT
22
MT.MS OM R
.
Từ đó
2 2 2 2
MT OM OT 2R
. Ho|n to|n tương tự ta cũng được
2 2 2 2
NT ON OT 2R
.
Do đó suy ra
2 2 2 2
MT NT OM ON
nên
OT MN
. Như vậy b|i to{n sẽ được chứng
minh nếu ta chỉ ra được
OK MN
.Muốn vậy ta cần chỉ ra được
0
OKM 90
.
Lời giải
F
H
E
C'
B'
A'
A
B
C
O
M
Nguyễn Công Lợi
TÀI LIU TOÁN HC
6
Gọi S l| giao điểm của đường tròn ngoại
tiếp tam gi{c ACM với MT. Khi đó tứ gi{c
AMCS nội tiếp đường tròn nên dễ d|ng suy
ra được
22
MT.TS AT.TC R OT
MSA MCA
,
MCA MBD
nên ta được
MBD MSA
. Do đó tứ gi{c ABTS nội tiếp
đường tròn, do đó ta được
22
MT.MS OM R
. Từ đó ta được
2 2 2
MT.MS MT.TS OM OT 2R
Suy ra
2 2 2 2
MT OM OT 2R
.
Tương tự ta cũng được
2 2 2 2
NT ON OT 2R
Do đó ta được
2 2 2 2
MT NT OM ON
. Từ đó ta được
OT MN
.
Mặt kh{c ta lại có
MBC ADC
CKN MBC
nên ta được
ADC CKN
Từ đó suy ra tứ gi{c DCKN nội tiếp đường tròn, do đó
DKN DCN
M| ta lại
DCN MAD
nên ta được
DKN MAD
, suy ra tứ gi{c AMKD nội tiếp đường tròn. Nên ta
được
AKM ADM CKN
.
Do đó
0
AOC AKC 2ADM AKC AKM CKN AKC 180
. Suy ra t gi{c AOCK nội
tiếp đường tròn. M| ta có
OA OC
nên
OA OC
, suy ra
AKO OKC
Do đó
0
OKM AKO AKM 90
hay
OK MN
. Như vậy ta có
OT MN
OK MN
nên
OT v| OK trùng nhau. Vậy ba điểm O, T, K thẳng h|ng.
dụ 5. Cho hình vuông ABCD hai đường chéo AC v| BD cắt nhau tại O. Trên cạnh
AB lấy điểm M sao cho
1
AM AB
3
. Đường thẳng qua D v| vuông góc với đường thẳng
MO cắt AC tại E. Gọi F l| giao điểm của MO v| CD. Chứng minh rằng ba điểm B, E, F
thẳng h|ng.
Phân tích tìm lời giải
Lấy K l| trung điểm của DF khi đó ta nhận thấy OK song song với BF. Để chứng
minh ba điểm B, E, F thẳng h|ng ta cần chỉ ra được EF vuông góc với OK. Muốn vậy ta
cần chứng minh EF l| đường trung bình của tam gi{c COK hay đi chứng minh E l| trung
điểm của OC.
Lời giải
S
K
O
T
N
M
D
C
B
A
Nguyễn Công Lợi
TÀI LIU TOÁN HC
7
Gọi H l| giao điểm của MO v| DE, khi đó ta được
HO DE
tại H, do đó tam gi{c OHE vuông tại H. Từ đó
ta được
0
HOE OEH 90
, mà ta có
0
MOA BOM 90
HOE MOA
nên ta suy ra được
OEH BOM
.
Ta lại có
11
MBO ABC; DAE DAB
22

.
Xét hai tam giác MBO và DAE có
MBO DAE
BOM AED
nên
MBO DAE
Do đó ta được
BO MB
AE AD
. Ta có
AM MB AB
1
AM AB
3
nên ta được
2
MB AB
3
,
suy ra
2
MB AD
3
. Do đó ta được
MB 2
AD 3
. Mà ta có
AE AO OE
OA OB
nên ta
được
2
BO OB OE OB 2OE
3
. Do đó
11
OE OB OC
22

, nên E l| trung điểm của
OC
Xét hai tam giác COF và AOM có
FOC MOA
,
OA OC
OCF OAM
Do đó ta được
COF AOM
nên
CF AM
. Mà
1
AM AB
3
nên
12
CF CD FD CD
33
.
Gọi K l| trung điểm của FD, khi đó ta được
11
FK KD FD CD
23
Trong tam gi{c BDF có O l| trung điểm của BD v| K l| trung điểm của FD nên OK l|
đường trung bình của tam gi{c DBF. Do đó OK//BF. Chứng minh ho|n to|n tương tự ta
được EF//OK
Do đó theo tiên đề Ơclit thì BF v| EF trùng nhau hay ba điểm B, E, F thẳng hàng.
Ví dụ 6. Cho hình vuông ABCD . Trên tia đối của tia CB lấy điểm E, trên tia đối của tia DA
lấy điểm F sao cho
AF BE
. Vẽ EH vuông c với BF lại H. Trên tia đối của tia EH lấy
điểm K sao cho
EK BF
. Chứng minh rẳng ba điểm A, C, K thẳng h|ng.
Lời giải
Kẻ KM vuông góc với AB tại M. Gọi N l| giao điểm của EF với KM. Trong tứ gi{c ABEF có
BE//AF và
BE AF
nên tứ gi{c ABEF l| hình bình h|nh. Lại có
0
ABF 90
nên ABEF là
hình chữ nhật. Từ đó ta được
0
BEN 90
. Tứ gi{c BENM có
0
BMN MBE BEN 90
nên
tứ gi{c BENM l| hình chữ nhật.
H
E
F
K
M
O
D
C
B
A
Nguyễn Công Lợi
TÀI LIU TOÁN HC
8
Từ đó
0
MNE 90
nên
0
ENK 90
.
Xét hai tam giác vuông EBF và NEK có
BF EK
EBF NEK
. Do đó ta được
EBF NEK
,
suy ra
BE EN, EF NK
Hình chữ nhật BENM
BE EN
nên tứ gi{c BENM l| hình vuông. Do
đó suy ra
BM MN
.
Mặt kh{c
AB NK EF
. Nên ta được
MA MB AB MN NK MK
.
Tam gi{c AMK vuông tại M có
MA MK
nên nó
là tam giác vuông cân. Suy ra
0
MAK 45
Mặt kh{c
0
BAC 45
. Như vậy hai tia AK v| AC trùng nhau hay ba điểm A, C, K thẳng
hàng.
dụ 7. Cho tam giác ABC
AB AC BC
. Gọi AD, BE, CF l| c{c đường ph}n gi{c
trong của tam gi{c ABC. Gọi G, I, K, H lần lượt l| điểm đối xứng của B, A, C, A qua AD,
BE, AD, CF. Lấy điểm M trên đoạn CK sao cho
BI GB
CI CM
. Chứng minh rằng ba điểm G, I,
M thẳng h|ng.
Phân tích tìm lời giải
Từ c{c giả thiết v|
BI GB
CI CM
ta suy ra được
BG BI
CK CH
nên
BGI CKH
. Từ đó
ta được GI//HK. Như vậy để chứng minh ba điểm G, I, M thẳng h|ng ta cần chỉ ra được
MI//KH. Muốn có được điều đó ta đi chứng minh
CM CI
CK CH
.
Lời giải
Ta có AD, BE, CF l| c{c đường ph}n gi{c
trong của tam gi{c ABC. Gọi G, I, K, H lần
lượt l| điểm đối xứng của B, A, C, A qua
AD, BE, AD, CF. Khi đó ta được
AG AB,G AC; AB BI, I BC
AK AC, K AB;CH AC, H BC
Trong tam giác ACK có
AB AG
AK AC
nên suy
ra được BG//CK. Do đó ta được
BG AG
CK AC
GBC KCN
.
H
K
F
E
N
M
D
C
B
A
M
K
H
I
G
F
E
D
C
B
A
Nguyễn Công Lợi
TÀI LIU TOÁN HC
9
Do
AG AB BI, AC CH
nên
BG BI
CK CH
Xét hai tam giác BGI và CKH có
BG BI
CK CH
GBI KCH
nên
BGI CKH
Từ đó ta được
BIG CHK
nên suy ra GI//HK. Do
BI GB
CI CM
BG BI
CK CH
nên ta được
CM BG
CI BI
BG CK
BI CH
. Điều n|y dẫn đến
CM CK CM CI
CI CH CK CH
. Trong tam giác CHK có
CM CI
CK CH
nên ta được MI//HK. Từ đó ta có GI//KH v| MI//HK nên hai đường thẳng GI v|
MI trùng nhau. Do đó ba điểm G, I, M thẳng h|ng.
dụ 8. Cho tứ gi{c ABCD. C{c đường thẳng AB, CD cắt nhau tại M v| c{c đường thẳng
AD, BC cắt nhau tại N. Gọi I, J, K lần lượt l| trung điểm của AC, BD, MN. Chứng minh
rằng ba điểm I, J, K thẳng h|ng.
Phân tích tìm lời giải
Trên cở sở giả thiết v| hình vẽ của b|i to{n ta nhận thấy nếu lấy
1 1 1
A , B ,C
lần lượt
l| trung điểm của NB, NA, AB txuất hiện c{c bộ ba điểm thẳng h|ng nên ta nghĩ đến
định Menelaus. Do đó ý tưởng đầu tiên đchứng minh ba điểm I, J, K thẳng h|ng đó l|
đi chứng minh
1 1 1
1 1 1
IC KB JA
. . 1
IB KA JC
. Ngo|i ra ta lại thấy nếu gọi K’ l| giao điểm của IJ v| MN
m| ta chứng minh được
NIJ MIJ
SS
thì suy ra được hai điểm K v| K’ trùng nhau.
Lời giải
Cách 1: Gọi
1 1 1
A , B ,C
lần lượt l| trung
điểm của NB, NA, AB. Ta có
1
AK
đường trung bình của tam gi{c NBM nên
ta được
1
AK
// BM. Ta có
1
BK
l| đường
trung bình của tam gi{c NAM nên ta được
1
BK
// BM. Theo tiên đề Ơclit ta được hai
đường thẳng
1
AK
1
BK
trùng nhau hay
ba điểm
1
A
,
1
B
, K thẳng h|ng. Như vậy
11
K A B
..
Chứng minh ho|n to|n tương tự ta được
11
J A C
11
I B C
Ta có
1 1 1 1 1 1
1 1 1 1 1 1
IC BC; IB CN; KB AM; KA BM; JA DN; JC AD
2 2 2 2 2 2
Xét tam gi{c NAB với M thuộc AB, C thuộc B, D thuộc NA v| ba đim M, C, D thẳng
hàng.
C
1
B
1
A
1
A
B
C
D
M
N
J
K
I
Nguyễn Công Lợi
TÀI LIU TOÁN HC
10
[p dụng định lí Menelaus ta có
AM BC DN
. . 1
MB CN AD
Suy ra
BC AM DN
. . 1
CN BM DA
hay
1 1 1
1 1 1
IC KB JA
. . 1
IB KA JC
Trong tam giác
1 1 1
A B C
11
K A B
,
11
J A C
,
11
I B C
1 1 1
1 1 1
IC KB JA
. . 1
IB KA JC
.
Như vậy theo định lí Menelaus thì ba điểm I, J, K thẳng h|ng.
Cách 2: Ta có
NAI MAC NBJ NBD BIJ BDI ABI ABC
1 1 1 1
S S ; S S ; S S ; S S
2 2 2 2
V| lại có
MAC NCD ACD NBD NCD BCD BDI ABD ABID
S S S ; S S S ; S S S
. Do đó ta được
NIJ NAB NAI NBJ BIJ ABI NAB MAC NBD BDI ABC
NAB NCD ACD NCD BCD ABD ABID ABC
NAB NCD ACD ADC BCD ABD ABID
ABCD ABCD ABCD ABCD ABCD
1 1 1 1
S S S S S S S S S S S
2 2 2 2
1 1 1 1
S S S S S S S S
2 2 2 2
1 1 1
S S S S S S S
2 2 2
1 1 1 1
S S S S S
2 2 4 4
Chứng minh tương tự ta cũng được
MIJ ABCD
1
SS
4
. Từ đó suy ra
NIJ MIJ ABCD
1
S S S
4

.
Gọi K’ l| giao điểm của IJ v| MN. Gọi khoảng c{ch tứ M, N đến IJ lần lượt l|
12
h , h
. Khi đó
ta được
NIJ MIJ 1 2 1 2
11
S S h .IJ h .IJ h h
22
. Từ đó
12
11
h .JK ' h .JK '
22
hay
JMK' JNK'
SS
nên ta được
K' M K' N
hay K’ l| trung điểm của MN. Do đó hai điểm K v| K’ trùng
nhau. Vậy ba điểm I, J, K thẳng h|ng.
dụ 8. Cho hình thang ABCD AB//CD. Trên c{c cạnh AD v| BC lấy lần lượt c{c điểm
M, N sao cho
AM CN
AD BC
. Đường thẳng qua M song song với AC cắt BD tại P v| cắt CD tại
K. Gọi I l| trung điểm của MN, O l| giao điểm của AC v| BD. Chứng minh rằng ba điểm
O, I, K thẳng h|ng.
Phân tích tìm lời giải
Gọi Q l| giao điểm của KN v| AC, S l| giao điểm của OK v| PQ. Dễ thấy tứ gi{c
KPOQ là hình bình h|nh nên S l| trung điểm của PQ. Như vậy để chứng minh ba điểm O,
I, K thẳng h|ng ta cần chứng minh được MN song song với PQ.
Lời giải
Nguyễn Công Lợi
TÀI LIU TOÁN HC
11
Gọi Q l| giao điểm của KN v| AC, S l|
giao điểm của OK v| PQ. Trong tam gi{c
ACD có MK//AC nên theo định lí Talets ta
AM CK
DM CD
. M| theo giả thiết ta có
AM CN
DM BC
nên ta được
CK CN
CD BC
Trong tam giác BCD có
CK CN
CD BC
nên theo
định lí Talets đảo ta được KN//BD.
Do đó tứ gi{c POQK l| hình bình h|nh, suy ra S l| trung điểm của PQ. Trong tam gi{c
DAO có MP//OA nên
MP DP
OA OD
và trong tam giác DOC có PK//OC nên
PK DP
OC OD
. Do đó
MP PK MP NQ
OA OC PK QK
nên suy ra PQ//MN. Gọi S’ l| giao điểm của KI v| MN. Chứng
minh tương tự ta được
PS' S ' Q KS'
MI IN KI

Mà ta có
IN IM
nên suy ra
PS' QS'
. Điều n|y dẫn đến hai điểm S v| S’ trùng nhau, do
đó ba điểm K, I, S thẳng h|ng. M| ba điểm K, S, O thẳng h|ng nên suy ra bốn điểm S, K, I,
O thẳng h|ng.
Vậy ba điểm O, I, K thẳng h|ng. Ta có điều phải chứng minh.
dụ 10. Cho t gi{c ABCD. Lấy c{c điểm E v| F trên c{c cạnh AB v| CD sao cho
EB FC
AB CD
. Gọi I l| trung điểm của EF, H l| trung điểm của AD, K l| trung điểm của BC.
Chứng minh rằng ba điểm H, I, K thẳng h|ng.
Phân tích tìm lời giải
Nhận thấy tứ gi{c PEQF l| hình bình h|nh nên I l| trung điểm của PQ. Để chứng
minh ba điểm H, I, K thẳng h|ng ta gọi K’ l| giao điểm của HI với BC v| chứng minh K’ l|
trung điểm của BC.
Lời giải
D
C
B
Q
P
N
M
K
O
I
S'
S
A
Nguyễn Công Lợi
TÀI LIU TOÁN HC
12
Vẽ EP//AD với P thuộc BH, vẽ FQ//AD với
Q thuộc CH. Trong tam gi{c ABH có
EP//AH nên theo định lí Talets ta có
EP EB
AH AB
. Trong tam giác CDH có FQ//HD
nê theo định lí Talets ta có
FQ FC
HD CD
Mà ta có
EB FC
AB CD
nên ta được
EP FQ
AH HD
.
Mà ta có
AH HD
nên suy ra
EP FQ
Mặt kh{c ta có EP//FQ nên tứ gi{c PEQF l| hình bình h|nh. Do I l| trung điểm của EF nên
I cũng l| trung điểm của PQ.
Trong tam giác ABH có EP//AH nên
EB BP
AB BH
và trong tam giác CDH có FQ//HD nên
FC CQ
CD CH
Mà ta có
EB FC
AB CD
nên
BP CQ
BH CH
. Trong tam giác HBC có
BP CQ
BH CH
nên suy ra PQ//BC.
Gọi K’ l| giao điểm của HI v| BC. Trong tam gi{c HBK’ có IP//K’B nên ta có
IP IH
K ' B HK '
Trong tam gi{c HCK’có IQ//K’C nên ta có
IQ HI
K ' C HK '
. Từ đó ta được
IP IQ
BK ' CK '
, mà ta có
IP IQ
nên suy ra
BK' CK'
. Điều n|y dẫn đến hai điểm K v| K’ trùng nhau. Vậy ba
điểm H, I, K thẳng h|ng.
Ví dụ 11. Cho hình thang vuông ABCD có
0
A D 90
. Đường trong đường kính CD cắt
AB tại M v| N (M nằm giữa N v| B). Đường thẳng qua A song song với MD cắt đường
thẳng qua B song song với MC tại E. Chứng minh rằng ba điểm C, E, D thẳng h|ng.
Phân tích tìm lời giải
Nhận thấy tứ gi{c MLEF l| hình chữ nhật. Khi đó tam gi{c FCE v| MCD
EFC CMD
FC EF
MC MD
nên
FCE MCD
do đó ta được
FCE MCD
. Điều n|y dẫn
đến hai tia DE v| DC trùng nhau, tức l| điểm E, C, D thẳng h|ng.
Lời giải
F
E
H
I
K'
K
Q
P
C
D
B
A
Nguyễn Công Lợi
TÀI LIU TOÁN HC
13
Gọi L l| giao điểm của AE v| MD, gọi F l| giao
điểm của MC v| BE. Do tam gi{c MCD nội tiếp
đường tròn đường kính CD nên ta được
0
CMD 90
. Do AE//MC và
CM MD
nên ta
được
AE DM
. Từ đó suy ra
0
ALM MLE 90
Do BE//MD và
CM MD
nên ta được
BE MC
.
Từ đó suy ra
0
BFC EFM 90
. Xét hai tam giác
BMC và ADM có
MBC DAM
BMC ADM
.
Suy ra
BMC ADM
nên
MC BC
MD MA
BCF AML
Xét hai tam giác BCF và AML có
BCF AML
Suy ra
BCF AML
nên
BC FC
AM ML
. Tứ gi{c MLEF có
0
LMF EFM MLE 90
nên là
hình chữ nhật. Do đó ta được
EF ML
. Do vậy
FC FD BC MC
EF ML MA MD
nên
FC EF
MC MD
Xét hai tam giác FE và MCD có
EFC CMD
FC EF
MC MD
Suy ra
FCE MCD
nên ta được
FCE MCD
. Điều n|y dẫn đến hai tia CE v| CD trùng
nhau
Vậy ba điểm E, C, D thẳng h|ng.
dụ 12. Cho tam gi{c ABC nhọn ba đường cao AD, BE, CF cắt nhau tại H. Vẽ HI
vuông góc với EF tại I, HK vuông góc với DE tại K. Gọi giao điểm của IK v| AD l| M, giao
điểm của FM v| DE l| N. Gọi S l| điểm đối xứng với B qua D. Chứng minh rằng ba điểm
A, N, S thẳng hàng.
Phân tích tìm lời giải
Do c{c tứ gi{c FAEH v| ABDE nội tiếp nên
HEF BED
. Từ đ}y ta được
HIE HKE
nên tam gi{c EIK c}n tại F v| tam gi{c HIK c}n tại H. Từ đó ta được c{c tứ
gi{c FIMH v| HMNK nội tiếp.
Để chứng minh ba điểm N, A, S thẳng h|ng ta cần chứng minh
MAN DAS
hay ta
chứng minh
AMN ADS
. Muốn vậy ta cần
AM MN
AD DS
. Chú ý l| từ c{c tứ gi{c nội
tiếp trên ta suy ra được tam gi{c HFN c}n nên MF//BD, do đó ta được
AM MF
AD BD
. Kết hợp
F
E
L
N
M
D
C
B
A
Nguyễn Công Lợi
TÀI LIU TOÁN HC
14
với
MF MN
BD DS
thì ta được
AM MN
AD DS
. Như vậy ta thể trình bày bài toán
như sau.
Lời giải
Tứ gi{c FAEH có
0
AFH AEH 90
nên tứ
gi{c FAEH nội tiếp. Suy ra
HEF FAH
Tứ gi{c ABDE có
0
ADB AEB 90
nên tứ
gi{c ABDE nội tiếp. Suy ra
BAD BED
Từ đó ta được
HEF BED
Xét hai tam giác vuông HIE và HKE có EH
chung và
HEI HEK
nên
HIE HKE
Từ đó suy ra
IE EK
HI HK
. Suy ra
tam gi{c EIK c}n tại F v| tam giác HIK cân
tại H
Duy ra
0 0 0
1
KIE 180 IEK 90 IEH KIE FAH 90
2
. Mà ta có
0
NHF FAH 90
, do
đó ta được
KIE MHF
nên tứ gi{c FIMH nội tiếp đường tròn. Nên suy ra
0
HMF HIF 90
Tứ gi{c HMNK có
0
HMN HKN 90
nên tứ gi{c HMNK nội tiếp đường tròn.
Ta có
HFN HIK, HNM HKI, HIK HKI HFN HNM
Từ đó suy ra tam gi{c HFN c}n tại H nên
HF HN
. Mà ta có
HM FN
nên HM l| đường
trung trực của tam gi{c HFN. Ta có
FM AD, BD AD
nên ta được FM//BD.
Trong tam gi{c ABD có FM//BD nên theo định lí Talets ta có
AM MF
AD BD
Lại có
MF MN
BD DS
nên ta được
AM MN
AD DS
Xét hai tam giác AMN và ADS có
AMN ADS
AM MN
AD DS
nên
AMN ADS
Từ đó suy ra
MAN DAS
, suy ra hai tia AN v| AS trùng nhau. Vậy ta có ba điểm A, N, S
thẳng h|ng.
Ví dụ 13. Cho hình thoi ABCD có
0
BAD 60
. Đường thẳng d đi qua C cắt c{c cạnh AB, AD
lần lượt tại M, N kh{c A. Đường thẳng d’ đi qua A v| song song với BD cắt c{c đường
thẳng BN, Dm lần lượt tại E v| F. Gọi P l| giao điểm của BN với DM. Chứng minh rằng
c{c trọng t}m của c{c tam gi{c ABD, AME, AFN thẳng h|ng.
Phân tích tìm lời giải
K
I
H
S
N
M
F
E
D
C
B
A
Nguyễn Công Lợi
TÀI LIU TOÁN HC
15
Trên sở hình vẽ v| giả thiết của b|i to{n ta nhận định được tam gi{c AFN đều.
Điều n|y dẫn đến
ANB AFD
hay tứ gi{c ADNF nội tiếp v| nếu gọi J l| trọng t}m tam
giác AFN thì J t}m đường tròn ngoại tiếp tứ gi{c ABNF nên
JA JP
. Ta lại thấy tam
gi{c MAE đều nên nếu gọi I l| trọng t}m tam gi{c AME thì I l| t}m đường tròn ngoại tiếp
tứ gi{c EAPM nên suy ra
IA IP
. Gọi O l| trọng t}m tam gi{c ABD. Để chứng minh ba
điểm I, J, O thẳng h|ng ta chỉ cần chứng minh được
OA OP
, tức l| ta cần chứng minh O
l| t}m đường tròn ngoại tiếp tứ gi{c ABPD.
Lời giải
Tam giác ABD cân
0
BAD 60
nên
tam gi{c ABD đều. Trong tam gi{c
MAF BD//AF nên ta
BD MB
AF MA
.
Trong tam giác MAN BC//NA nên
ta có
BC MB
AN MA
Suy ra
BD BC
AF AN
. ta
BD BC
nên ta được
AF AN
Ta lại có
0
FAN ADB 60
và tam giác AFN cân, có
0
AFN 60
nên l| tam gi{c đều.
Từ đó ta được
AF AN FN
0
AFN FNA 60
Do đó ta được
ABN ADF
nên suy ra
ANB AFD
. Từ đó suy ra tứ gi{c ADNF nội tiếp
đường tròn. Gọi J l| trọng t}m tam gi{c AFN, khi đó J l| t}m đường trong ngoại tiếp tam
gi{c AFN. Do đó J l| t}m đường tròn ngoại tiếp tứ gi{c ABNF, suy ra
JA JP
Ta có
0
NPF NAF 60
nên ta được
0
EPM NPF 60
M| ta lại có
0
EAM ABD 60
nên ta được
0
EAM EPM 60
, do đó tứ gi{c EAPM nội tiếp
đường tròn. Từ đó suy ra
0
AEM APN ANF 60
.
Trong tam giác MAE có
0
MAE AEM 60
nên l| tam gi{c đều. Gọi I l| trọng t}m tam gi{c
AME. Khi đó I l| t}m đường tròn ngoại tiếp tứ gi{c EAPM, suy ra
IA IP
.
Ta
0
NPD ABD 60
nên tứ gi{c ABPD nội tiếp đường tròn. Gọi O l| trọng t}m tam
gi{c ABD, khi đó O l| t}m đường tròn ngoại tiếp tứ gi{c ABPD, nên ta được
OA OP
Như vậy ta có
JA JP
,
IA IP
OA OP
, suy ra ba điểm J, I, O thuộc đường trung trực
của đoạn thẳng AP. Vậy ba điểm O, I, J thẳng h|ng.
dụ 14. Cho hình vuông ABCD O l| giao điểm của hai đường chéo. Trên tia đối của
tia CD lấy điểm E sao cho
1
CE CB
2
. Vẽ DM vuông góc với BE tại M. Gọi I l| trung điểm
của OB. Chứng minh rằng ba điểm A, I, M thẳng h|ng.
P
K
J
O
I
N'
M'
N
M
F
E
D
C
B
A
Nguyễn Công Lợi
TÀI LIU TOÁN HC
16
Lời giải
Cách 1: Hình vuông ABCD có O l| giao điểm của
hai đường chéo nên O l| trung điểm của AC, BD
v| AC vuông góc với BD tại O. Ta có
OI OI 1
OA OB 2

, m| ta lại có
1
CE CB
2
nên ta được
CE 1
CB 2
. Do đó
OI CE OI OA
OA CB CE CB
Hai tam giác AOI và BCE có
AOI BEC
OI OA
CE CB
nên
AOI BCE
, suy ra
AIO BEC
.
Gọi H l| giao điểm của BC v| DM. Đặt
B
C 2a a 0
Hai tam giác DCH và BCE có
DCH BCE
CDH CBE
nên
DCH BCE
Do đó
DC CH
CD.CE BC.CH
BC CE
. Mà ta có
CD BC
nên
CH CE
, suy ra
1
CH CB
2
.
Do H l| trung điểm của CB nên ta được
2
1
BH CB BH.BC 2a
2
Hai tam giác BMH và BCE có
BMH BCE
MBH CBE
nên
BMH BCE
Do đó ta được
BM BH
BM.BE BH.BC
BC BE
, suy ra
2
BM.BE 2a
.
Trong tam gi{c BCD vuông tại C ta tính được
2
BD 8a
Ta có
1
BI OB
2
1
OB BD
2
nên
1
BI BD
4
, do dó ta được
22
1
BI.BD BD BI.BD 2a
4
Do đó ta được
2
BI BM
BM.BE BI.BD 2a
BE BD
Hai tam giác BIM và BED có
IBM EBD
BI BM
BE BD
nên ta được
BIM BED
Do đó ta được
BIM BED
, mà ta có
AID BED
nên ta được
BIM AID
Từ đó suy ra
0
AID DIM BIM DIM 180
nên
0
AIM 180
. Vậy ba điểm A, I, M thẳng
hàng.
Cách 2: Vẽ AP vuông góc với MD tại P v| AQ vuông góc với MB tại Q. Từ đó suy ra tứ
gi{c APMQ l| hình chữ nhật nên
0
PAQ 90
. Mà ta có
0
DAB 90
nên ta được
.
Hai tam giác vuông DAP và BAQ có
AD AB
nên
DAP BAQ
Do đó
AP AQ
, suy ra tứ gi{c APMQ l| hình vuông. Nên MA l| ph}n gi{c của góc
BMD
.
Gọi H l| giao điểm của BC v| DM. Đặt
AB 2a
, suy ra
CE a
CD 2a
nên
DE 3a
.
Tam giác BCE và CDH có
CBE CDH
nên
BCE DCH
P
Q
O
G
M
H
I
E
D
C
B
A
Nguyễn Công Lợi
TÀI LIU TOÁN HC
17
Do đó ta được
CE BC
CE CH
CH CD
. Mà ta có
1
CE BC
2
nên
1
CH CB
2
nên H là trung
điểm của BC, suy ra
BH a
. Hai tam giác BMH và DME có
BMH DME
MBH MDE
Suy ra
BMH DME
nên ta được
MB BH 1
MD DE 3

Ta có
IB IB IB IB 1
ID IO OD IB OB IB 2IB 3
, do đó
IB MB
ID MD
Vẽ MG l| đường ph}n gi{c của tam gi{c BMD.
Ta có
GB MB IB GB
GD MD ID GD
, suy ra
IB ID IB ID BD
1 IB GB
GB GD GB GD BD
Từ đó suy ra hai điểm I v| G trùng nhau, nên MI l| ph}n gi{c của góc
BMD
Từ đó ta được hai tia MA v| MI trùng nhau, nên ba điểm A, I, M thẳng h|ng.
dụ 15. Cho hình vuông ABCD hai đường chéo cắt nhau tại E. Đường thẳng qua A
cắt cạnh BC v| cắt đường thẳng CD lần lượt tại M v| N. Gọi giao điểm của EM v| BN l| K,
giao điểm của CK v| AB l| F. Chứng minh rằng ba điểm D, M, F thẳng h|ng.
Lời giải
Hình vuông ABCD có hai đường chéo
cắt nhau tại E nên ta được E l| trung
điểm của hai đường chéo v| AC, BD
vuông góc với nhau tại E.
Ta có
EB EC
EBA ECB
. Trên
cạnh AB lấy điểm S sao cho
BS CM
.
Hai tam giác BES và CEM có
BS CM
,
EBS ECM
EB EC
nên
BES CEM
Do đó ta được
ES EM
SEM MEC
. Suy ra tam gi{c SEM c}n tại E.
Lại có
0
SEM SEB BEM MEC BEM BEC 90
.Do đó tam gi{c SEM vuông c}n tại E,
do đó
0
SME 45
. Tam gi{c AND có MC//AD nên theo định lí Talets ta có
AN AD AN AB
MN MC MN SB
Do đó ta được SM//BN, suy ra
0
BKE SME 45
.
Hai tam giác BKE và BDN có
BKE BDN
nên
BKE BDN
Do đó ta được
BK BE
BK.BN BE.BD
BD BN
Hai tam giác BEC và BCD có
EBC CBD
BEC BCD
nên
BEC BCD
K
S
A
B
C
D
E
N
M
F
Nguyễn Công Lợi
TÀI LIU TOÁN HC
18
Do đó ta được
2
BE BC
BC BE.BD
BC BD
. Từ đó suy ra
2
BK BC
BK.BN BC
BC BN
Hai tam giác BKC và BCN có
KBC CBN
BK BC
BC BN
nên
BKC BCN
Do đó ta được
BKC BCN
nên
0
BKC 90
Hai tam giác BCN và FBC có
CBN BFC
BCN FBC
nên
BCN FBC
Do đó ta được
BC CN BC BF AB BF
BF BC CN BC CN CD
Trong tam giác MNC có AB//CN nên ta có
MB AB
MC CN
. Do đó ta được
MB BF
MC CD
.
Hai tam giác MBF và MCD có
MBF MCD
MB BF
MC CD
nên
MBF MCD
Từ đó suy ra
DMF CMD BMF BMD CMD BMD
hay
0
DMF 90
Vậy ba điểm D, M, F thẳng h|ng.
Ví dụ 16. Cho s{u điểm bất kì A, B, C, D, E, F cùng nằm trên một đường tròn. Gọi G, H, K
lần lượt l| giao điểm của c{c cặp đường thẳng AB v| DE, BC v| EF, CD v| AF. Chứng
minh rằng ba điểm G, H, K thẳng h|ng.
Phân tích tìm lời giải
Ta nhận thấy
1
BIF sdBAF sdCDE
2

1
BHF sdBAF sdCDE
2

nên ta được tứ
gi{c BIHF nội tiếp, từ đó suy ra
0
BIH BFH 180
. Như vậy để chứng minh được G, H, K
thẳng h|ng ta cần chứng minh được
0
BIH BIG 180
. Muốn vậy ta cần chỉ ra được
BIG BFH
. Ngo|i ra nếu gọi A’, B’, C’ lần lượt l| giao điểm của AB v| EF, AB v| CD, CD
v| EF. Khi đó ta thấy bộ c{c điểm thẳng h|ng, từ đó ta liên tưởng đến định lí Menelaus.
Lời giải
Cách 1: Gọi I l| giao điểm của hai đường
trong ngoại tiếp tam gi{c GBD v| tam gi{c
KDF(I kh{c D). Khi đó ta có
BIF BID DIF BGD DKF
11
sdAFE sdBCD sdABC sdCDF
22
11
sdCAE sdBCF sdBAF sdCDE
22
Mà ta có
1
BHF sdBAF sdCDE
2

Do đó
BIF BHF
, do đó tứ gi{c BIHF nội tiếp đường tròn. Từ đó suy ra
0
BIH BFH 180
.
Mặt kh{c ta lại có
1
BIG BDG sdBG
2

. Mà ta có
BDG BFE
G
I
K
H
O
D
C
B
A
Nguyễn Công Lợi
TÀI LIU TOÁN HC
19
Nên ta được
0
BIH BIG BIH BFH 180
, suy ra
0
HIG 180
Do đó ba điểm H, I, G thẳng h|ng. Chứng minh tương tự ta được I, H, K thẳng h|ng.
Như vậy bốn điểm H, I, G, K thẳng h|ng. Do đó ba điểm G, H, K thẳng h|ng.
Cách 2: Gọi A’, B’, C’ lần lượt l| giao điểm của AB v| EF, AB v| CD, CD v| EF.
[p dụng định lí Menelaus cho tam gi{c A’B’C’ có ba điểm B, C, H thẳng h|ng ta được
BA ' CB ' HC '
. . 1
BB' CC ' HA '
[p dụng định lí Menelaus cho tam gi{c A’B’C’ với ba điểm D, G, E thẳng h|ng ta được
DB' EC ' GA '
. . 1
DC' EA ' GB '
[p dụng định lí Menelaus cho tam gi{c A’B’C’ với ba điểm A, F, K thẳng h|ng ta được
AA ' KB' FC '
. . 1
AB' KC' FA '
Xét hai tam gi{c A’AF v| A’EB có
AA ' F
chung và
A ' AF A ' EM
Do đó ta được
AAF A’EB
, suy ra
AA ' FA '
AA '.BA ' EA '.FA '
EA ' BA '
Ho|n to|n tương tự ta được
AB'.BB' CB'.DB'
FC'.EC' CC'.DC'
Từ đó ta được
HC' GA ' KB '
. . 1
HA ' GB ' KC'
.
Trong tam gi{c A’B’C’ có c{c điểm G, H, K thỏa mãn
HC' GA ' KB '
. . 1
HA ' GB ' KC'
, nên theo định lí
Menelaus ta được ba điểm G, H, K thẳng h|ng.
dụ 17. Cho điểm A nằm ngo|i đường tròn (O). Từ điểm A vẽ hai tiếp tuyến AB v|
AC(B, C l| hai tiếp điểm) v| c{t tuyến ADE với đường tròn (O) sao cho tia AD nằm giữa
hai tia AO v| AB. Đường thẳng qua D v| song song với BE cắt BC, AB lần lượt tại v| Q.
Gọi K đối xứng với B qua E. Chứng minh rằng ba điểm A, P, K thẳng h|ng.
Lời giải
Gọi H, I lần lượt l| giao điểm của BC với
OA, DE. Ta có AB, AC l| hai tiếp tuyến
với đường tròn (O) nên
AB AC
và AO
l| ph}n gi{c của góc
BAC
. Do đó AO l|
đường cao của tam gi{c ABC. Xét hai
tam giác ABD và AEB có
ABD AEB
BAD
chung, suy ra
AEB ABD
.
I
x
K
Q
P
H
O
E
D
C
B
A
Nguyễn Công Lợi
TÀI LIU TOÁN HC
20
Từ đó
2
AB AD.AE
Trong tam gi{c ABO vuông có BH l| đường cao nên
2
AB AH.AO
Từ đó ta được
AH AD
AD.AE AH.AO
AE AO
. Từ đó
AHD AEO
nên ta được
AHD AEO
Do đó tứ gi{c OEDH nội tiếp đường tròn, suy ra
OHE ODE
Tam giác ODE có
OD OE
nên c}n tại O, suy ra
Từ đó ta được
. Ta có
0
OHE EHI AHD IHD 90
nên ta được
EHI IHD
, do
đó HI l| tia ph}n gi{c của góc
HED
. Gọi Hx l| tia đối của tia HE, khi đó ta có
xHA AHD OHE
Do đó HA l| đường ph}n gi{c của
HED
. Từ đó ta được
ID AD
ED AE
Trong tam gi{c ABE có DQ//BE nên theo định lí Talets ta có
DQ AD
BE AE
Trong tam giác IBE có BE//PD nên ta có
DP ID
BE IE
. Từ đó ta được
DQ DP
BE BE
nên
DQ DP
.
Trong tam giác ABE có DQ//BE nên ta có
AQ QD
AB BE
. Do đó ta được
AQ 2DQ PQ
AB 2BE BK

Hai tam giác APQ và AKB có
AQ PQ
AB BK
AQP ABK
nên
APQ AKB
Từ đó ta được
nên hai tia AP v| AK trùng nhau. Vậy ba điểm A, P, K thẳng
hàng.
Ví dụ 18. Cho điểm A nằm ngo|i đường tròn t}m O b{n kính R. Vẽ c{c tiếp tuyến AB, AC
với đường tròn (O)(B, C thuộc đường tròn (O)). Vẽ c{t tuyến ADE với đường tròn (O)(D, E
thuộc đường tròn (O)) v| D nằm giữa A, E. Tia AD nằm giữa hai tia AO v| AB. Gọi F l|
điểm đối xứng với D qua AO, H l| giao điểm của EF với BC. Chứng minh rằng A, H, O
thẳng h|ng.
Phân tích tìm lời giải
Gọi H’ l| giao điểm của OA với BC. Để chứng minh A, H, O thẳng h|ng ta cần
chứng minh được hai điểm H v| H’ trùng nhau. Muốn vậy ta cần chứng minh
0
EH ' F AH ' F AH ' E 180
. Ngo|i ra ta cũng thấy A, O cùng thuộc đường trung trực của
BC nên để chứng minh được A, H, O thẳng h|ng ta cần chứng minh H thuộc đường trung
trực AO hay
HB HC
.
Lời giải
Nguyễn Công Lợi
TÀI LIU TOÁN HC
21
Cách 1: Gọi H’ l| giao điểm của AO v|
BC. Do D v| F đối xứng với nhau qua
AO nên ta được
OF OD R
. Suy ra F
thuộc đường tròn (O) v| có
AH ' D AH ' F
. Do AB, AC l| hai tiếp
tuyến của đường tròn (O) nên ta được
AB AC
v| AO l| tia ph}n gi{c của
góc
BAC
. Do đó tam gi{c ABC c}n tại
A và
AO BC
. Xét hai tam giác ABD
và AEB có
ABD AEB
BAD
chung
Do đó ta được
ABD AEB
nên
2
AB AD
AB AD.AE
AE AB
.
Tam gi{c ABO vuông tại B có BH’ l| đường cao nên
2
AB AH '.AO
.
Từ đó ta được
AD AH '
AD.AE AH '.AO
AO AE
Xét hai tam gi{c ADH’ v| AOE có
AD AH '
AO AE
DAH '
chung nên
ADH' AOE
Từ đó ta được
ADH' AOE
nên tứ gi{c DH’OE nội tiếp. Suy ra
AH ' D OED
OH ' E ODE
Mà ta có
OE OD R
nên tam gi{c ODE c}n tại O, suy ra
OED ODE
Do đó
AH ' D OH' E
, vì vậy ta được
AH ' F OH' E
.
Ta có
0
EH ' F AH' F AH ' E OH' E AH ' E 180
. Điều n|y dẫn đến ba điểm E, H’, F
thẳng h|ng, suy ra hai điểm H v| H’ trùng nhau. Vậy ba điểm A, H, O thẳng h|ng. Ta có
điều phải chứng minh.
Cách 2: Do AB, AC l| tiếp tuyến với đường tròn (O) nên ta được
AB AC
và AO là tia
ph}n gi{c của góc
BAC
. Do đó tam gi{c ABC c}n tại A v|
AO BC
. Do D v| F đối xứng
với nhau qua AO nên ta suy ra
OF OD R
DF AO
. Ta có DF//BC nên tứ gi{c DBCF
l| hình thang, m| từ gi{c DBCF nội tiếp đường trong (O) nên DBCF l| hình thang c}n. Từ
đó ta được
BD CE, BF CD
.
Xét hai tam giác ABD và AEB có
ABD AEB
BAD
chung.
Suy ra
ABD AEB
nên ta được
BD AB
BE AE
.
Ho|n to|n tương tự ta được
ACD AEC
nên suy ra
AC CD
AE CE
.
Xét hai tam giác HBF và HEC có
BHF EHC
BFH HCE
O
H'
H
D
E
F
C
B
A
Nguyễn Công Lợi
TÀI LIU TOÁN HC
22
Do đó ta được
HBF HEC
nên ta được
HB BF
HE CE
Tương tự ta cũng được
HFE HBE
nên suy ra
CF HC
BE HE
Từ đó suy ra
HB BF CD AC AB BD CF HC
HE CE CE AE AE BE BF HE
. Do đó ta được
HB HC
.
Ta có
HB HC
,
AB AC
OB OC
nên ba điểm H, A, O thuộc đường trung trực của
đoạn thẳng BC. Do đó ba điểm H, A, O thẳng h|ng.
Ví dụ 19. Cho đường tròn (O) nội tiếp tam gi{c ABC. Đường tròn (O) tiếp xúc với c{c cạnh
BC, CA, AB lần lượt tại D, E, F. Trên đoạn OD lấy điểm I, vẽ đường tròn tâm I bán kính
ID. Vẽ BG, CH l| c{c tiếp tuyến của đường tròn (I) với G v| H thuộc đường tròn (I). Gọi M
l| giao điểm của BG v| CH, N l| giao điểm của EF v| BC. Chứng minh rằng ba điểm N, G,
H thẳng h|ng.
Phân tích tìm lời gải
Nhận thấy tứ gi{c EFHG nội tiếp đường tròn. Để chứng minh được ba điểm N, H,
G thẳng h|ng ta gọi giao điểm của NG với đường tròn (I) l|
1
H
, giao điểm của NG với
đường tròn ngoại tiếp tứ gi{c EFHG l|
2
H
v| ta đi chứng minh hai điểm
1
H
2
H
trùng
nhau.
Lời giải
Từ tính chất hai tiếp tuyến cắt nhau ta có
AE AF, BG BD BF,CE CH CD, MG MH
Tam gi{c AEF c}n tại A nên
AEF AFE
, suy ra
0
2AEF BAC 180
Trong tam giác ABC có
0
BAC ABC ACB 180
. Do đó ta được
1
AEF ABC ACB
2

Tam gi{c CEH c}n tại C có
0
1
AEH 90 ECH
2

Do đó ta được
0
1
HEF AEH AEF 90 ECH ABC ACB
2
A
B
C
D
E
F
G
O
I
M
N
H
Nguyễn Công Lợi
TÀI LIU TOÁN HC
23
Ho|n to|n tương tự ta được
0
11
MGH MBC MCB; MGF 90 GBF
22
Do đó
0
1
HGF 90 MBC MCB GBF
2
. Suy ra
0
HEF HGF 180
nên tứ gi{c EHGF nội
tiếp
Gọi giao điểm của NG với đường tròn (I) l|
1
H
, giao điểm của NG với đường tròn ngoại
tiếp tứ gi{c EHGF l|
2
H
. Khi đó ta có
1
NGD NDH
nên ta được
2
1
1
NG ND
NG.NH ND
ND NH
Ta cũng có
2
NFG NH E
nên ta được
2
2
NF NG
NE.NF NH .NG
NH NE
NDF NED
nên ta có
2
ND NF
ND NE.NF
NE ND
Do đó ta được
12
NH NH
, suy ra hai điểm
1
H
2
H
trùng nhau. Nên ba điểm H,
1
H
2
H
trùng nhau.
Vậy ba điểm N, G, H thẳng h|ng.
dụ 20. Cho đường tròn (O; R) đường kính AB. Lấy điểm C nằm giữa hai điểm O v|
A. Vẽ đường tròn (I) đường kính BC. Vẽ tiếp tuyến AD v| c{t tuyến ACF với đường tròn
(I)(E nằm giữa A v| F) sao cho tia AO nằm giữa hai tia AD v| AE. Đường thẳng vuông góc
với AB v từ C cắt đường tròn (O) tại N v| P(N v| P thuộc nửa mặt phẳng bờ AB). Giao
điểm của DI v| NB l| S. Gọi J l| trung điểm của SD. Gọi L, T lần lượt l| t}m đường tròn
ngoại tiếp c{c tam gi{c SBC v| SEF. Chứng minh rằng ba điểm J, L, K thẳng h|ng.
Lời giải
t tam giác ADC và ABD
DAC
nên
ADC ABD
. Do đó
2
AD AC
AD AB.AC
AB AD
Tam gi{c ANB vuông tại N NC l| đường cao
nên
2
AN AB.AC
. Từ đó ta được
AN AD
.
Gọi R l| giao điểm của DN v| AS. Hai tam gi{c
vuông DAS NAS AS chung
AD AN
nên
ADS NAS
. Từ đó suy ra
DAS NAS
.
Ta giác AND
AD AN
nên c}n tại A, AR l|
đường ph}n gi{c nên đồng thời l| đường cao của
tam giác AND.
Tam gi{c ADS vuông tại D nên ta có
2
AD AR.AS
. Do đó
AC AR
AR.AS AB.AC
AS AB
.
O
S
L
I
J
R
T
N
P
F
E
D
C
B
A
Nguyễn Công Lợi
TÀI LIU TOÁN HC
24
Xét hai tam giác ACR và ASB có
CAR
chung và
AC AR
AS AB
nên
ACR ASB
Từ đó ta được
ACR ASB
, do đó tứ gi{c CBSR nội tiếp đường tròn. Suy ra L thuộc đường
trung trực của SR. Chứng minh tương tự ta được tứ gi{c RSEF nội tiếp đường tròn, suy ra
T thuộc đường trung trực của đoạn thẳng SR. Mặt kh{c ta tam gi{c RDS vuông tại R
RJ l| đường trung tuyến nên ta được
RJ SJ DJ
. Do đó J cũng thuộc đường trung trực
của SR. Vậy ba điểm J, L, T thuộc đường trung trực của SR nên ba điểm J, L, T thẳng h|ng.
Ví dụ 21. Cho tam gi{c nhọn ABC
AB AC
. Đường tròn (I) đường kính BC cắt AB, AC
lần lượt tại F, E. Giao điểm của BE v| CF l| H, giao điểm của AH v| BC l| D, giao điểm
của EF v| BC l| K. Đường thẳng qua D song song với EF cắt AB tại M. Đường tròn ngoại
tiếp tam gi{c IMK cắt AC tại N. Chứng minh rằng ba điểm M, D, N thẳng h|ng.
Lời giải
Ta
0
BFC BEC 90
nên tam giác ABC
hai đường cao BE v| CF, do đó ta được
AD BC
Tứ gi{c BFHD
0
BFH BDH 90
nên nội
tiếp được đường tròn. Suy ra ra được
DBH DFH
ta
DBH CFE
nên
1
DBH DFE
2
, lại
1
DBH CIE
2
, suy ra ta được
CIE DFE
Từ đó dẫn đến tứ gi{c IFED nội tiếp. Suy ra
IDF IEF
,
IEF IFE
nên ta được
IDF IFK
. Hai tam giác IDF IFK
DIF
chung và
IDF IFK
nên
IDF IFK
Do đó
2
ID IF
IF ID.IK
IF IK
nên
2
ID.IK IC
Gọi N’ l| giao điểm của MD v| AC. Ta có
DN ' C AEF
AEF MBD
nên
DN ' C MBD
Từ đó ta được
DN'C DBM
, suy ra
DN ' DC
DB.DC DM.DN '
DB DM
. Từ đó suy ra
22
2
DM.DN ' DB.DC IB ID IC ID IC ID IC ID IC ID
ID.IK ID ID IK ID ID.KD
Do đó
DM DI
DK DN '
, kết hợp với
MDI KDN '
ta được
DMI DKN'
nên
DMI DKN '
,
suy ra tứ gi{c IMKN’ nội tiếp đường tròn. Suy ra N v| N’ trùng nhau. Vậy ba điểm M, D,
N thẳng h|ng.
K
I
D
H
F
E
N'
N
M
C
B
A
Nguyễn Công Lợi
TÀI LIU TOÁN HC
25
dụ 22. Cho điểm A nằm ngo|i đường tròn (O; R). Vẽ c{c c{t tuyến AEB, ADC với
đường tròn (O)(E, B, C, D thuộc đường tròn (O), E nằm giữa A v| B, D nằm giữa A v| C).
Gọi H l| giao điểm của BD v| CE. Vẽ c{c tiếp tuyến AM, AN với đường trong (O) (M, N l|
c{ tiếp điểm, B v| M nằm cùng nửa mặt phẳng bờ AH). Gọi K l| giao điểm của BN v| CM.
Chứng minh rằng ba điểm A, H, S thẳng h|ng.
Lời giải
Gọi H’ l| giao điểm của MN v| BD, E’ l| giao điểm
của CH’ với đường tròn (O). Ta
0
AMO ANO 90
, nên tứ gi{c AMON nội tiếp đường tròn (I) đường
kính OA. Gọi S l| giao điểm của AH’ với dường tròn
(I). Khi đó dễ d|ng chứng minh được
H' E'.H'C H' M.H' N H' A.H' S
. Từ đó suy ra tứ
gi{c AE’SC nọi tiếp đường tròn, nên ta được
AE ' C ASC
. Gọi F l| giao điểm của OA với MN. Ta
cũng chứng minh được
2
AH '.H 'C AF.AO AN AC.AD
. Do đó tứ gi{c
H’SCD nội tiếp, suy ra
0
H ' SC H ' DC 180
. M| ta lại
BE ' C H ' DC
, từ đó ta suy ra được
0
AE ' C BE ' C 180
. Do đó ba điểm A, E’, B thẳng
h|ng, suy ra hai điểm E v| E’ trùng nhau, H v| H’
trùng nhau.
Vậy ba điểm M, H, N thẳng h|ng. Gọi K’ l| giao điểm của AH v| CM.
Xét đường tròn (I) có
ASN MNA
v| trong đường tròn (O) có
MCN MNA
Do đó ta được
ASN MCN
, suy ra tứ gi{c K’SNC nội tiếp. Nên ta có
0
K' SC K' NC 180
Ta cũng chứng minh được
HB.HD HM.HN HA.HS
, nên tứ gi{c ABSD nội tiếp đường
tròn.
Suy ra ta
ABD ASD
. Trong đường tròn (O) ta
ABD HCD
nên ta được
ASD HCD
, do đó tứ gi{c SHDC nội tiếp. Suy ra
0
HSC HDC 180
. M| ta lại
, do đó
K' NC BNC
Suy ra hai tia NK’ v| NB trùng nhau, do đó ba điểm B, K, N thẳng h|ng. Ta có hai điểm K
v| K’ trùng nhau nên A, K, S thẳng h|ng. Vậy ba điểm A, H, S thẳng h|ng.
H'
H
O
F
E'
S
K
N
M
B
E
D
C
A
Nguyễn Công Lợi
TÀI LIU TOÁN HC
26
Ví dụ 23. Cho đường tròn (I) nội tiếp tam gi{c ABC. Đường tròn (I) tiếp xúc với BC, AB,
CA lần lượt tại D, E, F. Gọi AM l| đường trung tuyến của tam gi{c ABC. Gọi giao điểm
của EF v| DI l| K. Chứng minh rằng ba điểm A, K, M thẳng h|ng.
Phân tích tìm lời giải
Trên cơ sở hình vẽ v| giả thiết của b|i to{n ta có hai nghĩa đến hai hướng chứng
minh ba điểm A, K, M thẳng h|ng là.
+ Qua K vẽ đường thẳng song song với BC cắt AB, AC lần lượt tại X v| Y. Gọi N l|
giao điểm của AK v| BC. Sử dụng tính chất hai tiếp tuyến cắt nhau, định Talets v| tính
chất đường ph}m gi{c ta đi chứng minh
NB NC
. Điều n|y sẽ dẫn đến M v| N trùng
nhau.
+ Qua K vẽ đường thẳng song song với BC cắt AB, AC lần lượt tại X v| Y. Sử dụng
tính chất hai tiếp tuyến cắt nhau, định Talets v| tính chất đường ph}n gi{c ta đi chứng
minh hai tia AK và AM trùng nhau.
Lời giải
Cách 1: Gọi N l| giao điểm của AK
và BC.
Ta có AE v| AF l| hai tiếp tuyến
của đường tròn (I) nên ta được
EAI FAI
. Qua A vẽ đường thẳng
song song với BC cắt DI tại L v| cắt
EF tại S. Qua K vẽ đường thẳng
song song với BC cắt AB, AC lần
lượt tại X v| Y.
Ta có
DI BC
và AS//BC nên
AL LI
. Do đó ta được
0
ALI AEI AFI 90
Từ đó suy ra c{c điểm A, E, I, F, L
cùng nằm trên một đường tròn.
Do đó ta được
ELI EAI IAF ILF
, suy ra LK là ph}n gi{c của tam gi{c LEF.
Do đó LK v| LS l| đường ph}n gi{c trong v| ph}n gi{c ngo|i của tam gi{c LEF.
Nên ta được
KE SE KE KF
KF SF SE SF
Ta có XK//AS và KY//AS nên ta có
XK KE
AS SE
KY KF
AS SF
. Từ đó suy ra
KX KY
S
L
K
N
M
I
Y
X
F
E
D
C
B
A
Nguyễn Công Lợi
TÀI LIU TOÁN HC
27
Trong tam gi{c ABN có XK//BN nên theo định lí Talets ta có
XK AK
BN AN
Trong tam gi{c ACN có KY//CN nên theo định lí Talets ta có
KY AK
CN AN
Kết hợp với
KX KY
ta suy ra được
BN CN
hay N l| trung điểm của BC.
Từ đó suy ra M v| N trùng nhau. Vậy ba điểm A, K, M thẳng h|ng.
Cách 2: Qua K vẽ đường thẳng song song với BC cắt AB, AC lần lượt tại X v| Y.
Ta có
DI BC
v| XY//BC nên ta được
DI XY
Ta có
0
XEI XKI 90
nên tứ gi{c EXKI nội tiếp đường tròn, suy ra
IEK IXK
Ho|n to|n tương tự ta được tứ gi{c IKFY nội tiếp nên
IFK IYK
M| tam gi{c IEF c}n tại I nên
IEK IFK
, do đó ta được
IXK IYK
Tam gi{c IXY c}n tại Y có IK l| đường cao nên IK cũng l| đường trug tuyến.
Do đó ta được
XY
XK YK
2

. Suy ra
XK 2XK XY
BM 2BM BC

Trong tam gi{c ABC có XY//BC nên theo định lí Talets ta có
AX XY
AB BC
Hai tam giác AXK và ABM có
AX XK
AB BM
AXK ABM
nên
AXK ABM
Điều n|y dẫn đến
XAK BAM
, suy ra hai tia AK v| AM trùng nhau. Vậy ba điểm A, K, M
thẳng h|ng
Ví dụ 24. Cho tam gi{c ABC nội tiếp đường tròn (O). Lấy điểm D trên cạnh BC. Đường
tròn t}m P tiếp xúc với AD, DC v| tiếp xúc trong với đường tròn (O). Đường tròn t}m Q
tiếp xúc với AD, BD v| tiếp xúc trong với đường tròn (O). Gọi I l| t}m đường tròn nội tiếp
tam gi{c ABC. Chứng minh rằng ba điểm P, I, Q thẳng h|ng.
Lời giải
Trước hết ta ph{t biểu v| chứng minh hai bổ để sau:
Bổ đề 1: Cho đường tròn (I) tiếp xúc trong với đường tròn
(O) tại C. AB là dây bất kì của đường tròn (O) và tiếp xúc
với đường tròn (I) tại T. Khi đó CT đi qua điểm chính giữa
của cung AB không chứa C.
Chứng minh
Gọi M l| giao điểm của CT với đường tròn (O)(M kh{c
C).
Do hai đường tròn (I) v| (O) tiếp xúc nhau nên ba
điểm C, O, I thẳng h|ng. Tam gi{c CIT có
ICT ITC
M
I
T
O
C
B
A
Nguyễn Công Lợi
TÀI LIU TOÁN HC
28
và tam giác OCM có
OCM OMC
Do đó ta được
ITC OMC
nên suy ra IT//OM
Mà ta có
IT AB
nên suy ra
OM AB
. Do đó M l| điểm chính giữa cung AB không chứa
C.
Bổ đề 2: Cho tam giác ABC nội tiếp đường tròn (O). Lấy điểm M thuộc BC. Đường tròn (O’) tiếp
xúc với hai đoạn thẳng MA, MC lần lượt tại E, F đồng thời tiếp xúc với đường tròn (O) tại K. Khi
đó tâm I của đường tròn nội tiếp tam giác ABC nằm trên đoạn thẳng EF.
Chứng minh
Vẽ tiếp tuyến Kx của đường tròn (O). Gọi N l| giao
điểm của KF với đường tròn (O). Khi đó theo bổ đề
1 thì điểm N nằm chính giữa cung BC của đường
tròn (O). Gọi I’ l| gao điểm của AN v| EF, khi đó ta
BN CN
nên ta được
NB NC
.
Do đó AN l| ph}n gi{c của góc
BAC
Ta có
I ' EK FKx
I ' AK NHx
suy ra
I ' EK I ' AK
nên tứ gi{c AKI’e nội tiếp đường
tròn. Do đó suy ra ta được
AI ' K AEK
, mà ta có
AEK EFK
nên
AI ' K EFK
Ta có
NI ' K NFI '
.
Xét hai tam gi{c NI’K v| NFI’ có
NI ' K NFI '
I ' NK
nên ta được
NI ' K NFI'
Do đó ta được
'2
NI ' NK
NI NK.NF
NF NI '
Xét hai tam giác NCF và NKC có
CNF
chung nên ta được
NCF NKC
Do đó ta được
2
NC NF
NC NF.NK
NK NC
Từ đó ta được
NC NI '
hay tam gi{c NCI’c}n tại N, do đó ta được
NI 'C NCI '
Từ đó suy ra
I ' CA I ' AC BCN BCI' I ' CA BCI '
hay CI’ l| ph}n gi{c của góc
ACB
Do đó I’ l| t}m đường tròn nội tiếp tam gi{c ABC, suy ra hai điểm I v| I’ trùng nhau.
Vậy I nằm trên đoạn thẳng EF.
x
K
M
N
F
E
I'
O
I
C
B
A
Nguyễn Công Lợi
TÀI LIU TOÁN HC
29
Trở lại b|i to{n: Gọi E, F lần lượt l| tiếp điểm
của đường tròn (P) với BC, AD. Gọi G, H lần
lượt l| tiếp điểm của đường tròn (Q) với BC,
AD. Gọi I l| giao điểm của EF với GH. Từ bổ
đề 2 suy ra I l| t}m đường tròn nội tiếp tam
gi{c ABC. Gọi M l| giao điểm của GH v| QD,
N l| giao điểm của EF v| PD. Dễ thấy MDNI
l| hình chữ nhật. Hai tam gi{c QDG v| DPE
QGD PEF
. Nên
QD QG
QDG DPE
PD DE
,
Xét hai tam giác QMG và DNE có
QMG DNE
MQG NDE
. Nên ta được
QMG DE
,
Suy ra
QG QM
DE DN
. Mà ta có
DN MI
nên
QM QG
MI DE
Từ đó
QM QD QG
MI DP DE

, suy ra
QM MI
QD DP
Xét hai tam giác QMI và QDP có
QM MI
QD DP
QMI QDP
. Nên ta được
QMI QDP
,
do đó ta được
IQM PQD
. Từ đó suy ra hai tia QI v| QP trùng nhau hay ba điểm Q, I, P
thẳng h|ng.
Nhận xét: Bổ đề 2 trong ví dụ 23 chính là nội dung định lí Lyness. Ba điểm P, I, Q thẳng hàng
trong ví dụ 23 nằm trên một đường thẳng có tên là đường thẳng The’bault.
Ví dụ 25. Cho tam gi{c nhọn ABC nội tiếp đường tròn (O; R). C{c tia AO, BO, CO cắt cạnh
BC, CA, AB lần lượt tại D, E, F. Gọi M l| giao điểm của hai tiếp tuyến tại A, B của đường
tròn (O). Chứng minh rằng nếu có
9
DE BE CF R
2
thì ba điểm C, F, M thẳng h|ng.
Lời giải
Ta có
ABD
AOB
S
AD
S AO
ACD
AOC
S
AD
S AO
. Do đó ta
được
ABD ACD ABD ACD
AOB AOC AOB AOC
AOB AOC BOC BOC
AOB AOC AOB AOC
S S S S
AD
AO S S S S
S S S S
1
S S S S


G
H
F
E
I
Q
P
N
M
D
C
B
A
O
M
F
D
E
C
B
A
Nguyễn Công Lợi
TÀI LIU TOÁN HC
30
Từ đó ta được
BOC
AOB AOC
S
AD
1
R S S

.
Ho|n to|n tương tự ta được
AOC AOB
BOC AOB AOC BOC
SS
BE CF
1 ; 1
R S S R S S

Từ đó ta được
BOC AOC AOB
AOB AOC BOC AOB AOC BOC
S S S
AD BE CF
3
R S S S S S S

Theo bất đẳng thức Neibitz ta có
BOC AOC AOB
AOB AOC BOC AOB AOC BOC
S S S
3
S S S S S S 2
Từ đó suy ra
AD BE CF 9
R2

hay
9
AD BE CF R
2
M| ta lại có
9
DE BE CF R
2
, tức l| dấu bằng của bất đẳng thức trên xẩy ra,
Do đó ta được
AOB AOC BOC
S S S
, điều n|y xẩy ra khi v| chỉ khi
0 0 0
1 1 1
OA.OB. sin 180 AOB OB.OC.sin 180 BOC OC.OA.sin 180 COA
2 2 2
Hay
AOB BOC COA
. Do đó ba tam gi{c AOB, BOC, COA bằng nhau nên
AB BC AC
Vậy tam gi{c ABC đều. M| ta lại có
OA OB
MA MB
nên C, O, M thuộc đường
trung tực của đoạn thẳng AB. Do đó ba điểm C, O, M thẳng h|ng. M| ba điểm C, O, F
thẳng h|ng. Từ đó ba điểm C, F, M thẳng h|ng.
dụ 26. Cho tam giác ABC
AB AC
nội tiếp đường tròn t}m O. Trên c{c tia đối của
tia BA, CA lấy c{c điểm M, N sao cho
BM AC;CN AB
. Gọi I l| điểm chính giữa cung
lớn BC. Gọi K l| điểm đối xứng với I qua BC. Chứng minh ba điểm M, N, K thẳng h|ng.
Phân tích tìm lời giải
Đầu tiên ta nhận thấy tam gi{c AMN c}n tại A, tam gi{c BIC c}n tại I v| tam gi{c
BKC c}n tại K. Lấy điểm H sao cho tứ gi{c ABHC l| hình bình h|nh thì ta được
BHC BAC
nên
, điều n|y dẫn đến BCHK nội tiếp đường tròn. Từ đó để
chứng minh ba điểm M, N, K thẳng h|ng ta đi chứng minh ba điểm H, N, H thẳng h|ng
v| ba điểm M, N, H thẳng h|ng. Tất nhiên ta cũng thể lấy điểm H trên MN thỏa mãn
CH//AB rồi chứng minh tứ gi{c ABHC l| hình bình h|nh cũng được.
Lời giải
Nguyễn Công Lợi
TÀI LIU TOÁN HC
31
Ta có
BM AC;CN AB
nên ta suy ra
được
AM AN
, do đó tam gi{c AMN c}n
tại A. Do I v| K đối xứng với nhau qua
BC nên
BIC BKC
.
Mà ta có
BIC BAC
nên ta được
BKC BAC
Tam gi{c BIC c}n tại I.
M| ta lại có
BKC BIC
nên tam giác
BKC c}n tại K.
Trên đoạn MN lấy điểm H sao cho
CH//AB. Ta có
CHN AMN
. M| ta lại có
AMN CNH
nên ta được
Do đó tam gi{c CHN c}n tại C, suy ra
CH CN
. Lại có
CN AB
nên ta được
CH AB
Tứ gi{c ABHC có CH//AB v|
CH AB
nên là hình bình hành.
Do đó ta được
BHC BAC
nên suy ra
BHC BKC
. Từ đó suy ra tứ gi{c BCHK nội tiếp
đường tròn.
Ta có
HCN BAC
nên ta được
Do hai tam gi{c HCN v| BKC c}n nên ta được
HCN BKC
Do đó ta được
CHN KBC
nên suy ra
CHN CHK KBC CHK
M| ta lại có
0
CHN CHK NHK; KBC CHK 180
nên ta được
0
NHK 180
.
Do đó ba điểm H, N, H thẳng h|ng. M| ta lại có M, N, H thẳng h|ng.
Vậy ba điểm M, N, K thẳng h|ng.
Ví dụ 27. Cho tam gi{c ABC vuông tại A
AB AC
, AH l| đường cao. Lấy điểm M nằm
giữa A v| H. Trên c{c đoạn MC, MB lấy c{c điểm E v| F sao cho
BE BA
CF CA
. Gọi
I l| giao điểm của BE v| CF. Gọi K l| giao điểm thứ hai của hai đường tròn ngoại tiếp tam
gi{c BHF v| CHE. Chứng minh rằng ba điểm I, H, K thẳng h|ng.
Phân tích tìm lời giải
Để chứng minh ba điểm H, I, K thẳng h|ng ta gọi G l| giao điểm thứ hai của IH với
đường tròn ngoại tiếp tam gi{c BFH v| đi chứng minh hai điểm G v| K trùng nhau. Muốn
vậy ta đi chứng tgi{c GHEC nội tiếp đường tròn. Vẽ BP vuông góc với CM tại P, BP cắt
AH tại S, BM cắt SC tại Q. Khi đó tam gi{c nhọn SBC A l| trực t}m. Từ đó ta chứng
minh được
SE SF
nên dẫn đến
IE IF
. Ta lại nhận thấy
BEH BCE
nên suy ra
O
I
H
K
N
M
C
B
A
Nguyễn Công Lợi
TÀI LIU TOÁN HC
32
BEH BCE
tương tự ta được
CFH CBF
nên từ đó ta suy ra được
CFH IGF
. Điều
n|y dẫn đến
IFH IGF
nên
22
IE IH
IF IH.IG IE IH.IG
IG IE
.
Từ đó suy ra hai tam gi{c IEG v| EIH đồng dạng với nhau, nên ta được
IEH IGE
nên
HGE HCE
, tức l| tứ gi{c GHEC nội tiếp. Vậy b|i to{n được chứng minh.
Lời giải
Vẽ BP vuông góc với CM tại P, BP cắt AH tại S,
BM cắt SC tại Q. Gọi O l| t}m của đường tròn
ngoại tiếp tam gi{c BHF. HI cắt đường tròn (O)
tại điểm thứ hai l| G. Trong tam gi{c SBC có
SH, CP l| c{c đường cao cắt nhau tại M nên M
l| trực t}m của tam gi{c SBC, do đó
BM SC
.
Hai tam giác BHS và BPC có
HBS
chung và
BHS BPC
nên
BHS BPC
. Suy ra ta được
BH BS
BP BC
nên suy ra
BH.BC BP.BS
. Tam giác
ABC vuông tại A có đường cao AH nên ta được
2
AB BH.BC
. Từ đó suy ra
2
BE BH.BC
, mà ta
BH.BC BP.BS
nên
2
BE BS
BE BP.BS
BP BE
. Hai tam giác BES và BPE có
EBS
chung và
BE BS
BP BE
nên
BES BPE
Do đó ta được
BES BPE
, mà ta có
0
BPE 90
nên
0
BES 90
hay tam gi{c BES vuông tại
E.
Trong tam gi{c BES vuông tại E có EP l| đường cao nên
2
SE SP.SB
Ho|n to|n tương tự ta được
.
Hai tam giác SPC và SQ có
PSC
chung và
SPC SQB
nên
SPC SQB
Do đó ta được
SP SC
SQ.SC SP.SB
SQ SB
. Từ đó ta được
SE SF
. Từ đó suy ra hai tam
gi{c IES v| IFS bằng nhau nên
IE IF
.
Hai tam giác BEH và BCE có
EBH
chung và
BE BH
BC BE
nên ta được
BEH BCE
Do đó ta được
BEH BCE
. Ho|n to|n tương tự ta được
CFH CBF
Mà ta có
CBF IGF
nên suy ra
CFH IGF
S
M
G
K
O
H
I
F
E
Q
P
C
B
A
Nguyễn Công Lợi
TÀI LIU TOÁN HC
33
Hai tam giác IFH và IGF có
FIH
chung và
IFH IGF
nên
IFH IGF
Do đó ta được
22
IF IH IE IH
IF IH.IG IE IH.IG
IG IF IG IE
.
Từ đó suy ra hai tam gi{c IEG v| EIH đồng dạng với nhau, nên ta được
IEH IGE
Do đó
IGE HCE
hay
, suy ra tứ gi{c ECGH nội tiếp đường tròn hay đường
tròn ngoại tiếp tam gi{c CHE đi qua điểm G. Do đó G l| giao điểm thứ hai của hai đường
tròn ngoại tiếp c{c tam gi{c BHF v| CHE. Suy ra hai điểm G v| K trùng nhau. Vậy ba
điểm I, H, K thẳng h|ng.
Ví dụ 28. Cho tam gi{c nhọn ABC
AB AC
, đường cao AH. Đường tròn (O; R) nội tiếp
tam gi{c ABC tiếp xúc với BC tại D. Gọi E l| trung điểm của AH, tia DE cắt đường trog (O)
tại F. Gọi L l| t}m đường tròn b|ng tiếp góc F của tam gi{c FBC. Chứng minh rằng ba
điểm F, D, L thẳng h|ng.
Phân tích tìm lời giải
Gọi M, N lần lượt l| tiếp điểm của đường tròn (O) trên c{c cạnh AB, AC. Đường
thẳng AD cắt đường tròn (O) tại T kh{c D. DO cắt đường tròn (O) tại điểm thứ hai l| G. Vẽ
OK vuông góc với DT tại K, OK cắt BC tại S. Qua A kẻ đường thẳng song song với BC cắt
MN tại J, BP vuông góc với GF tại P, CQ vuông góc với GF tại Q v| I l| giao điểm của OA
với MN. Ta FL l| ph}n gi{c của góc
BFC
. Nếu ta chứng minh được FD cũng l| ph}n
gi{c của góc
BFC
thì ba điểm F, D, L thẳng h|ng. Như vậy ta đi chứng minh FD l| ph}n
gi{c của góc
BFC
. Muốn vậy ta cần chứng minh được
BFD CFD
. Để ý l|
0
BFP BFD CFQ CFD 90
nên ta cần chứng minh được
BFP CFQ
. Như vậy ta cần
chứng minh được
BPF CQF
. Lại chu ý l|
BPF CQF
nên ta sẽ đi chứng minh
BP PF
CQ QF
.
Lời giải
Nguyễn Công Lợi
TÀI LIU TOÁN HC
34
Gọi M, N lần lượt l| tiếp điểm
của đường tròn (O) trên c{c cạnh
AB, AC. Đường thẳng AD cắt
đường tròn (O) tại T kh{c D. DO
cắt đường tròn (O) tại điểm thứ
hai l| G. Vẽ OK vuông c với
DT tại K, OK cắt BC tại S. Qua A
kẻ đường thẳng song song với
BC cắt MN tại J, BP vuông góc
với GF tại P, CQ vuông góc với
GF tại Q v| I l| giao điểm của
OA với MN.
Khi đó dễ d|ng chứng minh
được OA l| đường trung trực
của đoạn thẳng MN nên
MN OA
tại I. Ta OK vuông
góc với DT tại K nên K l| trung
điểm của DT.
Tam gi{c ODS vuông tại D có
DK l| đường cao nên
2
OD OK.OS
Tam gi{c OMA vuông tại M có MI l| đường cao nên
2
OM OI.OA
Mà ta có
OM OD
nên ta được
OK OA
OK.OS OI.OA
OI AS
Từ đó suy ra
OIS OKA
nên
0
OIS OKA 90
Do IS v| MN cùng vuông góc với OA tại I nên IS v| MN trùng nhau, suy ra ba điểm M, N,
S thẳng h|ng.
Hai tam giác DTG và AHD có
DTG AHD
GDT DAH
nên
DTG AHD
Từ đó ta được
TG TD 2TK TK
HD HA 2HE HE
, suy ra
GKT DEH
Do đó suy ra
GKT DEH
, mà ta có
0
GKT OKG 90
0
DEH EDH 90
nên
OKG EDH
Măt kh{c ta có
EDH OGF
nên ta được
OKG OGF
Ta có
2
OK.OS OD
OD OG
nên ta được
OKG OGS
O
T
L
F
E
K
I
S
Q
N
M
P
J
G
D
C
B
A
Nguyễn Công Lợi
TÀI LIU TOÁN HC
35
Từ đó suy ra
, do đó ta được
OGF OGS
nên
GF GS
Do đó ba điểm G, F, S thẳng h|ng nên BP//DF//CQ. [p dụng định lí Talets v| tính chất hai
tiếp tuyến cắt nhau ta được
BP BS BS AJ BM AN BM BD PF
..
CQ CS AJ CS AM CN CN CD QF
Kết hợp với
BPF CQF
nên
BPF CQF
, từ đó ta được
BFP CFQ
Mà ta có
0
BFP BFD CFQ CFD 90
nên ta được
BFD CFD
, suy ra FD là phân giác
của góc
BFC
. Mặt kh{c ta có FL cũng l| ph}n gi{c của góc
BFC
. Do đó FD v| FL trùng
nhau hay ba điểm F, D, L thẳng h|ng.
dụ 29. Cho tam gi{c đều ABC nội tiếp đường tròn (O). Lấy điểm M bất trên cung AB
koong chứa điểm C. Đường thẳng AM cắt tiếp tuyến tại B, tiếp tuyến tại C với đường tròn
(O) v| đường thẳng BC lần lượt tại E, F, H. Chứng minh rằng giao điểm của c{c đường
tròn ngoại tiếp c{c tam gi{c MAO, MBF, MCF v| O, H l| c{c điểm thẳng h|ng
Lời giải
Cách 1: Gọi N l| giao điểm
của BF v| CE. Kéo dại MN
cắt BC tại S. Khi đó ta có
ABE ACB
ACB BAC
nên ta suy ra được
ABE BAC
. Từ đó
BE//AC. Mặt kh{c ta có
0
EBC ABE ABC
BAC ABC 120

Ho|n to|n tương tự ta được CF//AB v|
0
BCF 120
Hai tam giác ABE và FCA có
BAE CFA
BEA CAF
nên
ABE FCA
Do đó ta được
2
AB BE BC BE
AB.AC BE.CF BC BE.CF
CF AC CF BC
Hai tam giác EBC và BCF có
EBC BCF
BC BE
CF BC
nên
EBC BCF
Do đó
CEB FBC
hay
NBC CEB
. Hai tam giác CBN và CEB có
NBC CEB
CBN
chung nên
CBN CEB
. Suy ra
0
BNC EBC BNC 120
nên
00
BNE 180 BNC 60
Mặt kh{c ta lại có
0
BME ACB 60
nên
BNE BME
A
B
C
O
M
N
H
E
F
S
Nguyễn Công Lợi
TÀI LIU TOÁN HC
36
Từ đó suy ra tứ gi{c BEMN nội tiếp đường tròn. Chứng minh tương tự ta được tứ gi{c
CFMN nội tiếp đường tròn. Từ đó ta có
NEB NMB
NBS NEB
nên
NBS NMB
hay
SBN SMB
Hai tam giác SBN và SMB có
BSN MSB
SBN SMB
nên
SBN SMB
Từ đó ta được
2
SB SN
SB SM.SN
SM SB
. Ho|n to|n tương tự ta cũng được
2
SC SM.SN
Từ đó ta được
SB SC
hay S l| trung điểm của BC.
Dễ d|ng chứng minh được
AB BC CA R 3
. Tam gi{c OBC c}n tại O có OS l| đường
OS cũng l| đường cao v| đường ph}n gi{c của tam gi{c BOC. Từ đó ta suy ra
R
SO
2
3R
SA
2
Từ đó ta được
2
SB SO.SA
, mà ta có
2
SB SM.SN
nên ta được
SO.SA SM.SN
hay
SN SO
SA SM
.
Xét hai tam giác SON và SMA có
SN SO
SA SM
MSA
chung nên
SON SMA
Do đó ta được
SON SMA
. Suy ra tứ gi{c AONM nội tiếp đường tròn. Mặt kh{c tứ gi{c
BNOC cũng nội tiếp đường tròn. Gọi
1
N
l| giao điểm của OH với đường tròn ngoại tiếp
tam gi{c OBC, gọi
2
N
l| giao điểm của DH với đường tròn ngoại tiếp tam gi{c OAM. Khi
đó
1
HN .HO BH.HC
2
HN .HO HM.HA
Mà ta có
HB.HC HM.HA
nên ta được
1 2 1 2 1 2
HN .HO HN .HO HN HN N N
Khi đó giao điểm của hai đường tròn ngoại tiếp hai tam gi{c OBC v| OAM l|
12
NN
M| N l| giao điểm của hai đường tròn ngoại tiếp hai tam gi{c OBC v| OAM nên
12
N N N
.
1
N
thuộc OH nên N cũng thuộc OH. Ta có N thuộc c{c đường tròn ngoại tiếp c{c tam
gi{c OBC, OAM, MCF nên N l| giao điểm của ba đường tròn đó. Từ đó ta được giao điểm
của c{c đường tròn ngoại tiếp c{c tam gi{c MAO, MBF, MCF v| O, H l| c{c điểm thẳng
hàng.
Nguyễn Công Lợi
TÀI LIU TOÁN HC
37
Cách 2: Gọi N l| giao điểm
của BF v| CE. Kéo d|i MN
cắt BC tại S. Hai tiếp tuyến
với đường tròn (O) tại B, C
cắt nhau ở P. Từ P kẻ đường
thẳng song song với BC cắt
FB tại Q. Chứng minh tương
tự c{ch 1 ta được
ABE FCA
EBC BCF
CBN CEB
SBN SMB




Từ đó ta được S l| trung
điểm của BC.
Ta có
BCP ABC
nên
CP//AB.
Tương tự ta cũng có AC//BP. Nên tứ gi{c ABPC l| hình bình h|nh. M| ta có
AB AC
nên
tứ gi{c ABPC l| hình thoi. Do đó S l| trung điểm của AP. Điều n|y dẫn đến c{c điểm A, O,
S, P thẳng h|ng.
Ta có
0
OBP OCP 90
nên c{c điểm B, C, O, P thuộc đường tròn đường kính OP.
Ta có
EPQ PBC
PBC ACB
nên
0
EPQ ACB 60
[p dụng định lí Talets cho c{c FPQ v| FPE ta được
BC FC AC FC
;
PQ FP FE FP

nên ta được
BC AC
PQ FE
Mà ta có
AC BC
nên
PQ PE
. Mặt kh{c ta có
0
EPQ 60
nên tam gi{c EPQ đều.
Ta có
NBC NQP
NBC NEP
nên
NQP NEP
, do đó tứ gi{c ENPQ nội tiếp đường
tròn.
Từ đó ta được
0
BNP BCP 60
, suy ra tứ gi{c BNCP nội tiếp đường tròn, nên N thuộc
đường tròn ngoại tiếp tam gi{c BCP. M| O cũng thuộc đường tròn ngoại tiếp tam gi{c
BCP nên năm điểm O, N, B, C, P cùng thuộc đường tròn đường kính OP. Do đó ta được
00
ONP 90 SNP SNO 90
.
Tam giác HAP có HS l| đường cao cũng l| đường trung tuyến nên c}n tại H.
Ta có
HPS HAS
HAS SNO
nên
HPS SNO
S
F
E
H
Q
P
N
M
O
C
B
A
Nguyễn Công Lợi
TÀI LIU TOÁN HC
38
Ta có
00
SHP HPS 90 ;SNP SNO 90 ;HPS SNO
nên ta được
SHP SNP
Từ đó suy ra tứ gi{c BNSP nội tiếp đường tròn. Suy ra
0
HNP 90
.
Ta có
0
OHN OHP HNP 180
nên ba điểm O, H, N thẳng h|ng.
Tứ gi{c BNOC nội tiếp đường tròn nên
0
BNC BOC 120
Mặt kh{c ta có
00
BNE 180 BNC 60
, m| ta lại có
0
BME ACB 60
Từ đó ta được
BNE BME
nên tứ gi{c BEMN nội tiếp đường tròn. Chứng minh tương tự
ta được tứ gi{c CFMN nội tiếp. Như vậy N thuộc c{c đường tròn ngoại tiếp c{c tam gi{c
MAO, MBE, MCF. Vậy giao điểm của c{c đường tròn ngoại tiếp c{c tam gi{c MAO, MBF,
MCF v| O, H l| c{c điểm thẳng h|ng.
dụ 30. Cho tam gi{c ABC v| đường tròn (I) nội tiếp tam gi{c tiếp xúc với c{c cạnh AB,
BC, CA lần lượt tại F, D, E. Gọi M l| giao điểm của BC với đường ph}n gi{c trong của góc
BIC
v| N l| giao điểm của EF với đường ph}n gi{c trong của góc
EDF
. Chứng minh rằng
ba điểm A, M, N thẳng h|ng.
Phân tích tìm lời giải
Nếu ABC c}n tại A thì hiển nhiên ba điểm
A, M, N thẳng h|ng. Như vậy ta cần chứng minh
trong trường hợp tam gi{c ABC không c}n tại A
thì ba điểm A, M, N thẳng h|ng. Chú ý đến t}m I
của đường tròn nội tiếp tam gi{c ABC ta có
00
ABC ACB ABC ACB
IMC 90 ; NDC 90
4 4 4 4
nên ta suy ra được IM//DN hay IM//PN. Như vậy
để chứng minh được ba điểm A, M, N thẳng
h|ng thì ta cần chứng minh được
IM AP AN
PN IA AM

.
Lời giải
+ Trường hợp tam gi{c ABC c}n tại A, khi đó hiển nhiên ba điểm A, M, N thẳng h|ng
+ Xét trường hợp tam gi{c ABC không c}n tại A, khi đó không mất tính tổng qu{t ta giả sử
AB AC
.
Gọi P, Q theo thứ tự l| giao điểm của AI với (I) v| EF. Dễ thấy ba điểm D, N, P thẳng h|ng
nên ta có
P
Q
N
M
D
I
F
E
C
B
A
Nguyễn Công Lợi
TÀI LIU TOÁN HC
39
0
00
0 0 0
0
00
11
IMC MIB MBI BIC MBI 180 IBC ICB MBI
22
1 ABC ACB ABC ABC ACB
180 90
2 2 2 2 4 4
11
NDC NDE EDC FDE EDC 180 90 IBD 90 ICD
22
1
180 FDB EDC EDC
2
IBD ICD ABC ACB
90 90
2 2 4 4







Do đó ta được
IMC NDC
do đó ta được IM//ND. Do đó ta suy ra IM//PN (*). Để ý l|
ID IP
nên ta được
MID IDP QPN
. Do BC tiếp xúc với (I) tại D v| P l| điểm chính
giữa cung
EF
nên ta được
1 1 1
IDM PDM IDP sdPED IDP sdPE sdED IDP
2 2 2
11
sdPF sdED IDP PNF QPN PQN
22
Do đó ta được
IDM PQN
, suy ra
IM DI
PN QP
m| ta lại có
ID IP
do đó
IM PI
PN QP
Mặt kh{c ta có
0
IAE 90
,
EQ IP
IE IP
nên ta được
22
IQ.IA IE IP
Do đó ta có
QP IQ IP PA IP IA
11
IP IP IA IA PQ AP
, nên ta được
IM AP
PN IA
(**)
Từ (*) v| (**) ta được A, M, N thẳng h|ng.
dụ 31. Cho tam gi{c nhọn ABC nội tiếp đường tròn (O). Gọi I l| điểm chính giữa cung
BC
không chứa A. Trên AC lấy K kh{c C sao cho
IK IC
. Đường thẳng BK cắt đường
tròn (O) tại D kh{c B. Trên DI lấy M sao cho CM song song với AD. Đường thẳng KM cắt
đường thẳng BC tại N. Đường tròn ngoại tiếp tam gi{c BKN cắt đường tròn (O) tại P kh{c
B. Chứng minh rằng PK đi qua trung điểm của đoạn thẳng AD.
Phân tích và lời giải
Do I l| điểm chính giữa cung
BC
v| theo giả
thiết ta được
IB IC IK
. Do đó ta được
00
IKD 180 IKB 180 IBK CID
Cũng do I l| điểm chính giữa cung
BC
nên
DI l| tia ph}n gi{c của góc
BDC
. Do đó ta
được
KID CID
Xét hai tam gi{c KID v| CID có ID l| cạnh
chung,
KID CID
IK IC
nên
KID CID
nên ta được
DK DC
. Từ đó
P
N
M
O
J
I
K
D
C
B
A
Nguyễn Công Lợi
TÀI LIU TOÁN HC
40
ta được AI l| đường trung trực của đoạn
thẳng BK. Gọi IJ l| đường kính của (O), ta có
AJ//DH v| DJ//AK. Nên tứ gi{c AJDK l|
hình bình h|nh. Do đó KJ đi qua trung điển
của AD.
Như vậy để chứng minh PK đi qua trung điểm của AD ta chỉ cần chứng minh ba điểm J,
K, P thẳng h|ng l| được. Thật vậy, ta có
IPK IPB BPK
. Do
DBC DIC DIK
nên tứ
gi{c BKNP nội tiếp, suy ra
. Lại có
BPI BAI
. Do đó ta được
IPK BAI BNK BAI NKC NCK
Do ID l| đường trung trực của CK nên ta được
IPK BAI NKC NCK BAI MCK BCK
Lại có CM//AD nên ta được
IPK BAI MCK BCK BAI CAD BCK
Do tứ gi{c ABCD nội tiếp nên
IPK BAI CAD BCK BAI CBD BCK IAK AKB
Do
BK AI
nên ta được
0
IPK 90 IPJ
. Từ đó ta được ba điểm P, K, J thẳng h|ng. Vậy
PK đi qua trung điểm của đoạn thẳng AD.
dụ 32. Cho tứ gi{c ABCD c{c cặp cạnh đối diện không song song với nhau, ngoại
tiếp đường tròn t}m O. Gọi M, N lần lượt l| trung điểm của AB, CD. Chứng minh rằng ba
điểm M, O, N thẳng h|ng khi v| chỉ khi
OA.OC OB.OD
.
Phân tích tìm lời giải
Giả sử ba điểm M, O, N thẳng h|ng. Khi đó gọi R v| S lần lượt l| trung điểm của
AD v| BC thì ta được tứ gi{c MRNS l| hình bình h|nh nên MN v| SR cắt điểm O. Gọi giao
điểm của AB v| CD l| P khi đó tam gi{c MPN c}n tại P. Để chứng minh được
OA.OC OB.OD
ta cần chứng minh
OA OB
OD OC
. Chú ý
NC ND
ta cần chứng minh được
BO BM MO
OC ON NC

OA MA OM
OD NO DN

hay ta cần chứng minh được hai tam gi{c BMO v|
ONC đồng dạng, hai tam gi{c OMA v| DNO đồng dạng.
Lời giải
Nguyễn Công Lợi
TÀI LIU TOÁN HC
41
+ Điều kiện cần: Giả sử ba điểm M, O, N
thẳng h|ng. Khi đó gọi R v| S lần lượt l|
trung điểm của AD v| BC. Dễ d|ng
chứng minh được tứ gi{c MRNS l| nh
bình h|nh nên MN v| SR cắt nhau tại
trung điểm của mỗi đường, điểm đó
chính l| điểm O. Gọi giao điểm của AB
v| CD l| P. Do đường tròn (O) nội tiếp
tứ gi{c ABCD nên đường tròn (O) nội
tiếp ta gi{c APD. Từ đó suy ra PO l|
ph}n gi{c của góc
MPN
, đồng thời PO l|
đường trung tuyến của tam gi{c MPN.
Do đó tam gi{c MPN c}n tại P.
Mặt kh{c ta có
0
MBC BPC BCP BPC 180 BCN
hay ta được
0
MBC BCN BPC 180
Suy ra
00
11
MBC BCN MPC 180 MBO NCO BPO 90
22
Ta lại có
0
0
MBO BPO BOP
MBO MOB BPO 90
MOB 90 BOP


Kết hợp c{c kết quả trên ta được
MBO NCO MBO MOB NCO MOB
Điều n|y dẫn đến hai tam gi{c BMO v| ONC đồng dạng với nhau nên ta được
BO BM MO
OC ON NC

Chứng minh tương tự ta cũng có hai tam gi{c OMA v| DNO đồng dạng với nhau nên
OA MA OM
OD NO DN

Để ý l|
NC ND
nên ta được
OA OB
OD OC
hay ta được
OA.OC OB.OD
.
+ Điều kiện đủ: Giả sử ta có
OA.OC OB.OD
. Khi đó để ý l|
00
00
AOB COD 180 OAB OBA 180 OCD ODC
1
360 DAB ABC BCD CDA 180
2
Vẽ c{c hình bình h|nh OAKB v| OCHD. Khi đó ta có
OA OA OD
AK OB OD

H
K
P
O
M
N
D
C
B
A
Nguyễn Công Lợi
TÀI LIU TOÁN HC
42
Từ đó hai tam gi{c OMB v| CNO đồng dạng nên
MOB OCN OCB
CON MBO OBC
Suy ra ta được
0
MOB BOC CON OCB BOC OBC 180
.
Điều n|y có nghĩa l| điểm O nằm trên đường thẳng MN. Hay ta được ba điểm M, O, N
thẳng h|ng.
dụ 33. Cho tứ gi{c ABCD nội tiếp đường tròn (O). Lấy điểm P nằm trên đường thẳng
AC sao cho PB v| PD l| hai tiếp tuyến của đường tròn. Tiếp tuyến tại C cắt PD v| AD lần
lượt tại Q v| R. Gọi E l| giao điểm thứ hai cả AQ với đường tròn (O). Chứng minh rẳng ba
điểm B, E, R thẳng h|ng.
Phân tích và lời giải
Gọi giao điểm của BE với AD l| R’, ta cần
chứng minh R v| R’ trùng nhau.
Thật vậy, ta có
APD PAC
DPC
chung nên
PAD PDC
. Lại có
PAB PBC
BPC
chung nên ta được
PAB PBC
Từ đó ta được
AD PA PA AB
CD PD PB BC

, do
đó suy ra
AD.BC AB.CD
.
Tương tự ta được
AC.DE AD.CE
[p dụng định lí Ptoleme cho tứ gi{c
ABCD nội tiếp ta được
AB.CD AD.BC AC.BD
Từ đó suy ra
1
AD.BC AB.CD AC.BD
2

[p dụng định lí Ptoleme cho tứ gi{c ACED nội tiếp ta được
AC.DE AD.CE AE.CD
Suy ra ta được
1
AC.DE AD.CE AE.CD
2

. Từ đó ta được
BD 2CD
AB CA
CD 2ED
CA EA
Ho|n to|n tương tự như trên ta chứng minh được
RDC RCA
nên suy ra
RD CD RC
RC CA RA

.
Từ đó ta được
2 2 2
2
RD RD.RA RC CD 2ED
RA RA CA AE
RA
Ta cũng có
ABR ' DR '
DBR ' EAR '
nên ta được
R ' D ED
R ' B AB
R ' A EA
R ' B BD
P
E
Q
O
R'
R
D
C
B
A
Nguyễn Công Lợi
TÀI LIU TOÁN HC
43
Suy ra
2
R ' D ED.BD 2ED
R ' A EA.AB AE




. Từ đó ta được
RD R ' D
RA R ' A
nên hai điểm R v| R’ trùng
nhau.
Vậy ba điểm B, E, R thẳng h|ng
dụ 34. Cho hai đường tròn
1
O
2
O
cắt nhau tại A v| B. C{c tiếp tuyến tại A v| B
của đường tròn
1
O
cắt nhau tại K. Lấy điểm M trên đường tròn
1
O
không trùng với A
v| B. Gọi P l| giao điểm thứ hai của MA với đường tròn
2
O
. Gọi C l| giao điểm thứ hai
của MK với đường tròn
1
O
. Gọi Q l| giao điểm thứ hai của đường thẳng CA với đườn
tròn
2
O
. Chứng minh rằng trung điểm của đoạn thẳng PQ nằm trên đường thẳng CM.
Lời giải
Không mất tính tổng qu{t, giả sử M thuộc
cung lớn
AB
, trường hợp M thuộc cung
nhỏ
AB
được chứng minh tương tự. Gọi
H l| giao điểm của PQ v| BC. Ta cần
chứng minh được
PH QH
.
Do BK l| tiếp tuyến của đường tròn
1
O
v| KCM l| c{t tuyến của tương ứng nên
ta dễ d|ng chứng minh được
BCK MBK
suy ra
BC CK
BM BK
. Hoàn
to|n tương tự ta được
AC CK
AM AK
M| ta lại có
KA KB
, nên từ c{c tỉ số trên ta được
AC BC
AM BM
. Ta lại có tứ gi{c AMBC nội
tiếp đường tròn
1
O
nên ta được
AMB BCQ PMB BCQ
Mặt kh{c tứ gi{c AQB nội tiếp đường tròn
2
O
nên ta được
BPM AQB BPM CQB
Do đó ta được
MP BM
BMP BCQ
CQ BC
[p dụng định lí Menelaus cho tam gi{c APQ với ba điểm C, M, H thẳng h|ng ta được
CA HQ MP HP AC MP BC BM
. . 1 . . 1 HP HQ
CQ HP MA HQ AM CQ BM BC
Hay H l| trung điểm của PQ. Ta có điều phải chứng minh.
Ví dụ 35. Cho tam gi{c ABC trực t}m H. Đường ph}n gi{c ngo|i của góc
BHC
cắt cạnh
AB, AC lần lượt tại D v| E. Đường ph}n gi{c trong của góc
BAC
cắt đường tròn ngoại tiếp
tam gi{c ADE tại K. Chứng minh rằng K, H v| trung điểm của BC thẳng h|ng.
Phân tích tìm lời giải
H
O
2
O
1
Q
P
K
M
C
B
A
Nguyễn Công Lợi
TÀI LIU TOÁN HC
44
Nhận thấy tam gi{c ADE c}n tại A. Gọi P l| giao điểm của KD v| HB, Q l| giao
điểm của KE v| HC. Khi đó tứ gi{c HPKQ l| hình bình h|nh. Để chứng minh H, K v|
trung điểm của BC thẳng h|ng ta cần chỉ ra được PQ song song với BC. Muốn vậy ta đi
chứng minh
PB QC
PH QH
Lời giải
Trước hết ta chứng minh tam gi{c ADE c}n tại A.
Thật vậy, vì HD l| ph}n gi{c ngo|i của góc
BHC
nên
ta có
00
11
DHB HBC HCB 90 ABC 90 ACB
22
1
BAC
2



Do đó ta được
00
11
ADE DBH DHB 90 BAC BAC 90 BAC
22
Tương tự ta cũng có
0
1
AED 90 BAC
2

, từ đó suy ra
hay tam giác ADE cân
tại E.
Mặt kh{c ta có AK l| ph}n gi{c của góc
DAE
nên cũng l| đường trung trực của đoạn DE,
từ đó suy ra AK l| đường kính của đường tròn ngoại tiếp tam gi{c ADE. Từ đó
KD AB
,
tương tự ta cũng có
KE AC
.
Gọi P l| giao điểm của KD v| HB, Q l| giao điểm của KE v| HC. Ta có
KP AB
HQ AB
, từ đó suy ra KP//HQ. Tương tự ta cũng được KQ//HP, do đo tứ gi{c KPKQ l|
hình bình hành.
Gọi BB’ v| CC’ l| c{c đường cao của tam gi{c ABC, ta có DP//HC’ v| QE//HB’
Nên theo định lí Talet ta có
PB DB QC EC
;
PH DC' QH EB '

Theo tính chất đường ph}n gi{c của tam gi{c ta có
DB HB EC DB
;
DC' HC ' EB ' HB'

Vì B, C, B’, C’ cùng thuộc đường tròn đường kính BC nên ta được
BHC' CHB'
Từ đó ta được
HB HC
HC ' HB '
. Kết hợp với c{c kết quả trên ta được
PB QC
PH QH
, nên theo định
Talet ta được PQ song song với BC. Do HK đi qua trung điểm của PQ nên HK cũng đi qua
trung điểm của BC.
Ví dụ 36. Cho tam gi{c ABC nhọn nội tiếp đường tròn (O) có trực t}m H, đường cao AF v|
M l| trung điểm của BC. Đường tròn đường kính AH cắt đường tròn O) tại Q kh{c A.
K
H
Q
P
C'
B'
D
E
C
B
A
Nguyễn Công Lợi
TÀI LIU TOÁN HC
45
Đường tròn đường kính HQ cắt đường tròn (O) tại K kh{c Q. Đường thẳng KQ cắt dường
tròn ngoại tiếp tam gi{c KFM tại N kh{c K. Chứng minh rằng MN đi qua trung điểm của
AQ.
Phân tích và lời giải
Gọi L v| R lần lượt l| t}m đường tròn
ngoại tiếp c{c tam gi{c KHQ v| KMF. Ta
cần chứng minh ba điểm K, L, R thẳng
hàng.
Thật vậy, gọi AE l| đường kính của đường
tròn (O) v| D đối xứng với H qua BC. Khi
đó dễ thấy tứ gi{c HBEC l| hình hình
h|nh nên ba điểm H, M, E thẳng h|ng. Lại
có Q thuộc đường tròn đường kính AE
nên ta được
0
AQF 90
. M| Q lại thuộc
đường tròn đường kính AH nên ta được
0
QAH 90
Từ đó
0
AQH AQF 90
nên ba điểm Q, H,
E thẳng h|ng. Do vậy bốn điểm Q, H, M. E
thẳng h|ng.
Gọi tiếp tuyến tại K v| H của đường tròn
ngoại tiếp tam gi{c KHQ cắt nhau tại X.
Khi đó ta có
XKH XHK KQH
(góc nội tiếp v| góc tạo bởi tia tiếp tuyến với d}y cung
cùng chắn một cung). M| ta lại có
KQH KAE
( hai góc nội tiếp cùng chắn cung
KE
của
đường tròn (O)). Từ đó ta có
0 0 0 0
KXH 180 2KHX 180 2KQH 2 90 KQH 2 90 KAE 2AEK 2KDH
Lại có
XH XK
nên X l| t}m đường tròn ngoại tiếp tam gi{c KHD.
Do D v| H đối xứng nhau qua BC nên BC l| đường trung trực của DH, từ đó suy ra X
thuộc đường thẳng BC. Do XH l| tiếp tuyến của đường tròn đường kính QH nên ta có
XH QH
Xét tam gi{c XHM vuông tại H có đường cao HF nên ta có
22
XK XH XF.XM
Từ đó suy ra XK l| tiếp tuyến chung của hai đường tròn ngại tiếp tam gi{c KQH v| KFM
với tiếp điểm K l| giao điểm của hai đường tròn. Do đó ta suy ra hai đường tròn ngại tiếp
tam gi{c KQH v| KFM với tiếp xúc nhau K. Từ đó suy ra hai t}m đường tròn v| tiếp điểm
N
R
L
A
B
C
D
X
F
E
M
H
Q
K
O
Nguyễn Công Lợi
TÀI LIU TOÁN HC
46
cùng nằm trên một đường thẳng hay ba điểm L, K, R thẳng h|ng. Gọi P l| trung điểm của
AQ, ta cần chứng minh ba điểm M, N, P thẳng h|ng.
Thật vậy, ta có bốn điểm Q, H, M, E thẳng h|ng, từ đó suy ra
KQH KAE
nên hai tam
gi{c vuông KQH v| KAE đồng dạng. Từ đó suy ra hai tam gi{c KQA v| KHF đồng dạng
M| ta lại có KP v| KM lần lượt l| hai trung tuyến của hai tam gi{c KQA v| KHE nên ta
được hai tam gi{c KQP v| KHM đồng dạng, suy ra
QPK QMK
QKP HKM
.
Từ đó suy ra tứ gi{c QPMK nội tiếp đường tròn. Ta có biến đổi góc sau
0
0 0 0
CMN QKF QMK LKM MKF KPQ RMK 90 RMF
90 PMK RMK 90 RMF 180 BMP CMP
Do đó ba điểm M, P, N thẳng h|ng hay MN đi qua trung điểm của AQ. Vậy ta có điều
phải chứng minh.
Ví dụ 37. Cho hai đường tròn
1
O
2
O
cắt nhau tại hai điểm A v| B. Lấy điểm Q nằm
trên đường tròn
2
O
. C{c đường thẳng AQ, BQ cắt đường tròn
1
O
lần lượt tại C, D. C{c
tiếp tuyến tại A, B với đường tròn
2
O
cắt nhau tại P. Chứng minh rằng PQ đi qua trung
điểm của CD
Phân tích và lời giải
Gọi M l| giao điểm của AB v|
12
OO
, khi
đó M l| trung điểm của AB. Để chứng
minh PQ đi qua trung điểm của CD ta cần
chứng minh
MQB PQC
.
Thật vậy, giả sử ta có
MQB PQC
. Khi
đó ta gọi K l| giao điểm của PQ v| CD.
Tứ gi{c ABCD nội tiếp đường tròn nên ta
có
QBM KCQ
, kết hợp với
MQB PQC
ta suy ra được hai tam gi{c MQB v| KQC
đồng dạng.
Từ đó ta được
MB QB
KC QC
. Cũng do tứ gi{c ABCD nội tiếp đường tròn nên ta có
QCD QBA
do đó
AB QB
CD QC
. Từ đó ta được
MA AB
KC CD
, m| ta lại có
1
MA AB
2
nên ta
thu được
1
KC DC
2
hay K l| trung điểm của DC. Do đó PQ đi qua trung điểm của CD.
Để b|i to{n sẽ được chứng minh ho|n tất ta cần chứng minh được
MQB PQC
. Thật vậy,
gọi N l| giao điểm của PQ với đường tròn
2
O
.
N
O
2
O
1
D
Q
P
K
M
C
B
A
Nguyễn Công Lợi
TÀI LIU TOÁN HC
47
Do PA l| tiếp tuyến của đường tròn
2
O
nên ta suy ra được
PAN PQA
Từ đó suy ra
PA AN PN
PQ AQ PA

. Tương tự ta cũng có
PB BN PN
PQ QB PB

.
M| ta lại có
PA PB
nên từ c{c kết quả trên ta thu được
BN AN
AQ.BN BQ.AN
QB AQ
.
[p dụng định lí Ptoleme cho tứ ANBQ nội tiếp ta được
AN.BQ BN.AQ AB.NQ
Do đó ta được
1
NB.AQ AN.BQ AB.QB BM.QN
2
. Do đó
BQ QN
BM AN
, kết hợp với
QBM QNA
ta được
QBM QAN
. Từ đó suy ra
MQB PQC
. Vậy b|i to{n được
chứng minh.
B. CÁC BÀI TOÁN VỀ BA ĐƯỜNG ĐỒNG QUY
I. Một số phương pháp chứng minh ba đường đồng quy
Phương pháp 1: Chuyển bài toán chứng minh ba đường thẳng đồng quy về bài toán chứng
minh ba điểm thẳng hàng.
Phương pháp 2: Chứng minh ba đường thẳng là đường trung tuyến, ba đường phân giác, ba
đường cao, ba đường trung trực trong tam giác.
Phương pháp 3: Gọi giao điểm của hai đường thẳng M chứng minh đường thẳng còn
lại cũng đi qua điểm M.
Phương pháp 4: Sử dụng định lí Ceva.
Cho tam giác ABC. Các điểm A’, B’, C’ lần lượt thuộc các đường thẳng BC, CA, AB. Khi đó
ba đường thẳng AA’, BB’, CC’ đồng quy khi và chỉ khi
' ' '
. . 1
' ' '
A B B C C A
A C B A C B
.
Chứng minh
+ Điều kiện cần: Qua A kẻ đường thẳng song song với BC cắt đường thẳng BB’, CC’ tại M,
N.
Ta có
B ' C BC C'A AN A ' B AM
;;
B ' A AM C ' B BC A ' C AN
. Vậy ta có
A'B B ' C C ' A AM BC AN
. . . . 1
A ' C B ' A C ' B AN AM BC

+ Điều kiện đủ: Gọi I l| giao của BB’ v| CC’. Giải sử AI cắt BC tại A’’, suy ra A’’ cũng
thuộc BC.
Theo định lý Ceva (phần thuận) ta có
A''B B ' C C ' A
. . 1
A '' C B ' A C ' B
A'B B ' C C' A
. . 1
A ' C B ' A C' B
Nên
. Từ đó suy ra
A'' A'
. Do đó AA’, BB’, CC’ đồng quy
II. Một số ví dụ minh họa
Nguyễn Công Lợi
TÀI LIU TOÁN HC
48
Ví dụ 1. Cho tam gi{c ABC có trực t}m H. Đường tròn (O) đi qua hai điểm B, C cắt AB,
AC lần lượt tại D, E
D
B;E C
. Gọi K l| trực t}m tam gi{c ADE. Chứng minh rằng c{c
đường thẳng BE, CD, HK đồng quy.
Phân tích tìm lời giải
Để chứng minh BE, CD, HK đồng quy ta sẽ đi chứng minh ba điểm H, I, K thẳng
h|ng. Muốn vậy ta cần phải chứng minh được
0
BIK BIH 180
Lời giải
Vẽ c{c hình bình h|nh HBMC v| IHCN. Khi đó ta có
ADE ECB
ADE ABH ACH
. Từ đó ta được
KDE HCB
Ho|n to|n tương tự ta chứng minh được
KED HBC
Xét hai tam giác KED và HCB có
KDE HCB
KED HBC
nên
KED HCB
, từ đó
KE DE
BH BC
. Lại
chứng minh được
IDE IBC
nên ta được
DE IE
BC IC
.
Suy ra

IE KE KE
IC KH CM
Lại có
IEK IED KED ICH HBC ICH BCM ICM
Xét hai tam giác IEK và ICM có
IEK ICM
IE KE
IC CM
nên ta được
IEK ICM
Từ đó suy ra
KIE MIC
. Ta có
NMC IBH ABH ABI ACH ACI NIC
Do đó tứ gi{c MINC nội tiếp đường tròn nên ta được
BIH MNC MIC KIE
Nên suy ra
0
BIK BIH BIK KIE 180
. Từ đó ta được ba điểm H, I, K thẳng hang hay
BE, CD, KH đồng quy.
Ví dụ 2. Cho hai đường tròn
1
O
2
O
không bằng nhau v| tiếp xúc ngo|i tại T. Kẻ
1
OA
tiếp xúc với
2
O
tại A v|
2
OB
tiếp xúc với
1
O
tại B sao cho c{c điểm A v| B cùng
nằm trên nửa mặt phẳng có bờ l|
12
OO
. Lấy H thuộc
1
OA
v| K thuộc
2
OB
sao cho BH và
AK cùng vuông góc với
12
OO
. Đường thẳng TK cắt đường tròn
2
O
tai điểm thứ hai l| F
v| TH cắt đường tròn
1
O
tại điểm thứ hai l| E. Biết EF cắt AB tại S. Chứng minh rằng ba
đường thẳng
12
O A, O B
v| TS đồng quy tại một điểm.
Phân tích tìm lời giải
O
N
M
D
E
K
I
H
C
B
A
Nguyễn Công Lợi
TÀI LIU TOÁN HC
49
Nhận thấy tứ gi{c
12
O O AB
nội tiếp đường tròn nên ta suy ra được IT l| đường ph}n
gi{c của góc
12
O IO
. Để chứng minh được ba đường thẳng
12
O A, O B
v| TS đồng quy tại
một điểm.Ta đi chứng minh ba điểm
T, I, S
thẳng h|ng. Muốn vậy ta cần chứng minh
được
SB IB
SA IA
.
Lời giải
Gọi I l| giao điểm của
1
OA
2
OB
. Xét tứ
giác
12
O O AB
0
1 2 1 2
O AO O BO 90
nên nội
tiếp đường tròn. Do đó
1
12
2
OI
IB
O I.IA O I.IB
O I IA
nên ta có
21
AIO BIO
. Từ đó
1 1 1
2 2 2
O T O B O I
O T O A O I

, do
đó IT l| đường ph}n gi{c của góc
12
O IO
Dễ thấy
1 1 2
O BH O O B
v| từ tứ gi{c
12
O O AB
nội tiếp nên ta có
1 2 1
O O B O AB
Nên ta được
11
O BH O AB
suy ra
1 1 1 1
1 1 1 1
O B O A O T O A
O H O B O H O T
Từ đó ta được
11
O TH O AT
nên ta được
111
O AT O TH O ET
. Do vậy tứ gi{c
1
O TAE
nội tiếp được. Suy ra
0 0 0
1 1 2 1 2
EO A ETA 180 O TH O TA 180 O AT O AT 90
Từ đó ta được
1
EO
//
2
AO
. Chứng minh tương tự ta được
1
BO
//
2
FO
. Tứ đó
12
O BE O FA
và có
21
AIO BIO
nên ta suy ra
11
22
O E O T
EB IB
FA O A O T IA
12
O BE O FA
Mà ta có
1
BO
//
2
FO
nên ta được BE // FA. [p dụng định lí Talet ta được
SB EB IB
SA FA IA

,
suy ra IS l| đường ph}n gi{c trong của góc
AIB
.Mà hai góc
12
O IO
AIB
đối đỉnh với
nhau nên ba điểm
T, I, S
thẳng h|ng. Do đó ta được ba đường thẳng
12
O A, O B
và TS
đồng quy tại điểm I.
dụ 3. Cho tam gi{c ABC, gọi I l| t}m đường tròn nội tiếp tam gi{c ABC. C{c đường
thẳng AI, BI, CI cắt đường tròn ngoại tiếp tam gi{c ABC lần lượt tại D, E, K. Trên cạnh BC
lấy điểm M. Hai điểm P v| Q thỏa mãn MP song song với BE, CP vuông góc với CK, MQ
song song với CK v| BQ vuông góc với BE. Chứng minh PE, QK, DM đồng quy.
Lời giải
A
O
1
O
2
B
T
S
E
F
I
H
K
Nguyễn Công Lợi
TÀI LIU TOÁN HC
50
Từ M kẻ đường thẳng song song với BE cắt
đường ph}n gi{c ngo|i của góc
C
, đó chính
l| điểm P, thỏa mãn
CP CK
và MP song
song với BE. Tương tự điểm Q thỏa mãn
nằm trên đường ph}n gi{c ngo|i góc
B
nên
BQ BE
v| MQ song song với CK. Đường
thẳng AD v| BE cắt đường ph}n gi{c ngo|i
tại hai điểm
AB
J ,J
. Giả sử DM cắt đường
tròn ngoại tiếp
ABC
tại N. Khi đó ta có
0
11
DAE A B 90 C
22
v| cũng có
0
BB
11
BCJ 90 C BJ c A
22
Lại có MP
song song với
B
BJ
nên ta được
1
MPC A
2
.
Mặt kh{c ta có
1
DNC A
2
nên ta được
1
MPC MNC A
2

suy ra tứ gi{c MNPC l| tứ gi{c
nội tiếp đường tròn. Do đó suy ra
0
B
BCJ MNP 180
.
Tứ gi{c BEND nội tiếp nên ta được
0 0 0
11
END 180 EBD 180 A B 90 C
22
Từ
00
BB
1
BCJ 90 C END MNP BCJ MNP 180
2
suy ra ba điểm P, N , E thẳng
hàng.
Chứng minh ho|n to|n tương tự ta cũng được Q, K, N thẳng h|ng. Vậy PE, QK, DM đồng
quy.
dụ 4. Cho đường tròn t}m O. Từ điểm M nằm ngo|i đường tròn vẽ tiếp tuyến MA,
MC(A, C l| tiếp điểm), B thuộc cung lớn AC sao cho MB nằm giữa MO v| MC. Tia MB cắt
đường tròn tại Q(Q kh{c B), cắt CA tại N. Gọi K l| điểm đối xứng với C qua B. Qua Q kẻ
đường thẳng song song với BC cắt CM tại H. Chứng minh rằng QH, AC, MK đồng quy.
Bài giải
Gọi giao điểm của MO với AC
l| E. Gọi giao điểm của QH với
BC l| I. Vì MA, MC l| hai tiếp
tuyến tại A, C của đường tròn
tâm O nên
MA MC
nên M
nằm trên đường trung trực của
AC. Mặt kh{c
OA OC
nên O
O
I
N
M
P
K
Q
J
A
J
B
E
D
C
B
A
O
M
H
N
I
Q
K
E
C
B
A
Nguyễn Công Lợi
TÀI LIU TOÁN HC
51
nằm trên đường trung trực của
AC. Vì vậy OM l| đường trung
trực của AC nên OM vuông góc
với AC tại E.
Vì MC vuông góc với OC nên tam gi{c MCO vuông tại C v| OM
EC nên
2
MC ME.MO
Mặt kh{c xét tam gi{c MQC v| MCB có
CMQ
là góc chung và
MCQ MBQ
Nên ta được
MQC MCB
do đó suy ra
2
MQ MC
MQ.MB MC
MC MB
Từ đó
MQ ME
MQ.MB ME.MO
MO MB
. Suy ra
MQE MOB
nên ta được
MEQ MBO
hay
MEQ QBO
. Lại có tứ gi{c QEOB có
MEQ MBO
nên tứ gi{c QEOB l| tứ gi{c nội
tiếp
Do đó ta được
BEO BQO
. M| tam gi{c BQO c}n tại O nên
QBO BQO BEO BQO
Từ đó ta được
MEQ BEO
. Mà
00
QEC MEQ 90 , BEO CEB 90
nên
QEC CEB
do đó
EC l| tia ph}n gi{c của
QEB
. Mà HA là phân giác ngoài
QEB
nên theo tính chất ph}n
giác suy ra
NQ EQ MQ EQ NQ MQ
,
NB EB MB EB NB MB
Mặt kh{c QH//BC nên ta có
MQ MH HQ
MB MC BC

và QI//CB nên ta có
QI NQ
CB NB
Từ đó ta được
HQ QI
BC CB
nên Q l| trung điểm của HI. Trong tam gi{c MCB có HQ//BC nên
MH QH
MC BC
hay
MH 2QH HI
MC 2BC CK

nên
MH HI
MC CK
. Kết hợp với
MHI MCK
ta được
MHI MCK
suy ra
nên hai tia MI, MK trùng nhau. Do đó M, I, K thẳng
h|ng hay QH, AC, MK đồng quy.
Ví dụ 5. Cho nửa đường tròn (O; R) đường kính AB. Trên nửa mặt phẳng bờ AB chứa nửa
đường tròn vẽ c{c tia Ax v| By vuông góc với AB. Lấy điểm M nằm trên nửa đường
tròn(M kh{c A, B). Tiếp tuyến tại M cắt Ax v| By lần lượt tại C v| D. Đường trong ngoại
tiếp tam gi{c OCD cắt nửa đường tròn (O) tại E v| F. Chứng minh rằng AD, BC v| EF
đồng tại một điểm.
Phân tích và lời giải
Gọi N l| giao điểm của AD v| BC. Ta đi chứng minh rằng ba điểm E, F, N thẳng h|ng.
Nguyễn Công Lợi
TÀI LIU TOÁN HC
52
Dễ thấy AC//BD,
AC CM, BD DM
0
COD 90
.
Gọi I l| trung điểm của CD, H l| giao điểm của MN
với AB, K v| S lần lượt l| giao điểm của EF với OM
và OI. Trong tam giác NBO có AC//BD nên ta có
CN AC CN CM
MN BD MB MD
. Từ đó suy ra MN//BD//AC.
Ta có
MN DM BN HN
AC CD BC AC
nên ta được
MN NH
.
Vẽ đường kính OJ của đường tròn (I) đường kính
CD. Hai đường tròn (O) v| (I) cắt nhau tại E v| F.
Suy ra OI l| đường trung trực của EF, nên ta được
OI EF
, do đó EF//AB.
Tam gi{c EOJ vuông tại E có ES l| đường cao nên
2
OE OS.OJ
.
Từ đó ta được
22
1
2OS.OI OM OS.OI OM
2
.
Mà ta có
OSK OMI
nên ta được
OS OK
OS.OI OM.OK
OM OI
.
Do đó ta được
2
11
OM.OK OM OK OM
22
hay K l| trung điểm của MO.
M| ta lại có EF//AB, do đó EF đi qua N. Vậy ba điểm E, F, N thẳng h|ng.
dụ 6. Cho tam gi{c ABC nội tiếp đường tròn (O). Đường tròn t}m A cắt BC tại D, E v|
cắt đường tròn (O) tại G, H sao cho D nằm giữa hai điểm B, E v| tia AB nằm giữa hai tia
AC, AG. Đường tròn ngoại tiếp c{c tam gi{c BDG v| CEH cắt lần lượt AB, AC tại K, L
kh{c B, C. Chứng minh rằng GK v| HL cắt nhau tại một điểm trên OA.
Trích đề thi Olympic Toán Quốc tế - IMO 2015
Phân tích và lời giải
Gọi giao điểm của GK với LH l| X. Ta cần
chứng minh X thuộc OA.
Thật vậy, dễ thấy AG v| AH l| b{n kính đường
tròn t}m A nên ta được
AG AH
, đồng thời G
v| H thuộc đường tròn (O) nên ta
OG OH
từ đó suy ra AO l| đường trung trực của GH.
Ta chỉ cần chứng minh X thuộc đường trung
trực của GH l| b|i to{n được chứng minh.
Thật vậy, dễ thấy tgi{c HCBG nội tiếp đường
tròn (O) nên ta được
0
GHC 180 GBD
v| tứ
J
H
O
F
E
I
S
K
N
M
D
C
B
A
O
H
K
G
L
X
E
D
C
B
A
Nguyễn Công Lợi
TÀI LIU TOÁN HC
53
gi{c GHED nội tiếp đường tròn tâm A nên ta
được
0
GDE 180 GHE
Khi đó ta có biến đổi sau
0
EHC GHC GHE 180 GBD GDB BGD
Do tứ gi{c KGBD nội tiếp nên ta
KGD KBD
v| do tứ gi{c HGDE nội tiếp đường tròn
tâm A nên ta
. Lại
GBA HCA
(hai go{c nội tiếp chắn hai cung bằng
nhau) và
EHC BGD
Lại do tứ gi{c GDEH nội tiếp nên
.Khi đó ta có biến đổi sau
0
0
XGH XGD HGD KBD HEC 180 GBA BGD BDG HEC
180 HCA EHC EHG HEC ACB GHE XHE GHE XHG
Do đó ta được tam gi{c XGH c}n tại X hay X tuộc đường trung trực của GH. Vậy X thuộc
AO nên GK v| HL cắt nhau tại một điểm trên OA.
Ví dụ 7. Cho tam gi{c nhọn ABC có c{c đường cao BD v| CE cắt nhau tại H. Gọi M, N lần
lượt l| trung điểm của c{c đoạn thẳng AH, BC. Chứng minh rằng MN v| c{c đường ph}n
gi{c của c{c góc
ABD; ACE
đồng quy tại một điểm.
Phân tích tìm lời giải
Gọi K l| giao điểm của c{c đường ph}n gi{c của c{c góc
ABD; ACE
khi đó
KBD KCD
. Lại thấy
0
BEC BKC BDC 90
. Nên c{c điểm B, E, K, D, C cùng nằm trên
đường tròn t}m N. M| ta lại
ECK KCD
nên điểm K nằm chính giữa cung DE của
đường tròn t}m N đường kính BC. M| ta lại thấy c{c điểm A, E, H, D nằm trên đường
tròn t}m M. Nên hai đường tròn t}m M v| t}m N cắt nhau tại D v| E. Như vậy MN đi qua
điểm chính giữa cung DE của đường tròn t}m N hay MN đi qua điểm K. Vậy MN v| c{c
đường ph}n gi{c của c{c góc
ABD; ACE
đồng quy tại một điểm.
Lời giải
Gọi K l| giao điểm của c{c đường ph}n gi{c của
ABD
ACE
. Dễ thấy
ABH ACH
do cùng phụ
với góc
BAC
.
M| BK v| CK l| ph}n gi{c của c{c góc
ABD; ACE
nên ta được
ABH ACH
KBD KCD
22
. Từ đó ta
được
0
KBC KCB DBC KBD KCB
DBC KCD KCB 90
K
D
E
N
M
C
B
A
Nguyễn Công Lợi
TÀI LIU TOÁN HC
54
Suy ra
0
BKC 90
nên ta được
0
BEC BKC BDC 90
. Từ đó suy ra c{c điểm B, E, K, D, C
cùng nằm trên đường tròn t}m N. M| ta lại có
nên điểm K nằm chính giữa
cung DE của đường tròn t}m N đường kính BC. Mặt kh{c ta lại có
0
AEH ADH 90
nên
c{c điểm A, E, H, D nằm trên đường tròn t}m M. Như vậy hai đường tròn t}m M v| t}m
N cắt nhau tại D v| E. Như vậy MN đi qua điểm chính giữa cung DE của đường tròn t}m
N hay MN đi qua điểm K. Vậy MN v| c{c đường ph}n gi{c của c{c góc
ABD; ACE
đồng
quy tại một điểm.
dụ 8. Chọn s{u điểm trên c{c cạnh của một tam gi{c đều ABC sao cho
12
A ; A
trên BC,
12
B ; B
trên CA,
12
C ; C
trên AB. Những điểm n|y l| đỉnh của một lục gi{c lồi
1 2 1 2 1 2
A A B B C C
với c{c cạnh bằng nhau.
Chứng minh rằng c{c đường thẳng
1 2 1 2
A B , B C
12
CA
đồng qui.
Lời giải
Bên trong tam gi{c đều ABC lấy điểm M sao
cho tứ gi{c
1 2 1
C C A M
l| hình thoi. Khi đó
2 1 1 1
C A A M MC
. Điều n|y dẫn đến tam gi{c
12
A MA
đều. Nên
12
A M A M
. Suy ra
2 1 2
MA B B
cũng l| hình thoi. Do đó
22
MA MB
.
Từ đó
1 2 2 1
MA MA MB MC
. Như vậy tứ gi{c
1 2 2 1
A A B C
l| tứ gi{c nội tiếp. Suy ra
0
1 1 2 1 2
1
A C A A MA 30
2

. Ho|n to|n tương tự ta
được
0
1 1 2 1 2
1
C A B C MB 30
2

.
Trong tam gi{c đều ABC lấy c{c điểm
1
M
2
M
sao cho c{c tứ gi{c
2 1 1 2
B M C C
1 2 2 1
B M B C
l| c{c hình thoi. Chứng minh ho|n to|n tương tự ta được hai tứ gi{c
1 1 2 2
A B B C
2 1 1 2
A B C C
nội tiếp. Từ đó ta được
0
1 1 2 1 1 2 1 1 2 1 1 2
A B C B A B B C A C B C 30
. Xét tam
giác
1 1 1
A B C
1 2 1 2 1 2
A B , B C , C A
l| c{c đường ph}n gi{c trong nên chúng đồng quy. Vậy ta
có điều phải chứng minh.
dụ 9. Cho điểm C thay đổi trên nửa đường tròn đường kính
A
B 2R A C, B C
.
Gọi H l| hình chiếu vuông góc của C lên AB. Gọi I v| J lần lượt l| t}m đường tròn nội tiếp
tam gi{c ACH v| BCH. C{c đường thẳng CI v| CJ cắt AB lần lượt tại M v| N. Chứng minh
rằng c{c đường thẳng MJ, NI, CH đồng quy.
Phân tích tìm lời giải
M
2
M
1
C
2
C
1
B
2
B
1
A
2
A
1
M
C
B
A
Nguyễn Công Lợi
TÀI LIU TOÁN HC
55
Để chứng minh ba đường thẳng MJ, NI, CH đồng quy ta đi chứng minh MJ, NI,
CH l| ba đường cao của tam gi{c CMN. Muốn vậy ta cần chứng minh được
0
CIN CJM 90
. Điều n|y sẽ được khẳng định khi ta chứng minh được c{c tứ gi{c CIHN
v| CMHJ nội tiếp đường tròn
Lời giải
Vì I, J l| t}m đường tròn nội tiếp tam gi{c
ACH và BCH nên
1
ICH ACH
2
1
JCH BCH
2
. Ta có
0
1
MCN MCH NCH ACH BCH 45
2
Lại có HI l| đường ph}n gi{c của góc
AHC
nên suy ra
00
11
MHI AHC 90 45
22
Từ đó ta được
nên tứ gi{c CIHN nội tiếp nên suy ra được
0
CIN CHN 90
.
Do đó ta được NI vuông góc với CM. Chứng minh tương tự ta được tứ gi{c CJHM nội tiếp
suy ra
0
CJM CHM 90
nên MJ vuông góc với CN. Trong tam gi{c CMN có c{c đường
cao CH, MJ, NI chúng đồng quy tại một điểm.
Ví dụ 10. Cho hai đường tròn
O
và
O'
cắt nhau tại A v| B. Giả sử CD v| EF l| hai tiếp
tuyến chung ngo|i của hai đường tròn n|y
C
, E O ; D, F O'
v| điểm A gần CD hơn.
Gọi
1
l| đường thẳng qua A tiếp xúc với đường tròn ngoại tiếp tam gi{c AEF v|
2
đường thẳng qua B tiếp xúc với đường tròn ngoại tiếp tam gi{c BCD. Chứng minh rằng
c{c đường thẳng
1
,
2
, CD, EF đồng quy.
Phân tích tìm lời giải
Gọi M l| điểm đối xứng với B qua EF. Khi đó ta
EMF EBF
nên
0
EMF EAF 180
nên tứ gi{c MEAF nội tiếp đường tròn. Gọi N l| giao điểm của c{c tiếp
tuyến tại A, M của đường tròn ngoại tiếp tứ gi{c MEAF. Ta sẽ chứng minh c{c đường
thẳng
1
,
2
, CD, EF. Muốn vậy ta cần chứng minh được ba điểm E, F, N thẳng h|ng. Khi
đó N thuộc đường tròn ngoại tiếp tam gi{c ABF. Do đó N thuộc đường trung trực của
đoạn thẳng AB nên N thuộc đường thẳng OO’. Chứng minh tương tự ta
2
v| CD cắt
nhau tại một điểm N’ thuộc đường thẳng OO’. Do tính chất đối xứng CD v| EF cắt nhau
tại một điểm thuộc đường thẳng OO’, do đó hai điểm N v| N’ trùng nhau. Vậy c{c đường
thẳng CD, EF,
1
2
đồng quy tại N.
J
I
N
M
H
C
B
A
Nguyễn Công Lợi
TÀI LIU TOÁN HC
56
Lời giải
Gọi M l| điểm đối xứng với B
qua EF. Khi đó ta có
EMF EBF
.
Mà ta có
0
EBF EAF EBF EAB FAB
EBF BEF BFE 180
Do đó ta được
0
EMF EAF 180
nên tứ gi{c MEAF nội tiếp
đường tròn. Gọi N l| giao điểm
của c{c tiếp tuyến tại A, M của
đường tròn ngoại tiếp tứ gi{c
MEAF.
Ta cần chứng minh ba điểm N, E, F thẳng h|ng. Thật vậy, gọi F’ l| giao điểm thứ hai của
đường tròn ngoại tiếp tứ gi{c MEAF với NE. Khi đó dễ thấy hai tam gi{c NAE v| NF’A
đồng dạng nên
AE NA
AF ' NF '
Ho|n to|n tương tự ta có
ME MN NA
MF ' NF ' NF '

. Từ kết quả trên ta được
AE ME
AF ' MF '
nên
AF ' AE
MF ' ME
.
Gọi I l| giao điểm của AB v| EF, ta có
22
IE IA.IB IF
nên
IE IF
Ta lại có
IEB IAE
nên ta được
EB IF IE
EA IA IA

. Tương tự ta có
BF IF
AF IA
nên
EB BF
EA AF
.
Do vậy ta có
ME MF
AE AF
hay
AE AF
ME MF
. Kết hợp với
AF ' AE
MF ' ME
ta được hai điểm F v| F’
trùng nhau. Do đó ba điểm N, E, F thẳng h|ng. Ta có N thuộc đường tròn ngoại tiếp tam
gi{c ABF. Do đó N thuộc đường trung trực của đoạn thẳng AB, suy ra N thuộc đường
thẳng OO’. Chứng minh tương tự ta có
2
CD cắt nhau tại một điểm N’ thuộc đường
thẳng OO’. Do tính chất đối xứng CD v| EF cắt nhau tại một điểm thuộc đường thẳng
OO’, do đó hai điểm N v| N’ trùng nhau. Vậy c{c đường thẳng CD, EF,
1
2
đồng
quy tại N.
Ví dụ 11. Cho lục gi{c ABCDEF có c{c cặp cạnh đối diện song song với nhau. Gọi M, N, P,
Q, R, S lần lượt l| trung điểm của AB, BC, CD, DE, EF, FA. Chứng minh rằng c{c đường
thẳng MQ, NR, PS đồng quy.
Phân tích tìm lời giải
1
O'
O
N
M
F
E
D
C
B
A
Nguyễn Công Lợi
TÀI LIU TOÁN HC
57
Nhận thấy nếu lục gi{c ABCDEF AD, BE, CF đồng quy tai một điểm O khi đó
hiển nhiên đường thẳng MQ, NR, PS đồng quy. Để ý l| khi AD, BE, CF không đồng quy
thì chúng cắt nhau theo đôi một tạo th|nh một tam gi{c nằm trong lục gi{c. Khi đó gọi O
l| giao điểm của MQ v| NR(điểm O nằm trong tam gi{c XYZ) v| ta cần chứng minh PS đi
qua điểm O. Chú ý đến c{c trung điểm ta được
OQD OQE
SS
XQD XQE
SS
nên
OXE OXD
SS
.
Tương tự ta được
OXA OXB
SS
nên
OAD OBE
SS
. Cũng tương tự như trên t ta được
OBE OCF
SS
. Do đó nên ta được
OAD OCF
SS
. Lại để ý l| khi điểm O không thuộc PS tta
thấy
OAD OCF
SS
. Nên ta sử dụng phép phản chứng để chứng minh tiếp bài toán.
Lời giải
Ta xét c{c trường hợp có thể xẩy ra như sau:
+ Trường hợp 1: Nếu AD, BE, CF đồng quy
tại một điểm O. Khi đó theo định lí Talets ta
OB AB 2BM BM
OE ED 2EQ EQ
Mặt kh{c ta có BM//EQ v| B, O, E thẳng
h|ng nên suy ra M, O, Q hay MQ đi qua
điểm O. Ho|n to|n tương tự ta cũng có NR,
SP đi qua điểm O. Vậy MQ, NR, PS đồng
quy tại O.
+ Trường hợp 2: Nếu AD, BE, CF không đồng quy. Khi đó chứng cắt nhau tạo th|nh tam
giác XYZ.
Lập luận tương tự trường hợp 1 ta MQ, NR, PS
lần lượt đi qua X, Y, Z. Gọi O l| giao điểm của
MQ v| NR(điểm O nằm trong tam gi{c XYZ).
Ta có
OQD OQE
SS
XQD XQE
SS
nên ta suy ra
được
OXE OXD
SS
. Tương tự ta được
OXA OXB
SS
nên ta được
OAD OBE
SS
. Cũng tương tự ta
chứng minh được
OBE OCF
SS
nên ta được
OAD OCF
SS
Giả sử điểm O không thuộc đường thẳng SZP, không mất tính tổng qu{t ta xem điểm A,
O, C cùng nằm một phía so với SP. Khi đó ta được
OASX OFSZ
SS
m| ta lại có
ZAS ZSF
SS
nên
suy ra
OAZ OFZ
SS
Tương tự ta cũng có
PDZ OPCZ
SS
OPD OCP
SS
nên ta được
ODZ OCZ
SS
O
S
R
Q
P
N
M
F
E
D
C
B
A
Z
Y
X
S
R
O
Q
P
N
M
F
E
D
B
C
A
Nguyễn Công Lợi
TÀI LIU TOÁN HC
58
Do đó ta được
OAD OCF
SS
, điều nay m}u thuẫn với
OAD OCF
SS
. Suy ra điểm O phải thuộc
đường thẳng SP hay SP đi qua O. Vậy ba đường thẳng MQ, NR, PS đồng quy tại O.
dụ 12. Cho tam gi{c nhọn ABC nội tiếp đường tròn
O
. Gọi
1 1 1
A , B ,C
2 2 2
A , B ,C
lần
lượt l| ch}n đường cao của tam gi{c ABC kẻ từ A, B, C v| c{c điểm đối xứng với
1 1 1
A , B ,C
qua trung điểm của BC, CA, AB. Gọi
3 3 3
A , B , C
lần lượt l| giao điểm thứ hai của c{c đường
tròn ngoại tiếp
2 2 2 2
AB C , BA C
22
CA B
với đường tròn (O). Chứng minh rằng
1 3 1 3 1 3
A A , B B , C C
đồng quy.
Phân tích và lời giải
Ta sẽ chứng minh
1 3 1 3 1 3
A A , B B , C C
cùng đi qua trọng t}m G của tam gi{c ABC. Gọi M l|
trung điểm của BC, A’ l| điểm đối xứng với A qua trung trực của BC. Ta sẽ chứng minh
A’ trùng với
3
A
. Thật vậy, gọi I l| giao điểm của AA’ v| trung trực của BC.
Ta có
A, A
đối xứng nhau qua trung trực của BC nên
CA BA ,CA BA CAA BA A
. Suy ra
ACA ABA

nên ta được A’ thuộc đường
tròn (O). Mà
11
ABB ACC
nên suy ra
1
1
AC
AC
AB AB
. Do A, B và
12
C , C
cùng đối xứng nhau
qua trung điểm của AB nên
21
BC AC
. Chứng minh ho|n to|n tương tự ta được
21
B C AB
.
Ta có
21
21
BC AC
AC BA
CB AB AB CA
. M| ta lại có
22
C BA B CA

nên ta được
22
C BA B CA


Từ đó suy ra
2 2 2 2
A C B A B C AC A AB A
nên tứ gi{c
22
AC B A
l| tứ gi{c nội tiếp.
Suy ra
3
AA
. Gọi G l| giao điểm của AM v|
13
AA
. Lại có
1
AA
//
MI
AI
//
1
MA
từ đó
ta được tứ gi{c
1
AIMA
l| hình bình h|nh. Do đó
1
13
3
MA
11
AI MA AA
2 AA 2
. Vì
1
MA
//
C
3
B
3
A
3
C
2
B
2
A
2
C
1
B
1
A
1
C
B
A
M
I
O
G
A'
B
2
C
1
A
1
C
B
A
Nguyễn Công Lợi
TÀI LIU TOÁN HC
59
3
AA
nên theo định lí Talets ta được
1
3
MA
MG 1
GA AA 2

. Suy ra G l| trọng t}m tam gi{c ABC.
Vậy
13
AA
đi qua G. Tương tự
1 3 1 3
B B ,C C
đi qua G. Suy ra
1 3 1 3 1 3
A A , B B , C C
đồng quy tại G.
Ví dụ 13. Cho tam gi{c ABC nội tiếp đường tròn (O). Một đường tròn
1
K
tiếp xúc với
c{c cạnh AB, AC v| tiếp xúc trong với đường tròn (O) lần lượt tại
1 1 1
M , N , P
. C{c điểm
2 2 2
M , N , P
3 3 3
M , N , P
đượcx{c định một c{ch tương tự. Chứng minh rằng c{c đoạn thẳng
1 1 2 2 3 3
M N , M N , M N
cắt nhau tại trung điểm của mỗi đoạn.
Phân tích và lời giải
Gọi E, F lần lượt l| giao điểm của
1
AP
,
1
CP
với
đường tròn
1
K
. Gọi D l| giao điểm của
11
NP
với
đường tròn (O). Ta sẽ chứng minh D l| điểm chính
giữa cung nhỏ
AC
Thật vậy, gọi
1
Px
l| tiếp tuyến của đườngtròn (O)
tại
1
P
, khi đó ta có
1 1 1 1
NAE CAP CAx FP X FN P
Ta lại có tứ gi{c
11
EN FP
nội tiếp nên
1 1 1
N EA N FP
.
Từ đó ta được
1 1 1
AEN N FP
nên
1 1 1 1 1
FPN AN E AP N
Do đó
11
PN
l| ph}n gi{c của
1
APC
nên D l| điểm
chính giữa cung nhỏ
AC
Từ đó ta suy ra BD l| ph}n gi{c của góc
ABC
. Gọi I l| giao diểm của
11
MN
v| BD. Khi đó
ta suy ra được
1 1 1 1 1 1 1 1 1 1
IM P N M P N P x DP x DBP IBP
. Suy ra tứ gi{c
11
IM BP
nên ta
được
1 1 1
P IB PM B
m| ta lại có
1 1 1 1 1
P M B P N M
. Do đó
1 1 1 1 1 1
P IB P N M DIP DN I
nên
11
DIP DN I
suy ra
2
1
11
1
DP
DI
DI DN .DP
DN DI
. Ta cũng có
1 1 1 1 1
DAN CP N DP N
nên
11
DAN DP A
suy ra
2
1
11
1
DN
DA
DA DN .DP
DP DA
. Do đó ta được
AD DI
nên tam giác
ADI c}n tại D
Suy ra
1 1 1
ABC ABC
DIA DAI ABI IAB DAN N AI IAB N AI
22
Do đó ta được
1
IAB N AI
nên AI l| ph}n gi{c của góc
BAC
. Vậy I l| giao điểm hai đường
ph}n gi{c trong của tam gi{c ABC nên I l| t}m đường tròn nội tiếp tam gi{c ABC. Dễ thấy
x
D
I
F
E
P
1
N
1
K
1
M
1
C
B
A
Nguyễn Công Lợi
TÀI LIU TOÁN HC
60
11
AM N
c}n tại A v| AI l| ph}n gi{c của góc
BAC
nên AI l| đường trung tuyến của
11
MN
.
Nên I l| trung điểm của
11
MN
. Ho|n to|n tương tự ta cũng được I l| trung điểm của
2 2 3 3
M N , M N
. Vậy c{c đoạn thẳng
1 1 2 2 3 3
M N , M N , M N
cắt nhau tại trung điểm của mỗi đoạn.
Ví dụ 14. Cho tam gi{c ABC v| c{c điểm
1 1 1
A ; B ;C
lần lượt nằm trên cạnh BC, CA, AB. Gọi
G l| trọng t}m tam gi{c ABC. Gọi
a b c
G ;G ;G
lần lượt l| trọng t}m c{c tam gi{c
1 1 1 1 1 1
AB C ; BC A ; CA B
. Chứng minh rằng c{c đường thẳng
a b c
AG ; BG ;CG
đồng quy khi
chỉ khi
111
AA ; BB ; CC
đồng quy.
Phân tích và lời giải
Theo định lí Ceva thì ba đường thẳng
111
AA ; BB ; CC
đồng quy khi v| chỉ khi
1 1 1
1 1 1
AC BA CB
. . 1
BC CA AB
.
Gọi giao điểm tương ứng của
a b c
AG ; BG ;CG
với
BC, CA, AB lần lượt l|
2 2 2
A ; B ;C
. Khi đó ba đường
thẳng
a b c
AG ; BG ;CG
đồng quy khi v| chỉ khi
2 2 2
2 2 2
CA BC AB
. . 1
BA AC CB
.
Như vậy ta cần chứng minh được
2 2 2 2 2 2
2 2 2 2 2 2
CA BC AB CA BC AB
. . . . 1
BA AC CB BA AC CB

Thật vậy, gọi
3
A
l| trung điểm của
11
BC
, gọi
12
h ;h
lần lượt l| đường cao của c{c tam gi{c
2
AA C
31
AA B
tương ứng hạ từ
23
A ; A
. Khi đó ta có
2
31
AA C
12
AA B 2 1 3 1
S
h .AC AA .AC
S h .AB AA .AB

. Tương tự
ta cũng có
2
31
AA B
2
AA C 3 1
S
AA .AB
S AA .AC
. Từ đó
22
3 1 3 1
AA C AA B
22
AA B AA C 3 1 3 1
SS
AA .AC AA .AB
::
S S AA .AB AA .AC
hay
2 3 1
3 1 2
AA C AA C
1
AA B AA B 1
SS
AC
AC
..
S S AB AB
Mặt kh{c do
3
A
l| trung điểm của
11
BC
nên
3 1 3 1
AA B AA C
SS
.Từ đó suy ra
2
2
AA C
1
AA B 1
S
AC
AC
.
S AB AB
.
M| ta cũng có
2
2
AA C
2
AA B 2
S
AC
S A B
. Do đó ta được
21
21
A C AC
AC
.
A B AB AB
.
Chứng minh ho|n to|n tương tự thì ta được
2 1 2 1
2 1 2 1
BC CB AB BA
CB BA
. ; .
AC CA CA CB BC BC

Nh}n theo vế c{c đẳng thức trên thì ta được
2 2 2 2 2 2
2 2 2 2 2 2
CA BC AB CA BC AB
. . . . 1
BA AC CB BA AC CB

.
C
2
B
2
A
3
A
2
G
c
G
b
G
a
C
1
B
1
A
1
C
B
A
Nguyễn Công Lợi
TÀI LIU TOÁN HC
61
Suy ra c{c đường thẳng
222
AA ;BB ;CC
đồng quy, điều n|y dẫn đến ba đường thẳng
a b c
AG ; BG ;CG
đồng quy. Vậy c{c đường thẳng
a b c
AG ; BG ;CG
đồng quy khi v| chỉ khi
111
AA ; BB ; CC
đồng quy
dụ 15. Cho tam gi{c ABC v| D l| một điểm bất không thuộc c{c đường thẳng AB v|
AC. Trên đoan thẳng AD lấy điểm M(M kh{c A, D). Gọi I một điểm thuộc đoạn AD(I kh{c
A, D v| kh{c giao điểm của AD với BC). Gọi giao điểm của BI, CI lần lượt với AC, AB l| E,
F. Gọi giao điểm của DE, DF lần lượt với CM, BM l| K, H. Chứng minh rằng c{c đường
thẳng CM, AD, BK đồng quy.
Phân tích tìm lời giải
Trên cơ sở hình vẽ v| giả thiết ta thấy có nhiều bộ ba điểm thẳng h|ng nên theo định
lí Menelaus ta sẽ lập được c{c đẳng thức về tỉ số như
DM FA HB
. . 1
DA FB HM
DM EA KC
. . 1
DA EC KM
. Từ đó ta thu được
EC FA KC HM
..
EA FB KM HB
. Ta lại ba đường thẳng AG, BE, CF đồng quy
nên
GB EC FA
. . 1
GC EA FB
. Từ đó ta rút ra được
GC EC FA
.
GB EA FB
nên
GC KM HM
.
GB KC HB
hay
GB KC HM
. . 1
GC KM HB
. M| theo định Ceva thì hiển nhiên c{c đường thẳng CM, AD, BK đồng
quy.
Lời giải
Gọi G l| giao điểm của AD v| BC. [p dụng định lí
Menelaus cho tam gi{c AMB với bộ ba điểm F, H, D
thẳng h|ng ta được
DM FA HB
. . 1
DA FB HM
.
[p dụng định lí Menelaus cho tam gi{c AMC với bộ
ba điểm K, K, D thẳng h|ng ta được
DM EA KC
. . 1
DA EC KM
. Từ đó ta được
DM FA HB
..
EC FA KC HM
DA FB HM
1 . .
DM EA KC EA FB KM HB
..
DA EC KM
.
Ta lại có ba đường thẳng AG, BE, CF đồng quy nên theo định lí Ceva ta được
GB EC FA
. . 1
GC EA FB
Từ đó ta được
GC EC FA
.
GB EA FB
, kết hợp với hệ thức
EC FA KC HM
..
EA FB KM HB
Suy ra
GC KM HM
.
GB KC HB
hay ta được
GB KC HM
. . 1
GC KM HB
.
Như vậy theo định lí Ceva thì c{c đường thẳng CH, BK, MG đồng quy.
H
K
F
E
G
I
M
D
C
B
A
Nguyễn Công Lợi
TÀI LIU TOÁN HC
62
Ví dụ 16. Cho tam giác ABC có
AB AC
. Đường ph}n gi{c của góc
BAC
cắt BC tại V. Gọi
D l| ch}n đường vuông góc hạ từ A xuống BC. Đường tròn ngoại tiếp tam gi{c DAV cắt
AB, AC lần lượt tại F, E. Chứng minh rằng ba đường thẳng AD, BE, CF đồng quy tại một
điểm.
Phân tích tìm lời giải
Để chứng minh AD, BE, CF đồng quy tại một điểm ta cần chỉ ra được
DB EC FA
. . 1
DC EA FB
hay ta cần chứng minh
DB EC EA
.
DC FB FA
. Chú ý
EA FA
nên để kết thúc
chứng minh ta cần phải chỉ ra được
DB FB
DC EC
Lời giải
Ta có
0
ADV 90
nên tam gi{c ADV nội tiếp
đường tròn đường kính AV. Từ đó suy ra
0
AFV AEV 90
nên ta được
0
BFV 90
. Xét hai
tam giác VFB và ABD có
ABD
là góc chung và
VFB ADB
nên
ADB VFB
. Từ đó ta được
DB AB FB VB
FB VB DB AB
. Chứng minh ho|n to|n
tương tự ta được
EC VC
DC AB
.
Lại có AV l| đường ph}n gi{c của tam gi{c ABC nên ta được
VB VC
AB AC
. Kết hợp c{c kết
quả trên ta được
FB EC
DB DC
hay ta được
DB FB
DC EC
. Xét hai tam giác AFV và AEV có
0
AFV AEV 90
, AV chung và
FAV EAV
. Từ đó
AFV AEV
nên suy ra
FA EA
.
Do đó
DB FB DB EC
.1
DC EC DC FB
.
Từ đó ta được
DB EC FA
. . 1
DC EA FB
. Như vậy theo định lí Ceva thì ba đường thẳng AD, BE, CF
đồng quy.
Ví dụ 17. Cho tam gi{c nhọn ABC nội tiếp đường tròn (O; R) có H l| trực t}m. Chứng
minh rằng tồn tại c{c điểm D, E , F lần lượt trên c{c cạnh BC, CA, AB sao cho
OD HD OE HE OF HF
v| ba đường thẳng AD, BE, CF đồng quy.
Lời giải
D
F
E
V
C
B
A
Nguyễn Công Lợi
TÀI LIU TOÁN HC
63
Gọi L l| giao điểm của AH với đường tròn (O). Gọi
D l| giao điểm của OL v| BC. Nối D với H, khi đó ta
chứng minh được
HD DL
. Từ đó ta được
OD DL OD DH OL R
Tương tự ta được
OE EH OF HF R
. Nối OB,
OC v| BL thì ta được
BOC 2BAC
0
BOC 180 2OBC
.
Do đó ta được
00
2OBC 180 2BAC OBC 90 BAC
V| ta lại có
0
CBL CAL 90 ACB
. Từ đó ta được
00
OBL OBC CBL 90 BAC 90 ACB ABC
.
OB L R
nên ta được
OLB OBL ABC
.
Do đó ta được
00
BLO 180 2ABC BOD 180 2ABC
.
Như vậy trong tam gi{c BOD có
0
BOD 180 2ABC
. Ho|n to|n tương tự ta được trong
tam giác COD có
0
COD 180 2ACB
. Theo định lí sin ta có
BD OD
sin BOD sin OBD
CD OD
sin COD sin OCD
Mà ta có
nên ta được
OD OD
sin OBD sin OCD
. Từ đó ta được
BD CD
sin BOD sin COD
Hay ta được
0
0
sin 180 2ABC
BD sin BOD
CD
sin COD
sin 180 2ACB

Chứng minh ho|n to|n tương tự ta được
00
00
sin 180 2ACB sin 180 2BAC
EC FA
;
EA FB
sin 180 2BAC sin 180 2ABC



.
Do đó ta được
0 0 0
0 0 0
sin 180 2ABC sin 180 2ACB sin 180 2BAC
BD EC FA
. . . . 1
CD EA FB
sin 180 2ACB sin 180 2BAC sin 180 2ABC

.
Từ đó theo định lí Ceva ta được AD, BE, CF đồng quy.
Ví dụ 18. Cho tam gi{c nhọn ABC có I, O l|n lượt l| t}m đường tròn nội tiếp v| ngoại tiếp
tam gi{c. Đường thẳng AI cắt đường tròn (O) tại điểm thứ hai l| D kh{c A. Gọi E v| F l|
hai điểm lấn lượt nằm trên cạnh BC v| cung
BDC
sao cho
1
BAF CFE BAC
2

. Gọi G l|
trung điểm của EF. Chứng minh rằng GD, EI cắt nhau tại một điểm nằm trên đường tròn
(O).
H
K
Q
P
L
O
F
E
D
C
B
A
Nguyễn Công Lợi
TÀI LIU TOÁN HC
64
Lời giải
Gọi giao điểm thứ hai của EI với đường tròn (O) l|
X, giao điểm của AD v| BC l| L. Gọi G’ v| T lần lượt
l| giao điểm của DX cới IF v| AF. Đường thẳng AF
cắt đường tròn (O) tại điểm thứ hai l| K. Ta có
BAK CAE
nên ta được
BK CE
suy ra KE//BC.
Lại có
suy ra
BAD BAK DAC CAE
hay ta được
KAD DAE
nên
IAT IXT
. Từ đó suy
ra tứ gi{c ITXA nội tiếp đường tròn. Do đó
ITA IXA EXA EKA
.
Từ đó ta được IT//KE//BC, nên suy ra
TF IL
AT AI
.
Vì CI l| đường ph}n gi{c của tam gi{c ACL nên
IL CL
AI AC
.
Chứng minh được
CLD ACD
nên ta được
CL DC
AC AD
.
Ta có
1 1 1
ICD ICB BCD ACB BAC sdABC; IDC sdAC
2 4 2
Từ đó ta được
00
1 1 1
CID 180 ICD IDC 180 sdABC sdAC sdACB
4 2 4
Từ đó suy ra
ICD DIC
nên tam gi{c ICD c}n tại D, suy ra
CD ID
DI DC
AD AD
Kết hợp c{c kết quả trên ta thu được
TF ID TF AD
.1
TA AD TA ID
.
[p dung định lí Menelaus cho tam gi{c AIF với ba điểm T, D, G’ thẳng h|ng ta được
TF AD IG'
. . 1
TA DI G ' F
.
Kết hợp với
TF AD
.1
TA ID
ta được
G ' I
1 G ' I G' F
G ' F
nên G’ l| trung điểm của IF. Đo đó
hai điểm G v| G’ trùng nhau. Vậy GD, EI cắt nhau tại một điểm nằm trên đường tròn (O).
Nhận xét: Bài toán này có đường giải quyết theo một hướng khác. Để chứng minh
TF ID
TA AD
ta
có thể quy về chứng minh
TF TA ID AD TA AF
TA AD AD ID AD

.
dụ 19. Cho tam gi{c ABC nhọn nội tiếp đường tròn (O), đường cao AD. Đường tròn
(A) bất t}m A. Gọi E v| F l| hai điểm trên đường tròn (A) sao cho E, F đối xứng với
nhau qua AD v| tia AE nằm giữa hai tia AB, AF. Đường tròn (A) cắt đường tròn (O) tai G
v| H sao cho tia AB nằm giữa hai tia AG v| AC. Giao điểm của CE, BF với đường tròn (O)
G'
G
X
L
K
E
F
D
T
O
I
C
B
A
Nguyễn Công Lợi
TÀI LIU TOÁN HC
65
lần lượt l| P, Q kh{c E, F. Đường tròn ngoại tiếp tam gi{c BPG v| CQH lần lượt cắt BA,
CA tại K, L kh{c B, C. Chứng minh rằng GK v| HL cắt nhau tại một điểm trên AO.
Phân tích tìm lời giải
Gọi X l| giao điểm của GK v| HL. Dễ thấy AG v| AH l| b{n kính đường tròn t}m
A nên ta được
AG AH
, đồng thời G v| H thuộc đường tròn (O) nên ta có
OG OH
từ đó
suy ra AO l| đường trung trực của GH. Ta EF//BC nên ta được
EFB FBC
. Lại
EPQ EFB
do đó ta được
QPE FBC
nên tứ gi{c PQCB nội tiếp. Lại có
0
GHC 180 GBC
GHE GFE
nên
ECH BGF
. Do tứ gi{c GPEH nội tiếp nên
HGP HEC
,
PGK PBK
từ đó
HGX HEC GBF GBA PBF
. Tương tự
GHX GFB HCE HCA QCE
. Như vậy ta cần chỉ ra
HEC GBF GFB ECH
thì ta sẽ
HGX GHX
v| như vậy
HX GX
hay X thuộc đừng trung trực của HG hay X thuộc
OA, điều n|y có nghĩa l| GK v| HL cắt nhau tại X trên AO.
Lời giải
Gọi X l| giao điểm của GK v| HL. Ta cần
chứng minh X thuộc OA.
Thật vậy, dễ thấy AG v| AH l| b{n nh
đường tròn t}m A nên ta được
AG AH
,
đồng thời G v| H thuộc đường tròn (O)
nên ta
OG OH
từ đó suy ra AO l|
đường trung trực của GH. Ta EF//BC
nên ta được
EFB FBC
. M| ta tứ gi{c
PEFQ nội tiếp đường tròn t}m A nên ta
lại
EPQ EFB
. Do đó ta được
QPE FBC
, từ đó suy ra tứ gi{c PQCB
nội tiếp đường tròn.
Do tứ gi{c GBCH nội tiếp nên ta lại
0
GHC 180 GBC
. Lại
GHE GFE
(hai góc nội
tiếp của đường tròn t}m A). Ta có biến đổi góc sau
00
00
ECH GHC GHE 180 GBC GHE 180 GBP PBC GFE
BGP GPB 180 BPC BCP 180 GPE BGP FEC
BGP PGF BGF
Do tứ gi{c GPEH nội tiếp nên
HGP HEC
, m| lại có
PGK PBK GBF GBA PBF
K
L
O
X
Q
P
F
E
H
G
C
B
A
Nguyễn Công Lợi
TÀI LIU TOÁN HC
66
Từ đó ta được
HGX HGP PGK HEC PBK HEC GBF GBA PBF
(1)
Ho|n to|n tương tự ta được
GHX GFB HCE HCA QCE
(2)
Lại có
GBA HCA, PBF QCE
BGF CHE
nên ta được
GBF GFB HEC EHC
Hay ta được
HEC GBF GFB ECH
(3)
Từ (1), (2) v| (3) nên ta được
HGX GHX
nên ta được
HX GX
. Từ đó suy ra X thuộc
đừng trung trực của HG hay X thuộc OA. Vậy GK v| HL cắt nhau tại một điểm trên AO.
Ví dụ 20. Cho một đường tròn với hai d}y AB v| CD không song song. Đường vuông góc
với AB kẻ từ A cắt đường vuông góc với CD kẻ từ C v| từ D lần lượt tại M v| P. Đường
vuông góc với AB ktừ B cắt đường vuông góc với CD kẻ từ C v| từ D lần lượt tại Q v|
N. Chứng minh rằng c{c đường thẳng AD, BC, MN đồng quy v| c{c đường thẳng AC,
BD, PQ đồng quy.
Lời giải
Gọi giao điểm của MN với BC v| AD lần lượt
l| I v| J. Ta có:
MBC
NBC
1
MC.BC. sin MCB
S
MI MC.cos BCD
2
IN S 1
NB.cos ABC
NB.BC. sin NBC
2
Tương tự ta có
MJ AM. cos BAD
JN
ND. cos ADC
Nhưng do
BCD BAD, ABC ADC AMC DNB
nên
AM MC
ND NB
suy ra
MI MJ
IN JN
hay I v| J trùng nhau. Do đó ta được AD, BC, MN đồng quy. Tương tự ta được AC, BD,
PQ đồng quy
dụ 21. Cho tam gi{c ABC. Đường tròn nội tiếp t}m I của tam gi{c tiếp xúc với c{c cạnh
BC, CA, AB lần lượt tại D, E, F. Gọi M l| trung điểm của cạnh BC. Chứng minh rằng c{c
đường thẳng AM, DI, EF đồng quy tại một điểm.
Phân tích và lời giải
Q
P
N
M
I
D
C
B
A
Nguyễn Công Lợi
TÀI LIU TOÁN HC
67
Để chứng minh AM, EF, ID đồng quy ta gọi L, N
lần lượt l| giao điểm của ID với EF v| của AL với
BC v| chứng minh hai điểm M v| N trùng nhau,
tức N l| trung điểm BC. Trong tam gi{c ABC ta
1
.AB.AN.sin BAN
S
sin BAN
NB AB
ABN 2
.
NC S 1 AC
sin CAN
.AC.AN.sin CAN
ACN
2
Trong tam giác AEF ta có
2S 2S S
sin BAN
FL
AFL AEL AFL
:
AF.AL AE.AL S EL
sin CAN
AEL
Trong tam giác IEF ta có
1
.IF.IL.sin FIL
S
sin FIL sin B
FL AC
FIL 2
EL S 1 AB
sin EIL sin C
.IE.IL.sin EIL
EIL
2
Từ đó ta có được
NB
1
NC
nên suy ra N l| trung điểm của BC hay hai điểm M v| N trùng
nhau. Vậy c{c đường thẳng AM, EF, ID đồng quy.
Ví dụ 22. B|i to{n . Cho ADBC l| tứ gi{c nội tiếp được được. Gọi P, Q, R lần lượt l| hình
chiếu của D trên c{c đường thẳng BC, CA v| AB. Chứng minh rằng
PQ QR
khi v| chỉ
khi c{c đường ph}n gi{c của c{c góc
ABC
ADC
cắt nhau tại một điểm nằm trên đường
thẳng AC
Đề thi Olympic Toán Quốc tế - IMO 2003
Lời giải
Trước hết ta chứng minh
AQR CQP
.
Thật vậy, dễ d|ng nhận thấy c{c tứ gi{c AQRC,
CDPQ v| BPDR nội tiếp được. Do đó ta có
AQR ADR
CQP CDP
. M| ta lại có
RDP ADC
. Do đó ta được
. Từ đó ta
được
AQR CQP
+ Điều kiện cần:Khi c{c đường ph}n gi{c của c{c góc
ABC
ADC
cắt nhau tại một điểm nằm trên đường
thẳng AC. Ta cần chứng minh thì
PQ QR
.
Thật vậy, gọi E l| giao điểm trên đường thẳng AC
D
L
F
E
N
M
I
C
B
A
E
M
P
R
Q
D
C
B
A
Nguyễn Công Lợi
TÀI LIU TOÁN HC
68
của hai đường ph}n gi{c của c{c góc
ABC
ADC
.
Trên tia đối của tia DA l}y điểm M sao cho
AM AD
. [p dụng tính chất đường ph}n gi{c
cho hai tam gi{c ABC v| ADC ta được
AB DA EA
BC DC EC

. Từ đó ta được
AB DM
BC DC
.
Mặt kh{c do tứ gi{c ABCD nội tiếp nên ta có
ABC MDC
. Từ đó suy ra
ABC MDC
nên ta được
ABC MCD
CAB CMD
. Lại có tứ gi{c AQDR nội tiếp nên
DQR DAQ
RDQ CMD CAB
. Từ đó
DRQ MAC
suy ra
QR DR DR
AC MA 2AD

Dễ thấy
ADC APR
nên
RP DR
AC DA
. Từ đó ta được
QR PR 1
QR PR PQ QR
AC 2AC 2
+ Điều kiện đủ: Khi
PQ QR
, ta cần chứng minh c{c đường ph}n gi{c của c{c góc
ABC
ADC
cắt nhau tại một điểm nằm trên đường thẳng AC.
Thật vậy, gọi E l| giao điểm của đường ph}n gi{c của góc
ADC
với AC, ta đi chứng minh
DE cũng l| đường ph}n gi{c của góc
ABC
. Theo tính chất đường ph}n gi{c trong tam
giác ta có
AD EA DM
CD EC DC

.
Dễ thấy
ADC APR
nên ta được
RP DR
AC DA
.
PQ QR
nên ta được
2QR 2DR QR DR DR
AC 2AD AC 2AD MA
suy ra
DRQ MAC
Suy ra
RDQ CMD CAB
, lại có
ABC CDM
nên
ABC MDC
Do đó ta được
AB DM
BC DC
. Từ đó suy ra
AB EA
BC EC
. Do đó BE l| ph}n gi{c của góc
ABC
.
dụ 23. Cho tam gi{c nhọn ABC v| lấy c{c điểm P, Q nằm trên cạnh BC sao cho
PAB BCA
. C{c điểm M v| N lần lượt trên AP v| AQ sao cho P l| trung
điểm của AM v| Q l| trung điểm của AN. Chứng minh rằng giao điểm của BM v| CN
nằm trên đường tròn ngoại tiếp tam gi{c ABC.
Trích đề thi Olympic Toán Quốc tế - IMO 2014
Phân tích tìm lời giải
Gọi giao điểm của MB v| CN l| D. Để chứng minh giao điểm của BM v| CN nằm
trên đường tròn ngoại tiếp tam gi{c ABC ta đi chứng minh tứ gi{c ABDC nội tiếp đường
tròn. Muốn vậy ta cần chứng minh được tứ gi{c BQDN nội tiếp đường tròn để
BAC BQN BDN
hoặc
0
BDC A 180
. Với hai kết quả trên ta đều tứ gi{c ABDC nội
tiếp đường tròn.
Nguyễn Công Lợi
TÀI LIU TOÁN HC
69
Lời giải
Cách 1: Gọi giao điểm của MB v| CN l| D. ta cần
chứng minh tứ gi{c ABDC nội tiếp đường tròn. Thật
vậy, theo giả thiết ta có
AP PM
AQ QN
do đó
ta được MN//PQ. Cũng theo giả thiết
PAB BCA
CAQ ABC
ta có
APQ AQP A
Mặt kh{c dễ d|ng chứng minh được
ABP CAQ
Suy ra
PB QA PB QA
PA QC PM QC
, do đó
BPM NQC
nên
CNQ MBP
. Do đó tứ gi{c BQDN nội tiếp
đường tròn, suy ra
BAC BQN BDN
. Vậy tứ gi{c
ABDC nội tiếp đường tròn, ta có điều phải chứng
minh.
Cách 2: Do
CAQ ABC
nên ta được
ABC QAC
, suy ra
AB BC
AQ AC
.
Gọi E v| F lần lượt l| điểm nằm trên tia đối của tia AB v| AC sao cho
AE AB, AF AC.
Do
BC AB 2AB EB
AC AQ 2AQ NA
nên ta được
EBC NAC
, do đó ta được
BEC ANC
Tương tự ta được
.Mặt kh{c ta thấy tứ gi{c BFEC l| hình bình h|nh. Gọi I l|
trung điểm của BC thì ta có AI//CE//FB. Suy ra
BAC BAI IAC BEC CFB
.
Do đó ta được
DNM ANM ANC A BEC
. Tương tự ta được
DMN A CFB
.
Ta có
DNM DMN 2A BEC BFC A
, suy ra
0
BDC A 180
Do đó tứ gi{c ABDC nội tiếp đường tròn.
dụ 24. Cho tam giác ABC
AB AC
v| H l| trực t}m. Ta kí hiệu (O) l| đường trong
ngoại tiếp tam gi{c ABC. Gọi M l| trung điểm của BC. Đường thẳng AM cắt đường trong
(O) tại điểm thứ hai l| N. Đường tròn đường kính AM cắt đường tròn (O) tại P. Chướng
minh rẳng c{c đường thẳng AP, BC, OH đồng quy khi v| chỉ khi
HA HN
Trích đề thi chọn đội tuyển IMO Pháp 2012
Lời giải
Q
P
N
M
D
C
B
A
Nguyễn Công Lợi
TÀI LIU TOÁN HC
70
Trước hết ta sẽ chứng minh ba điểm P, H, M
thẳng h|ng.
Thật vậy, Gọi V l| điểm đối xứng với A qua
O. Khi đó ta có
BH AC
VC AC
. Từ đó
suy ra BH//VC. Ho|n to|n tương tự ta được
CH//VB nên tứ gi{c BHCV l| hình bình hành.
Do M l| trung điểm của BC nên M l| ta có HV
nhận M l| trung điểm.
Do P thuộc đường tròn (O) nên ta được
0
APV 90
Gọi U l| giao điểm thứ hai của HV với đường
tròn (O) khi đó ta có
0
AUV ABV 90
. Suy ra
hai điểm U v| P trùng nhau. Do đó ba điểm P,
M, H thẳng hàng.
Gọi J l| giao điểm của AP v| BC. Do AH v| MP l| hai đường cao của tam gi{c AIM, do đó
H l| trực t}m của tam gi{c AJM. Từ đó suy ra OH đi qua J khi v| chỉ khi OH vuông góc
với AN hay
OA ON
.
Điều n|y chỉ ra rằng
OH AN
khi v| chỉ khi OH l| đường trung trực của AN, có nghĩa l|
HA AN
.
Từ đó ta có điều khải chứng minh.
Ví dụ 25. Cho đường tròn (O) ngoại tiếp tam gi{c ABC. Một đường tròn đi qua A v| C cắt
cạnh BC v| BA lần lượt tại D v| E. C{c đường thẳng AD v| CE cắt đường tròn lần thứ hai
tại G v| H tương ứng. C{c tiếp tuyến với đường tròn (O) tại A v| C cắt DE lần lượt tại L v|
M. Chứng minh rằng c{c đường thẳng LH v| MG cắt nhau tại một điểm trên đường tròn
(O).
Phân tích tìm lời giải
Gọi giao điểm của đường thẳng MG với đường tròn (O) l| P kh{c G. Chú ý rằng
tứ gi{c BPED nội tiếp đường tròn nên P l| giao điểm thứ hai của hai đường tròn ngoại tiếp
tam gi{c ABC v| BDE. Do đó để chứng minh c{c đường thẳng LH v| MG cắt nhau tại một
điểm trên đường tròn (O) ta sẽ chứng minh LH cũng đi qua P. Muốn vậy ta gọi Q l| giao
điểm của đường thẳng LH với đường tròn (O) v| ta cần chứng minh Q l| giao điểm tứ hai
của hai đường tròn ngoại tiếp hai tam gi{c ABC v| BDE.
Lời giải
N
M
P
V
U
J
O
H
C
B
A
Nguyễn Công Lợi
TÀI LIU TOÁN HC
71
Gọi giao điểm của đường thẳng MG với đường tròn (O) l| P kh{c G. Do
MCD CAE
MDC CAE
nên ta được
MCD MDC
, suy ra tam gi{c MCD c}n tại M. Do đó ta được
MC MD
.
Vì MC l| tiếp tuyến tại C của đường tròn (O) nên
ta được
22
MC MD MG.MP
Suy ra MD l| tiếp tuyến của đường tròn ngoại
tiếp tam gi{c DGP. Từ đó ta được
DGP EDP
.
Gọi (O’)l| đường tròn ngoại tiếp tam gi{c BDE.
Khi đó
+ Nếu B v| P trùng nhau thì với
BGD BDE
ta
được hai đường tròn (O) v| (O’) tiếp xúc trong tại
B.
+ Nếu B v| P kh{c nhau, khi đó ta có hai trường
hợp sau
Trường hợp P nằm cùng phía với G so với
đường thẳng BC, khi đó ta được
0
EDP ABP 180
Mà ta có
0
DGP ABP 180
nên
tứ gi{c BPED nội tiếp đường tròn.
Từ đó suy ra P l| giao điểm của hai đường tròn (O) v| (O’)(kh{c D v| kh{c E)
Trường hợp P nằm kh{c phía với G so với đường thẳng BC, khi đó ta có
EDP DPG AGP ABP EBP
Suy ra tứ gi{c PBDE nội tiếp đường tròn nên P cũng l| giao điểm của hai đường tròn (O)
v| (O’)
Ho|n to|n tương tự ta gọi Q l| giao điểm của đường thẳng LH với đường tròn (O). ta
cũng chứng minh được Q l| giao điểm của hai đường tròn (O) v| (O’) ( kh{c D v| kh{c E).
Do đó hai điểm P v| Q trùng nhau,
Vậy hai đường thẳng LH v| MG cắt nhau tại một điểm trên đường tròn (O).
Ví dụ 26. Cho tam giác ABC với I l| t}m nội tiếp v| (O) l| đường tròn ngoại tiếp tam gi{c.
Đường thẳng AI cắt (O) tại điểm thứ hai l| D (kh{c A). Gọi E l| một điểm trên cung
BDC
của đường tròn (O) v| F l| một điểm nằm trên đoạn BC sao cho
1
BAF CAE BAC
2

.
Chứng minh rằng đường thẳng EI v| DG cùng đi qua một điểm trên nằm trên (O).
Trích đề thi Olympic Toán Quốc tế - IMO 2010
Phân tích và lời giải
M
L
E
P
G
H
O
D
C
B
A
Nguyễn Công Lợi
TÀI LIU TOÁN HC
72
Gọi K l| giao điểm của tia AD với đường tròn t}m D,
b{n kính DI. Do D l| giao điểm của ph}n gi{c góc
BAC
với (O) nên D chính l| trung điểm cung
BC
. Dễ
thấy khi đó
DI DB DC
.
Suy ra bốn điểm B, I, C, K cùng thuộc (D) v| D l|
trung điểm IK. Theo giả thiết vì G l| trung điểm IF nên
DG l| đường trung bình của tam gi{c IFK. Nên ta
được GD // FK
IDG IKF
. Ta sẽ chứng minh
AEI AKF
.
Thật vậy, do
1
BAF CAE BAC
2

ABF AEC
nên
suy ra
AB AF
ABF AEC
AE AC
.
Hơn nữa lại có
ABI IBC AKC
IAB IAC
nên
AB AI
ABI AKC
AK AC
.
So s{nh hai đẳng thức ở trên ta được
AE AK
AE.AF AB.AC AI.AK
AI AF
.
Mặt kh{c
IAE IAC CAE IAB BAF IAF
nên
AEI AKF IEA IKF
(2)
Từ (1) v| (2) ta được
IDG IEA ADG AEI
, tức l| c{c góc n|y cùng chắn một cung trên
đường tròn (O) đã cho. Từ đó suy ra c{c đường thẳng EI v| DG cùng đi qua một điểm
nằm trên (O).
Ví dụ 27. Cho tam gi{c ABC với c{c góc nhọn v|
AB AC.
Đường tròn đường kính BC
t}m O cắt AB, AC lần lượt tại M v| N. Ph}n gi{c trong của góc
BAC
MON
gặp nhau tại
R. Chứng minh rằng hai đường tròn ngoại tiếp của hai tam gi{c BMR v| CNR có một điểm
chung nằm trên BC
Trích đề thi Olympic Toán Quốc tế - IMO 2004
Lời giải
Gọi J, K lần lượt l| t}m đường tròn ngoại tiếp c{c tam gi{c BMR, CNR. Với đường tròn
tâm O ta
MNA MBC
(cùng chắn cung MC). Tam gi{c CON c}n tại O nên ta được
ONC OCN
, nên ta được
ONM BAC
. Điều n|y chứng tỏ ON l| tiếp tuyến của đường
tròn t}m I ngoại tiếp tam gi{c AMN. Ho|n to|n tương tự ta được OM l| tiếp tuyến với
đường tròn t}m I ngoại tiếp tam gi{c AMN
K
I
O
G
F
E
D
C
B
A
Nguyễn Công Lợi
TÀI LIU TOÁN HC
73
Từ đó suy ra
OM ON
nên tam giác MON cân. Theo giả thiết OR l| ph}n gi{c
MON
nên
nó l| trung trực của MN. Do đó ta được tam gi{c
MRN
c}n, suy ra MR, NR l| hai đường
ph}n gi{c kh{c của MON.
Điều n|y dẫn đến R l| t}m đường tròn nội
tiếp tam gi{c MON v| R cũng l| điểm chính
giữa cung MN thuộc đường tròn t}m I. Hai
đường tròn t}m J v| K ngoại tiếp với hai
tam gi{c BMR v| CNR điểm chung thứ
hai là P.
Ta AMB l| c{t tuyến chung của hai
đường tròn t}m J v| O, ANC l| c{t tuyến
chung của hai đường tròn t}m K v| O,
nên ARP l| c{t tuyến chung của hai đường
tròn tâm J K. Vậy A, R, P thẳng h|ng. Do
đó c{c tứ gi{c PRMB v| PRNC nội tiếp. Từ
đó ta được
0
1
RPB RMB RPB ABC BAC 180
2
0
1
RPC RNC RPC ACB BAC 180
2
. Từ đó ta được
o
BPC RPB RPC 180
. Do
đó P phải nằm trên BC. Vậy hai đường tròn ngoại tiếp hai tam gi{c BMR v| CNR một
điểm chung trên BC
Ví dụ 28. Gọi I l| t}m đường tròn nội tiếp của tam gi{c ABC v|
l| đường tròn ngoại tiếp
của nó. Đường thẳng AI lại cắt
tại D. Lấy điểm E trên cung
BDC
v| F trên cạnh BC sao
cho
1
BAF CAE BAC
2

. Gọi G l| trung điểm của IF. Chứng minh rằng c{c đường thẳng
DG v| EI cắt nhau tại một điểm trên
Trích đề thi Olympic Toán Quốc tế - IMO 2010
Lời giải
K
J
I
R
M
N
P
O
C
B
A
Nguyễn Công Lợi
TÀI LIU TOÁN HC
74
Gọi giao điểm của EI với đường tròn
P. Ta
thể chứng minh DP đi qua trung điểm G của FI.
Do
IPX IAX
nên tứ gi{c XPAI nội tiếp, suy ra
AXI API ALE AFC
. Từ đó suy ra XI song
song với BC. Ta
DIC DAC ACI DCB BCI DCI
nên ta được
DI DC DB
. Nên D l| t}m đường tròn ngoại
tiếp tam gi{c BIC. Từ đó
DBP DYB
nên suy ra
DB DP
DY DB
Từ đó suy ra
DI DP
DY DI
, nên ta được
DIP DYI
Suy ra
DIY DPI DAF
nên IY song song với XP. Từ c{c điều trên suy ra FXIY l| hình
bình h|nh nên hai đường chéo FI v| XY cắt nhau tại trung điểm G. Vậy DG v| EI cắt nhau
tại một điểm trên
.
Ví dụ 29. Cho lục gi{c ABCDEF
BC EF
v| c{c đỉnh nằm trên đường tròn đường kính
AD. Gọi H l| giao điểm của AC với BD, K l| giao điểm của AE với DE. Gọi P, Q lần lượt l|
hình chiếu vuông góc của H trên AF, DE; R, S lần lượt l| hình chiếu vuông góc của K trên
AB, CD. Chứng minh rằng RS, PQ, HK đồng quy.
Lời giải
Gọi M l| giao điểm của AB v| CD. Vẽ KI vuông góc với AD tại I. Do
BC EF
và AD là
đường kính của đường tròn ngoại tiếp đa gi{c ABCDEF nên được
BHC AKD
. M| ta lại
0
BHC BMC 180
nên suy ra
0
AKD BMC 180
. Do đó tứ gi{c MAKD nội tiếp đường
tròn.
Giả sử
MAK MDK
. Do c{c tứ gi{c
AIKR, DSIK. AMDK nội tiếp đường
tròn nên ta được
0
0
AIR AKM MAK 90
90 MDK DKS DIS
Từ đó suy ra ba điểm R, I, S thẳng
hang.
Gọi
12
K ; K
lần lượt l| điểm đối
xứng với K qua AB, AD. Vì
1
AK M AKM ADM
G
D
P
F
I
E
L
Y
X
C
B
A
Q
P
H
M
J
I
F
E
R
S
K
K
2
K
1
D
C
B
A
Nguyễn Công Lợi
TÀI LIU TOÁN HC
75
0
ADM AHM 180
nên suy ra
0
1
AK M AHM 180
.
Do đó tứ gi{c
1
AHMK
nội tiếp đường tròn. Suy ra
11
AHK AMK AMK
.
Ho|n to|n tương tự ta có tứ gi{c
2
AHK D
nội tiếp nên
22
DHK DAK DAK DMK
Từ đó ta được
0
12
AHK DHK AHD AMK DMK AKD AMD AHD 180
Từ đó suy ra ba điểm
12
K ; H; K
thẳng h|ng. M| RI l| đường trung bình của tam gi{c
12
KK K
nên RS đi qua trung điểm J của HK. Tương tự PQ cũng đi qua trung điểm J. Do vậy RS,
PQ, HK đồng quy.
dụ 30. Cho nửa đường tròn (O; R) đường kính AB v| một điểm M trên nửa đường
tròn(M kh{c A, B). Trên nửa mặt phẳng bờ AB chứa điểm M vẽ c{c tiếp tuyến Ax v| By
với nửa đường tròn. Tiếp tuyến tại M với nửa đường tròn cắt Ax, By lần lượt tại C, D.
Đường tròn ngoại tiếp tam gi{c COD cắt đường tròn (O) tại E v| F. Chứng minh rằng c{c
đường thẳng AD, BC, EF đồng quy.
Lời giải
Dễ thấy AC//BD. Theo tính chất hai tiếp tuyến cắt
nhau ta có
AC CM;BD DM
0
COD 90
. Gọi I
l| trung điểm của CD, N l| giao điểm của AC v|
BC, H l| giao điểm của MN v| AB. Gọi K, S lần
lượt l| giao điểm của EF với OM, OI.
Theo định lí Talets ta có
CN AC CN CM
BM BD NB MD
Trong tam giác CBD có
CN CM
NB MD
nên theo định lí
Talets đảo ta suy ra MN//BD. Do đó ta được
MN//AC//BD. Từ đó ta được
MN DM BN NH
AC DC BC AC
, suy ra
MN NH
Đường tròn ngoại tiếp tam gi{c COD l| đường tròn (I) đường kính CD. Vẽ đường kính OJ
của đường tròn (I). Vì đường tròn (I) cắt đường tròn (O) tại E, F nên OI l| đường trung
trực của EF. Từ đó ta được OI vuông góc với EF nên EF//AB. Tam gi{c EOJ vuông tại E
ES l| đường cao nên
2
OS.OJ OE
. Do đó ta được
22
1
2OS.OI OM OS.OI OM
2
. Dễ thấy
OSK OMI
nên
OS OK
OS.OI OM.OK
OM OI
. Từ đó
2
11
OM.OK OM OK OM
22
hay
F
E
H
O
I
J
K
N
M
D
C
B
A
Nguyễn Công Lợi
TÀI LIU TOÁN HC
76
K l| trung điểm của OM. Do đó NK l| đường trung bình của tam gi{c MHO nên KN//AB.
M| ta lại có EF//AB nên EF đi qua N. Vậy c{c đường thẳng AD, BC, EF đồng quy tại N.
Ví dụ 31. Cho điểm A nằm ngo|i đường tròn(O; R). Vẽ c{c c{t tuyến AEB, ADC với đường
tròn (O)(c{c điểm B, C, D, E thuộc đường tròn). Gọi H l| giao điểm của BD v| CE. Vẽ c{c
tiếp tuyến AM, AN với đường tròn (O)(M, N l| c{c tiếp điểm) trong đó B v| M nằm cung
nửa mặt phẳng bờ AH. Chứng minh rằng c{c đường thẳng AH, BN, CM đồng quy.
Lời giải
Gọi H’ l| giao điểm của giao điểm của MN v| BD,
E’ l| giao điểm của CD’ v| đường trong (O). Ta có
0
AMO ANO 90
nên tứ gi{c AMON nội tiếp
đường tròn đường kính AH. Gọi I l| t}m của
đường tròn đó. Gọi S l| giao điểm của AH’ với
đường tròn (I). Khi đó
H' E.H' C H' M.H' N H' A.H' S
Từ đó suy ra tứ gi{c AE’SC nội tiếp đường tròn, do
đó ta được
AE ' C ASC
. Gọi F l| giao điểm của
AO v| MN. Khi đó dễ d|ng chứng minh được
2
AH '.AS AF.AO AN AC.AD
Từ đó suy ra tứ gi{c H’SCD nội tiếp đường tròn,
do đó ta được
0
H ' SC H ' DC 180
. Mà ta có
BE ' C H ' DC
. Từ đó ta được
0
AE ' C BE' C 180
Từ đó suy ra ba điểm A, E’, B thẳng hang nên hai điểm E v| E’ trùng nhau, hai điểm H v|
H’ trùng nhau.
Như vậy ba điểm M, H, N thẳng hang. Gọi K’ l| giao điểm của AH v| CM.
Xét đường tròn (I) có
ASN MNA
v| trong đường tròn (O) có
MCN MNA
Do đó ta được
ASN MCN
suy ra tứ gi{c K’SCN nội tiếp. Từ đó ta được
0
K' CN K' SN 180
Ta chứng minh được
HB.HD HM.HN HA.HS
nên suy ra tứ gi{c ABSD nội tiếp đường
tròn.
Do đó suy ra
ABD ASD
. Trong đường tròn (O) có
.
Suy ra
ASD HCD
nên tứ gi{c SHDC nội tiếp đường tròn. Do đó
0
HSC HDC 180
.
K'
K
N
M
S
I
O
E'
H'
H
F
E
D
C
B
A
Nguyễn Công Lợi
TÀI LIU TOÁN HC
77
M| ta lại có
. Kết hợp c{c kết quả trên ta được
K' NC BNC
, suy ra hai tia NK’
v| NB trùng nhau hay ba điểm B, K’, N thẳng hang. Từ đó suy ra hai điểm K v| K’ trùng
nhau.
Như vậy ba điểm A, K, S thẳng hang nên suy ra AH, BN, CM đồng quy tại một điểm.
Ví dụ 32. Cho tam gi{c ABC có AH, BM, CD lần lượt l| đường cao, đường trung tuyến,
đường ph}n gi{c của tam gi{c. Biết
sin BAC cosABC.tan ACB
. Chứng minh rằng AH, BM,
CD đồng quy.
Lời giải
Ta có
ABC
1
S AH.BC
2
. Trong tam giác vuông ABH có
AH AB.sin ABC
nên ta được
ABC
1
S AB.BC.sin ABC
2
. Ta lại có
ABC
1
S AB.AC. sin BAC
2
. Do đó
AH.BC
sin BAC
AB.AC
.
Trong tam giác vuông AHB có
BH
cos ABC
AB
và trong tam giác vuông ACH có
AH
tan ACB
CH
.
Từ giả thiết
sin BAC cosABC.tan ACB
ta suy ra được
AH.BC AH BH BC BH
.
AB.AC AB CH AC CH
.
Gọi I l| giao điểm của AH v| BM. Vẽ MK vuông góc
với AH tại K.. Trong tam gi{c AHC có
MA MC
MK//HC nên ta được
AK KH
nên KM l| đường
trung bình của tam gi{c AHC, do đó
HC 2MK
.
Từ đó ta được
BC BH BC BH
2CM 2MK CM MK
.
Cũng do MK//HC nên ta được
BH BI
MK IM
. Do đó ta được
BC BI
CM MI
.
Gọi I’ l| giao điểm của BM v| CD. [p dụng tính chất đường ph}n gi{c cho tam gi{c BCM
ta được
BC BI ' BI BI ' BI BI ' BI BI '
BI BI '
MC MI ' MI MI ' BI MI BI ' MI ' BM BM

Từ đó suy ra hai điểm I v| I’ trùng nhau, do đó AH, BM, CD đồng quy.
dụ 33. Cho tam gi{c nhọn ABC. Đường tròn (O) đi qua hai điểm B, C cắt AB, AC lần
lượt tại F v| E. Gọi H l| giao điểm của BE v| CF. Chứng minh rằng c{c tiếp tuyến tại E, F
với đường tròng (O) v| AH đồng quy tại một điểm.
Lời giải
I'
I
K
H
M
D
C
B
A
Nguyễn Công Lợi
TÀI LIU TOÁN HC
78
Gọi giao điểm của BC v| EF l| P.
Đường thẳng HP cắt đường tròn
(O) tại X, Y(X nằm giữa P v| H).
Gọi giao điểm của đường tròn
ngoại tiếp tam gi{c HEF với OA l|
V. Khi đó ta chứng minh được
PH.PV PE.PF PB.PC
. Từ đó
suy ra tứ gi{c BCVH nội tiếp
đường tròn. Do đó
BVF BCF BEF BOF
nên tứ
gi{c BFOV nội tiếp đương tròn.
Ho|n to|n tương tự ta được tứ
gi{c VECO nội tiếp đường tròn.
M| hai đường tròn giao nhau tại
O v| V nên BF v| CE cắt nhau tại
một điểm trên đường thẳng VO.
M| ta lại có
AF.AB AE.AC
.
Do đó ba điểm O, V, A thẳng h|ng. Từ đó
nên
0
OVX 90
hay PH vuông góc
với OA.
Chứng minh tương tự ta được AH vuông góc với OP. Gọi S l| giao điểm của AH v| OP.
Khi đó ta được
OVF OFA
nên ta có
2
OV OF
OF OV.OA
OF OA
Từ đó ta được
2
OX OV.OA
, suy ra
0
OXA 90
. Do đó AX, AY l| tiếp tuyến của đường
tròn (O).
Từ đó ta được
PD.PJ PS.PO PH.PV PE.PE
suy ra tứ gi{c EFDJ nội tiếp đường tròn.
Do đó ta thu được
PB.PC PE.PF PD.PJ
Qua D kẻ đường thẳng song song với EF cắt AB, AC lần lượt tại R, Q.
Từ đó suy ra
DBR DQC
nên ta được
DB DR
DB.DC DQ.DR
DQ DC
. Do đó ta được
22
2 2 2 2 2 2
DQ.DR DB.DC BJ JD CJ JD BJ JD BJ JD BJ DJ
PJ DJ PJ BJ PD PJ JD PJ CJ
PD PJ JD PB.PC PD.PJ PD.JD PB.PC PD.DJ
Từ đó suy ra
DPR DQJ
nên ta được
DPR DQJ
, suy ra tứ gi{c RPQJ nội tiếp đường
tròn.
Vậy đường tròn ngoại tiếp tam gi{c PQR đi qua trung điểm J của BC.
Q
O
H
P
S
R
J
T
Y
X
E
V
C
B
D
A
F
Nguyễn Công Lợi
TÀI LIU TOÁN HC
79
Ta có
BEC BFC
nên ta được
, suy ra
HSB HEA
Từ đó ta chứng minh được
HC.HF HE.HB HA.HS
, suy ra tứ gi{c AFSC nội tiếp đường
tròn.
Gọi T l| giao điểm của hai tiếp tuyến tại E, F với đường tròn (O). Khi đó ta có
0
00
ESF ETF CSF CSE ETF 180 BAC CHE ETF
1 1 1
180 sdBC sdEF sdBF sdCE sdBF sdBC sdCE sdEF 180
2 2 2
Từ đó suy ra tứ gi{c SETF nội tiếp đường tròn, suy ra
FST FET
. M| ta lại có
FSA FCA FET
, do đó suy ra
FST FSA
. Suy ra hai tia SA, ST trùng nhau hay bốn
điểm S, T, A, H thẳng h|ng.
Vậy tiếp tuyến tại E, F với đường tròn (O) v| AH đồng quy tại một điểm.
dụ 34. Cho tứ gi{c ABCD không phải l| hình thang nội tiếp đường tròn (O). Gọi giao
điểm của AB v| CD l| E, giao điểm của AD v| BC l| F, giao điểm của AC v| BD l| H.
Đường tròn (O) cắt đường thẳng EH tại M v| N. Gọi I l| trung điểm của MN. Chứng minh
rằng hai tiếp tuyến tại M, N với đường tròn (O) v| đường thẳng OI đồng quy tại một
điểm.
Lời giải
Gọi
1
I
l| giao điểm của đường tròn
ngoại tiếp tam gi{c HAB v| MN. Ta sẽ
chứng minh ba điểm O,
1
I
v| F thẳng
h|ng, từ đó suy ra được hai điểm I v|
1
I
trùng nhau.
Thật vậy, do tứ gi{c
1
ABHI
nội tiếp
đường tròn nên ta được
1
BI H BAH
. Lại
nên ta được
1
BI H BDE
,
từ đó suy ra tứ gi{c
1
I BED
nội tiếp
đường tròn. Từ đó ta được
11
I BH I EC
.
Mà ta có
11
I AC I BH
nên ta được
11
I AC I EC
.
Điều n|y dẫn đến tứ gi{c
1
I AEC
nội tiếp đường tròn. Ta có
1
HAB HI C
nên ta được
1
HDC HI C
, do đó tứ gi{c
1
I DCH
nội tiếp đường tròn. Sử dụng tính chất góc
F
A
D
B
C
I
E
M
N
H
O
Nguyễn Công Lợi
TÀI LIU TOÁN HC
80
ngo|i của một tứ gi{c nội tiếp ta được
11
AI N ABH; NI D HCD
. M| ta lại có
ABH HCD
nên ta được
1
AI D 2ABD
.
Mặt kh{c ta có
AOD 2ABD
, do đó suy ra
1
AI D AOD
nên tứ gi{c
1
I ODA
nội tiếp đường
tròn.
Chứng minh ho|n to|n tương tự ta được
1
I OCB
nội tiếp đường tròn.
Khi đó
1
I
v| O l| hai giao điểm của hai đường trong ngoại tiếp c{c tứ gi{c
1
I ODA
1
I ODA
.
M| ta lại có
FA.FD FB.FC
nên suy ra F thuộc đường thẳng
1
OI
hay ba điểm O,
1
I
và F
thẳng hang.
Ta có
1
11
1
1
AI F ODA
AI F OI D
OAD ODA
OI D ODF
OAD OI D




Mà ta có
11
AI N NI D
nên kết hợp với kết quả trên ta suy ra được MN vuông góc với
1
OI
.
M| ta lại có MN vuông góc với OI. Từ đó suy ra được hai điểm I v|
1
I
trùng nhau hay suy
ra OI đi qua F.
Cũng từ kết quả trên ta được
OID ODF
nên ta được
2
OI.OF OD
.
Do đó suy ra
2
OI.OF ON
nên hai tam gi{c OIN v| ONF đồng dạng với nhau.
Từ đó suy ra
0
ONF OIN 90
nen NF l| tiếp tuyến của đường tròn (O) tại N. Chứng minh
tương tự ta cũng được MF l| tiếp tuyến của đường trong (O) tại M.
Vậy hai tiếp tuyến tại M, N với đường tròn (O) v| đường thẳng OI đồng quy tại một điểm.
dụ 35. Cho tam gi{c ABC không c}n. Đường tròn nội tiếp (I) của tam gi{c tiếp xúc với
BC, CA, AB lần lượt tại
0 0 0
A ;B ;C
.
1 1 1
IA ; IB ; IC
theo thứ tự l| ph}n gi{c cảu tam gi{c IBC,
ICA, IAB. C{c đường ph}n gi{c
0 2 0 2 0 2
A A ; B B ;C C
của tam gi{c
0 0 0
A B C
đồng quy tại
0
I
.
Chứng minh rằng c{c đường thẳng
1 2 1 2 1 2
A A ; B B ;C C
đồng quy tại một điểm trên
0
II
.
Lời giải
Nguyễn Công Lợi
TÀI LIU TOÁN HC
81
Không mất tính tổng qu{t ta có thể giả sử
AB BC CA
. Gọi
a b c
K ; K ; K
lần lượt l|
giao điểm của c{c đường thẳng
1 2 1 2 1 2
A A ; B B ;C C
với
0
II
. Gọi
3 3 3
A ;B ;C
theo thứ tự l| hình chiếu của
0
I
trên
0 0 0 0 0 0
B C ;C A ; A B
tương ứng. Gọi
0
r; r
lần
lượt l| b{n kính đường tròn nội tiếp tam
giác ABC và
0 0 0
A B C
. Chú ý l| c{c tứ gi{c
0 0 0 0 0 0
AB IC ; BC IA ;CA IB
nội tiếp đường
tròn nên ta được
0
0 0 0 0 0 0 0 0
111
B A C ABC ACB ; A B C BAC ACB ; A C B BAC ABC
222
Theo tính chất góc ngo|i của tam gi{c v| chú ý đến tam gi{c
00
CA B
c}n tại C.
Lại có
0
BAC ABC ACB 180
nên ta thu được
0
1 0 1 1
0
2 0 0 0 0 2 0
1 1 ABC ACB
IA A IBA BIA ABC 180
2 2 2
ABC ACB 1
180 ACB A A B B A C A A C
42




0 2 3 2 0 0 2 0 0
0
10
11
I A A A C A A A C BAC ABC ABC ACB
24
11
180 ACB ABC ACB IA A
24
Từ c{c kết quả trên ta thu được
1
IA
song song với
02
IA
1 0 0 2 3
IA A I A A
Từ đó
10
0 2 0 3 0
IA IA
r
I A I A r

. Chú ý là
12
A ; A
thuộc nửa mặt phẳng có bờ
0
II
nên ta có
a
a 0 0
KI
r
K I r
.
Ho|n to|n tườn tự ta cũng có
b
b 0 0
KI
r
K I r
c
c 0 0
KI
r
K I r
. Từ đó suy ra ba điểm
a b c
K ; K ; K
trùng nhau hay c{c đường thẳng
1 2 1 2 1 2
A A ; B B ;C C
đồng quy tại một điểm trên
0
II
.
A
3
I
C
2
B
2
A
2
K
a
I
0
A
1
C
0
B
0
A
0
C
B
A
| 1/80

Preview text:

2
CÁC BÀI TOÁN CHỨNG MINH
BA ĐIỂM THẲNG HÀNG - BA ĐƯỜNG THẲNG ĐỒNG QUY
A. CÁC BÀI TOÁN VỀ BA ĐIỂM THẲNG HÀNG
I. Một số phương pháp chứng minh ba điểm thẳng hàng
Phương pháp 1: Sử dụng góc bù nhau Nếu có 0
ABx xBC  180 thì 3 điểm A, B, C thẳng hàng theo thứ tự đó.
Phương pháp 2: Sử dụng tiên đề về đường thẳng song song
Tiên đề Ơclít: Qua một điểm ở ngoài một đường thẳng chỉ kẻ được duy nhất một đường thẳng
song song với đường thẳng đã cho. Do đó, nếu qua điểm A ta kẻ được AB và AC cùng song song
với một đường thẳng d nào đó thì A, B, C thẳng hàng.
Để chứng minh ba điểm A, B, C thẳng hàng ta chứng minh AB và AC cùng song song với
một đườngthẳng d.
Phương pháp 3: Sử dụng tiên đề về đường thẳng vuông góc
Để chứng minh ba điểm A, B, C thẳng hàng ta đi chứng minh AB và AC cùng vuông góc với
một đường thẳng d.
Phương pháp 4: Sử dụng 2 tia trùng nhau hoặc đối nhau
Nếu hai tia MA, MB trùng nhau hoặc đối nhau thì 3 điểm M, A, B thẳng hàng.
Phương pháp 5: Thêm điểm
Để chứng minh 3 điểm A, B, C thẳng hàng có thể xác định thêm điểm D khác A, B, C sau đó
chứng minh hai trong ba bộ ba điểm A, B, D; A, C, D; B, C, D thẳng hàng.
Phương pháp 6: Phương pháp sử dụng hình đuy nhất
Để chứng minh ba điểm A, B, C thẳng hàng với C thuộc hình H nào đó. Ta gọi C’ là giao điểm
của AB với hình H và tìm cánh chứng minh hai điểm C và C’ trùng nhau.
Phương pháp 7: Sử dụng định lý Menelaus
Cho tam giác ABC. Các điểm A’, B’, C’ lần lượt nằm trên các đường thẳng BC, CA, AB sao
cho trong chúng hoặc không có điểm nào, hoặc có đúng 2 điểm thuộc các cạnh của tam giác ABC.
A' B B 'C C ' A
Khi đó A’, B’, C’ thẳng hàng khi và chỉ khi . .  1
A'C B ' A C ' B Chứng minh Nguyễn Công Lợi TÀI LIỆU TOÁN HỌC 3
+ Trường hợp 1: Trong 3 điểm A’, B’, C’ có đúng 2 điểm thuộc cạnh tam gi{c ABC. Giả sử l| B’, C’
- Điều kiện cần: Qua A kẻ đường thẳng song song với BC cắt đường thẳng B’C’ tại M. C'A AM B ' C A ' C A'B B ' C C ' A AM A ' C A ' B Ta có  ;  . Vậy . .  . .  1 C ' B A ' B B ' A AM A ' C B ' A C ' B A ' B AM A ' C
- Điều kiện đủ: Gọi A’’ l| giao của B’C’ với BC. A''B B ' C C ' A A'B B ' C C ' A
[p dụng định lý Menelaus (phần thuận) ta có . .  1 mà . .  1 A '' C B ' A C ' B A ' C B ' A C ' B A' B A ' B nên 
. Do B’, C’ lần lượt thuộc cạnh CA, AB nên A’’ nằm ngo|i cạnh BC. A ' C A ' C A' B A ' B Vậy 
v| A’, A’’ nằm ngo|i cạnh BC suy ra A '  A ' . Do đó A’, B’, C’ thẳng A ' C A ' C hàng
+ Trường hợp 2: Trong 3 điểm A’, B’, C’ không có điểm thuộc cạnh tam gi{c ABC được
chứng minh tương tự.
II. Một số ví dụ minh họa
Ví dụ 1. Cho hình thang ABCD có AB//CD. Gọi O l| giao điểm của hai đường chéo AC v|
BD. Gọi M, N, P lần lượt l| trung điểm của AB, BC, AD. Gọi E l| trung điểm của PN.
Chứng minh rằng ba điểm M, O, E thẳng h|ng.
Phân tích tìm lời giải
Trên cơ sở hình vẽ v| c{c yếu tố trung điểm ta nhận thấy nếu gọi K l| trung điểm
của CD thì tứ gi{c MNKP l| hình bình h|nh, khi đó ba điểm M, O, E thẳng h|ng. Để có
được M, O, E ta cần chỉ ta được M, K, O thẳng h|ng. Do O l| giao điểm của hai đường
chéo nên ta thấy có c{c tam gi{c đồng dạng. Do đó rất tự nhiên ta nghĩ đến chứng minh 0 KOM  180 . Lời giải
Gọi K l| trung điểm của CD. Khi đó trong tam gi{c ABD có M v| P l| trung điểm của AB 1
v| AD nên PM l| đường trung bình, do đó PM//BD và PM  BD . 2
Từ đó suy ra tứ gi{c MNKP l| hình bình A M B
h|nh, do đó hai đường chéo NP v| MK cắt O
nhau tại E hay ba điểm M, K, E thẳng h|ng . P N E
Dễ thấy hai tam gi{c OAB v| OCD đồng OA AB dạng nên ta được  . M| lại có OC CD D K C Nguyễn Công Lợi TÀI LIỆU TOÁN HỌC 4 1 1
AM  AB,CK  CD nên ta được 2 2 OA AM  . OC CK OA AM
Xét hai tam giác OAM và OCK có OAM  OCK và  nên ta được O  AM ∽ O  CK. OC CK
Từ đó suy ra AOM  COK . Mà ta có 0
AOM  MOC  AOC  180 nên ta được 0
MOK  COK  MOC  AOM  MOC  180
Do đó ba điểm M, O, K thẳng h|ng. Từ đó dẫn đến ba điểm M, O, E thẳng h|ng.
Ví dụ 2. Cho tam gi{c ABC nội tiếp đường tròn (O). Gọi M l| một điểm tuỳ ý thuộc đường
tròn (O). Gọi A ; B ;C theo thứ tự l| hình chiếu của M trên BC, CA, AB. Chứng minh ba 1 1 1
điểm A ; B ;C thẳng h|ng. 1 1 1
Phân tích tìm lời giải
Trên cơ sở hình vẽ v| giả thiết của b|i to{n ta nhận thấy c{c tứ gi{c nội tiếp. Điều
n|y cho ta c{c góc nội tiếp bằng nhau. Do đó từ yêu cầu chứng minh ba điểm A ; B ;C 1 1 1
thẳng h|ng ta nghĩ đến chứng minh 0
C A B  BA B  180 . Muốn vậy ta cần chỉ ra được 1 1 1 1 C A B  B A C . 1 1 1 1 Lời giải
Không mất tính tổng qu{t giả sử điểm M thuộc cung A nhỏ BC . Ta có 0
BC M  BA M  90 nên tứ gi{c MA C B nội tiếp. 1 1 1 1
Do đó ta được BA C  BMC . Lại có 0 MA C  MB C  90 O B1 1 1 1 1 1 A1 B C
nên tứ gi{c MA CB nội tiếp. Do đó ta được 1 1 C1 CA B  CMB M 1 1 1 Mặt kh{c ta lại có 0
BAC  BMC  BAC  B MC  180 nên 1 1 BMC  B MC 1 1
Từ đó ta được B MC  C MB . Kết hợp c{c kết quả trên ta được C A B  B A C 1 1 1 1 1 1 Từ đó suy ta 0
C A B  BA B  B A C  BA B  180 nên ba điểm A ; B ;C thẳng h|ng 1 1 1 1 1 1 1 1 1 1 1
Nhận xét: Đường thẳng chứa ba điểm A ; B ;C gọi là đường thẳng Simsơn của tam giác ABC ứng 1 1 1
với điểm M. Nếu M trùng với đỉnh của tam giác ABC thì đường thẳng Simsơn chính là đường cao tương ứng. Nguyễn Công Lợi TÀI LIỆU TOÁN HỌC 5
Ví dụ 3. Cho tam gi{c ABC nhọn nội tiếp đường tròn (O). Điểm M bất kỳ trên cung nhỏ
BC. Gọi E, F thứ tự l| c{c điểm đối xứng của M qua AB, AC. Gọi H l| trực t}m trực t}m
ABC. Chứng minh rằng E, H, F thẳng h|ng.
Phân tích tìm lời giải
Trên cơ sở hình vẽ, tính tính đối xứng v| c{c tứ gi{c nội tiếp ta suy ra được c{c cặp
góc bằng nhau như BHA '  BEA , EHB  EAB  MAB hay A ' HC  ABC và CHF  MAC .
Do đó để chứng minh ba điểm E, H, F thẳng h|ng ta đi chứng minh 0
EHB  BHA '  A ' HC  CHF  180 . Lời giải
Gọi B’ l| giao điểm của BH v| AC, A’ l| A
giao điểm của AH v| BC. Khi đó tứ gi{c B' HA’CB’ nội tiếp nên F C'
BHA '  A ' CB'  BCA  AMB  BEA . H O E
Từ đó ta được tứ gi{c AHBE nội tiếp nên B A' C
suy ra EHB  EAB  MAB . Hoàn toàn M
tương tự ta có A ' HC  ABC và CHF  MAC . Từ đó ta được 0
EHB  BHA '  A ' HC  CHF  MAB  ACB  ABC  MAC  ABC  BAC  ACB  180 Suy ra 0
EHF  180 nên ba điểm E, H, F thẳng h|ng.
Nhận xét: Đường thẳng đi qua 3 điểm E, H, F nói trên có tên là đường thẳng Steiner ứng với điểm M.
Ví dụ 4. Cho tứ gi{c ABCD nội tiếp đường tròn (O; R). C{c tia AB, DC cắt nhau tại M, c{c
tia AD, BC cắt nha tại N. Đường tròn ngoại tiếp tam gi{c MBC cắt MN tại K kh{c M. Gọi T
l| giao điểm của AC v| BD. Chứng minh rằng ba điểm O, T, K thẳng h|ng.
Phân tích tìm lời giải
Quan s{t hình vẽ ta nhận thấy OK v| TK cùng vuông góc với MN. Do đó ta hướng
đến sử dụng quan hệ vuông góc để chứng minh ba điểm thẳng h|ng. Ta gọi S l| giao điểm
của đường tròn ngoại tiếp tam gi{c ACM với MT. C{c tứ gi{c AMCS v| ABTS nội tiếp nên 2 2 MT.TS  R  OT và 2 2 MT.MS  OM  R . Từ đó 2 2 2 2
MT  OM  OT  2R . Ho|n to|n tương tự ta cũng được 2 2 2 2 NT  ON  OT  2R . Do đó suy ra 2 2 2 2
MT  NT  OM  ON nên OT  MN . Như vậy b|i to{n sẽ được chứng
minh nếu ta chỉ ra được OK  MN .Muốn vậy ta cần chỉ ra được 0 OKM  90 . Lời giải Nguyễn Công Lợi TÀI LIỆU TOÁN HỌC 6
Gọi S l| giao điểm của đường tròn ngoại N
tiếp tam gi{c ACM với MT. Khi đó tứ gi{c
AMCS nội tiếp đường tròn nên dễ d|ng suy D ra được 2 2
MT.TS  AT.TC  R  OT Và K C
MSA  MCA , MCA  MBD nên ta được S T
MBD  MSA . Do đó tứ gi{c ABTS nội tiếp O đường tròn, do đó ta được M B 2 2
MT.MS  OM  R . Từ đó ta được A 2 2 2
MT.MS  MT.TS  OM  OT  2R Suy ra 2 2 2 2 MT  OM  OT  2R . Tương tự ta cũng được 2 2 2 2 NT  ON  OT  2R Do đó ta được 2 2 2 2
MT  NT  OM  ON . Từ đó ta được OT  MN .
Mặt kh{c ta lại có MBC  ADC và CKN  MBC nên ta được ADC  CKN
Từ đó suy ra tứ gi{c DCKN nội tiếp đường tròn, do đó DKN  DCN M| ta lại có
DCN  MAD nên ta được DKN  MAD , suy ra tứ gi{c AMKD nội tiếp đường tròn. Nên ta
được AKM  ADM  CKN . Do đó 0
AOC  AKC  2ADM  AKC  AKM  CKN  AKC  180 . Suy ra tứ gi{c AOCK nội
tiếp đường tròn. M| ta có OA  OC nên OA  OC , suy ra AKO  OKC Do đó 0
OKM  AKO  AKM  90 hay OK  MN . Như vậy ta có OT  MN và OK  MN nên
OT v| OK trùng nhau. Vậy ba điểm O, T, K thẳng h|ng.
Ví dụ 5. Cho hình vuông ABCD có hai đường chéo AC v| BD cắt nhau tại O. Trên cạnh 1
AB lấy điểm M sao cho AM 
AB . Đường thẳng qua D v| vuông góc với đường thẳng 3
MO cắt AC tại E. Gọi F l| giao điểm của MO v| CD. Chứng minh rằng ba điểm B, E, F thẳng h|ng.
Phân tích tìm lời giải
Lấy K l| trung điểm của DF khi đó ta nhận thấy OK song song với BF. Để chứng
minh ba điểm B, E, F thẳng h|ng ta cần chỉ ra được EF vuông góc với OK. Muốn vậy ta
cần chứng minh EF l| đường trung bình của tam gi{c COK hay đi chứng minh E l| trung điểm của OC. Lời giải Nguyễn Công Lợi TÀI LIỆU TOÁN HỌC 7
Gọi H l| giao điểm của MO v| DE, khi đó ta được A M B
HO  DE tại H, do đó tam gi{c OHE vuông tại H. Từ đó ta được 0
HOE  OEH  90 , mà ta có 0 MOA  BOM  90 và
HOE  MOA nên ta suy ra được OEH  BOM. O 1 1 E
Ta lại có MBO  ABC; DAE  DAB . 2 2 H
Xét hai tam giác MBO và DAE có MBO  DAE và D K F C BOM  AED nên M  BO ∽ D  AE BO MB 1 2 Do đó ta được 
. Ta có AM  MB  AB và AM 
AB nên ta được MB  AB , AE AD 3 3 2 MB 2 suy ra MB  AD . Do đó ta được
 . Mà ta có AE  AO  OE và OA  OB nên ta 3 AD 3 2 1 1
được BO  OB  OE  OB  2OE . Do đó OE  OB  OC , nên E l| trung điểm của 3 2 2 OC
Xét hai tam giác COF và AOM có FOC  MOA , OA  OC và OCF  OAM 1 1 2 Do đó ta được C  OF  A
 OM nên CF  AM . Mà AM  AB nên CF  CD  FD  CD . 3 3 3 1 1
Gọi K l| trung điểm của FD, khi đó ta được FK  KD  FD  CD 2 3
Trong tam gi{c BDF có O l| trung điểm của BD v| K l| trung điểm của FD nên OK l|
đường trung bình của tam gi{c DBF. Do đó OK//BF. Chứng minh ho|n to|n tương tự ta được EF//OK
Do đó theo tiên đề Ơclit thì BF v| EF trùng nhau hay ba điểm B, E, F thẳng hàng.
Ví dụ 6. Cho hình vuông ABCD . Trên tia đối của tia CB lấy điểm E, trên tia đối của tia DA
lấy điểm F sao cho AF  BE . Vẽ EH vuông góc với BF lại H. Trên tia đối của tia EH lấy
điểm K sao cho EK  BF . Chứng minh rẳng ba điểm A, C, K thẳng h|ng. Lời giải
Kẻ KM vuông góc với AB tại M. Gọi N l| giao điểm của EF với KM. Trong tứ gi{c ABEF có
BE//AF và BE  AF nên tứ gi{c ABEF l| hình bình h|nh. Lại có 0 ABF  90 nên ABEF là
hình chữ nhật. Từ đó ta được 0
BEN  90 . Tứ gi{c BENM có 0
BMN  MBE  BEN  90 nên
tứ gi{c BENM l| hình chữ nhật. Nguyễn Công Lợi TÀI LIỆU TOÁN HỌC 8 Từ đó 0 MNE  90 nên 0 ENK  90 . M N K
Xét hai tam giác vuông EBF và NEK có BF  EK
và EBF  NEK . Do đó ta được E  BF  N  EK,
suy ra BE  EN, EF  NK Hình chữ nhật BENM C B E
có BE  EN nên tứ gi{c BENM l| hình vuông. Do đó suy ra BM  MN . H
Mặt kh{c AB  NK  EF . Nên ta được A D F
MA  MB  AB  MN  NK  MK .
Tam gi{c AMK vuông tại M có MA  MK nên nó
là tam giác vuông cân. Suy ra 0 MAK  45 Mặt kh{c 0
BAC  45 . Như vậy hai tia AK v| AC trùng nhau hay ba điểm A, C, K thẳng hàng.
Ví dụ 7. Cho tam giác ABC có AB  AC  BC . Gọi AD, BE, CF l| c{c đường ph}n gi{c
trong của tam gi{c ABC. Gọi G, I, K, H lần lượt l| điểm đối xứng của B, A, C, A qua AD, BI GB
BE, AD, CF. Lấy điểm M trên đoạn CK sao cho 
. Chứng minh rằng ba điểm G, I, CI CM M thẳng h|ng.
Phân tích tìm lời giải BI GB BG BI Từ c{c giả thiết v|  ta suy ra được  nên B  GI ∽ C  KH . Từ đó CI CM CK CH
ta được GI//HK. Như vậy để chứng minh ba điểm G, I, M thẳng h|ng ta cần chỉ ra được CM CI
MI//KH. Muốn có được điều đó ta đi chứng minh  . CK CH Lời giải
Ta có AD, BE, CF l| c{c đường ph}n gi{c A
trong của tam gi{c ABC. Gọi G, I, K, H lần
lượt l| điểm đối xứng của B, A, C, A qua E F
AD, BE, AD, CF. Khi đó ta được G
AG  AB,G  AC; AB  BI, I  BC B
AK  AC, K  AB;CH  AC, H  BC H D I C M AB AG Trong tam giác ACK có  nên suy AK AC K BG AG
ra được BG//CK. Do đó ta được  và CK AC GBC  KCN . Nguyễn Công Lợi TÀI LIỆU TOÁN HỌC 9 BG BI
Do AG  AB  BI, AC  CH nên  CK CH BG BI
Xét hai tam giác BGI và CKH có  và GBI  KCH nên B  GI ∽ C  KH CK CH BI GB BG BI
Từ đó ta được BIG  CHK nên suy ra GI//HK. Do  và  nên ta được CI CM CK CH CM BG  BG CK CM CK CM CI và  . Điều n|y dẫn đến    . Trong tam giác CHK có CI BI BI CH CI CH CK CH CM CI 
nên ta được MI//HK. Từ đó ta có GI//KH v| MI//HK nên hai đường thẳng GI v| CK CH
MI trùng nhau. Do đó ba điểm G, I, M thẳng h|ng.
Ví dụ 8. Cho tứ gi{c ABCD. C{c đường thẳng AB, CD cắt nhau tại M v| c{c đường thẳng
AD, BC cắt nhau tại N. Gọi I, J, K lần lượt l| trung điểm của AC, BD, MN. Chứng minh
rằng ba điểm I, J, K thẳng h|ng.
Phân tích tìm lời giải
Trên cở sở giả thiết v| hình vẽ của b|i to{n ta nhận thấy nếu lấy A , B ,C lần lượt 1 1 1
l| trung điểm của NB, NA, AB thì xuất hiện c{c bộ ba điểm thẳng h|ng nên ta nghĩ đến
định lí Menelaus. Do đó ý tưởng đầu tiên để chứng minh ba điểm I, J, K thẳng h|ng đó l| IC KB JA đi chứng minh 1 1 1 . .
 1 . Ngo|i ra ta lại thấy nếu gọi K’ l| giao điểm của IJ v| MN IB KA JC 1 1 1 m| ta chứng minh được S  S
thì suy ra được hai điểm K v| K’ trùng nhau. NIJ MIJ Lời giải
Cách 1: Gọi A , B ,C lần lượt l| trung 1 1 1 M
điểm của NB, NA, AB. Ta có A K là 1 B
đường trung bình của tam gi{c NBM nên C K
ta được A K // BM. Ta có B K l| đường 1 1 C1 I J A1
trung bình của tam gi{c NAM nên ta được
B K // BM. Theo tiên đề Ơclit ta được hai 1 B N A D 1
đường thẳng A K và B K trùng nhau hay 1 1
ba điểm A , B , K thẳng h|ng. Như vậy 1 1 K  A B .. 1 1
Chứng minh ho|n to|n tương tự ta được J  A C và I  B C 1 1 1 1 1 1 1 1 1 1
Ta có IC  BC; IB  CN; KB  AM; KA  BM; JA  DN; JC  AD 1 1 1 1 1 1 2 2 2 2 2 2
Xét tam gi{c NAB với M thuộc AB, C thuộc B, D thuộc NA v| ba điểm M, C, D thẳng hàng. Nguyễn Công Lợi TÀI LIỆU TOÁN HỌC 10 AM BC DN BC AM DN
[p dụng định lí Menelaus ta có . .  1Suy ra . .  1 hay MB CN AD CN BM DA IC KB JA 1 1 1 . .  1 IB KA JC 1 1 1 IC KB JA
Trong tam giác A B C có K  A B , J  A C , I  B C và 1 1 1 . .  1 . 1 1 1 1 1 1 1 1 1 IB KA JC 1 1 1
Như vậy theo định lí Menelaus thì ba điểm I, J, K thẳng h|ng. 1 1 1 1 Cách 2: Ta có S  S ; S  S ; S  S ; S  S NAI MAC NBJ NBD BIJ BDI ABI ABC 2 2 2 2 V| lại có S  S  S ; S  S  S ; S  S  S . Do đó ta được MAC NCD ACD NBD NCD BCD BDI ABD ABID 1 1 1 1 S  S  S  S  S  S  S  S  S  S  S NIJ NAB NAI NBJ BIJ ABI NAB MAC NBD BDI ABC 2 2 2 2 1 1 1 1  S  S  S  S  S  S  S  S NAB
 NCD ACD  NCD BCD  ABD ABID ABC 2 2 2 2   1 1 1 S  S  S  S  S  S  S NAB NCD   ACD ADC  BCD ABD ABID 2 2 2 1 1 1 1  S  S  S  S  S ABCD ABCD ABCD ABCD ABCD 2 2 4 4 1 1
Chứng minh tương tự ta cũng được S  S . Từ đó suy ra S  S  S . MIJ ABCD 4 NIJ MIJ ABCD 4
Gọi K’ l| giao điểm của IJ v| MN. Gọi khoảng c{ch tứ M, N đến IJ lần lượt l| h , h . Khi đó 1 2 1 1 1 1 ta được S  S
 h .IJ  h .IJ  h  h . Từ đó h .JK '  h .JK ' hay S  S NIJ MIJ 1 2 1 2 2 2 1 2 2 2 JMK ' JNK '
nên ta được K' M  K' N hay K’ l| trung điểm của MN. Do đó hai điểm K v| K’ trùng
nhau. Vậy ba điểm I, J, K thẳng h|ng.
Ví dụ 8. Cho hình thang ABCD có AB//CD. Trên c{c cạnh AD v| BC lấy lần lượt c{c điểm AM CN M, N sao cho 
. Đường thẳng qua M song song với AC cắt BD tại P v| cắt CD tại AD BC
K. Gọi I l| trung điểm của MN, O l| giao điểm của AC v| BD. Chứng minh rằng ba điểm O, I, K thẳng h|ng.
Phân tích tìm lời giải
Gọi Q l| giao điểm của KN v| AC, S l| giao điểm của OK v| PQ. Dễ thấy tứ gi{c
KPOQ là hình bình h|nh nên S l| trung điểm của PQ. Như vậy để chứng minh ba điểm O,
I, K thẳng h|ng ta cần chứng minh được MN song song với PQ. Lời giải Nguyễn Công Lợi TÀI LIỆU TOÁN HỌC 11
Gọi Q l| giao điểm của KN v| AC, S l| A B
giao điểm của OK v| PQ. Trong tam gi{c O
ACD có MK//AC nên theo định lí Talets ta M AM CK I P có  . M| theo giả thiết ta có N DM CD S S' Q AM CN  CK CN nên ta được  D K C DM BC CD BC CK CN Trong tam giác BCD có  nên theo CD BC
định lí Talets đảo ta được KN//BD.
Do đó tứ gi{c POQK l| hình bình h|nh, suy ra S l| trung điểm của PQ. Trong tam gi{c MP DP PK DP DAO có MP//OA nên 
và trong tam giác DOC có PK//OC nên  . Do đó OA OD OC OD MP PK MP NQ   
nên suy ra PQ//MN. Gọi S’ l| giao điểm của KI v| MN. Chứng OA OC PK QK PS ' S ' Q KS ' minh tương tự ta được   MI IN KI
Mà ta có IN  IM nên suy ra PS'  QS' . Điều n|y dẫn đến hai điểm S v| S’ trùng nhau, do
đó ba điểm K, I, S thẳng h|ng. M| ba điểm K, S, O thẳng h|ng nên suy ra bốn điểm S, K, I, O thẳng h|ng.
Vậy ba điểm O, I, K thẳng h|ng. Ta có điều phải chứng minh.
Ví dụ 10. Cho tứ gi{c ABCD. Lấy c{c điểm E v| F trên c{c cạnh AB v| CD sao cho EB FC 
. Gọi I l| trung điểm của EF, H l| trung điểm của AD, K l| trung điểm của BC. AB CD
Chứng minh rằng ba điểm H, I, K thẳng h|ng.
Phân tích tìm lời giải
Nhận thấy tứ gi{c PEQF l| hình bình h|nh nên I l| trung điểm của PQ. Để chứng
minh ba điểm H, I, K thẳng h|ng ta gọi K’ l| giao điểm của HI với BC v| chứng minh K’ l| trung điểm của BC. Lời giải Nguyễn Công Lợi TÀI LIỆU TOÁN HỌC 12
Vẽ EP//AD với P thuộc BH, vẽ FQ//AD với A E
Q thuộc CH. Trong tam gi{c ABH có B
EP//AH nên theo định lí Talets ta có P EP EB 
. Trong tam giác CDH có FQ//HD H I AH AB K K' Q FQ FC
nê theo định lí Talets ta có  HD CD EB FC EP FQ D F C Mà ta có  nên ta được  . AB CD AH HD
Mà ta có AH  HD nên suy ra EP  FQ
Mặt kh{c ta có EP//FQ nên tứ gi{c PEQF l| hình bình h|nh. Do I l| trung điểm của EF nên
I cũng l| trung điểm của PQ. EB BP
Trong tam giác ABH có EP//AH nên 
và trong tam giác CDH có FQ//HD nên AB BH FC CQ  CD CH EB FC BP CQ BP CQ Mà ta có  nên  . Trong tam giác HBC có  nên suy ra PQ//BC. AB CD BH CH BH CH IP IH
Gọi K’ l| giao điểm của HI v| BC. Trong tam gi{c HBK’ có IP//K’B nên ta có  K ' B HK ' IQ HI IP IQ
Trong tam gi{c HCK’có IQ//K’C nên ta có  . Từ đó ta được  , mà ta có K ' C HK ' BK ' CK '
IP  IQ nên suy ra BK'  CK' . Điều n|y dẫn đến hai điểm K v| K’ trùng nhau. Vậy ba điểm H, I, K thẳng h|ng.
Ví dụ 11. Cho hình thang vuông ABCD có 0
A  D  90 . Đường trong đường kính CD cắt
AB tại M v| N (M nằm giữa N v| B). Đường thẳng qua A song song với MD cắt đường
thẳng qua B song song với MC tại E. Chứng minh rằng ba điểm C, E, D thẳng h|ng.
Phân tích tìm lời giải
Nhận thấy tứ gi{c MLEF l| hình chữ nhật. Khi đó tam gi{c FCE v| MCD có FC EF EFC  CMD và  nên F  CE ∽ M
 CD do đó ta được FCE  MCD . Điều n|y dẫn MC MD
đến hai tia DE v| DC trùng nhau, tức l| điểm E, C, D thẳng h|ng. Lời giải Nguyễn Công Lợi TÀI LIỆU TOÁN HỌC 13
Gọi L l| giao điểm của AE v| MD, gọi F l| giao A D
điểm của MC v| BE. Do tam gi{c MCD nội tiếp E L
đường tròn đường kính CD nên ta được N 0
CMD  90 . Do AE//MC và CM  MD nên ta
được AE  DM. Từ đó suy ra 0 ALM  MLE  90 M
Do BE//MD và CM  MD nên ta được BE  MC. F Từ đó suy ra 0
BFC  EFM  90 . Xét hai tam giác B C
BMC và ADM có MBC  DAM và BMC  ADM . MC BC Suy ra B  MC ∽ A  DM nên  và MD MA BCF  AML
Xét hai tam giác BCF và AML có BFC  ALM và BCF  AML BC FC Suy ra B  CF ∽ A  ML nên  . Tứ gi{c MLEF có 0
LMF  EFM  MLE  90 nên là AM ML FC FD BC MC FC EF
hình chữ nhật. Do đó ta được EF  ML . Do vậy    nên  EF ML MA MD MC MD FC EF
Xét hai tam giác FE và MCD có EFC  CMD và  MC MD Suy ra F  CE ∽ M
 CD nên ta được FCE  MCD . Điều n|y dẫn đến hai tia CE v| CD trùng nhau
Vậy ba điểm E, C, D thẳng h|ng.
Ví dụ 12. Cho tam gi{c ABC nhọn có ba đường cao AD, BE, CF cắt nhau tại H. Vẽ HI
vuông góc với EF tại I, HK vuông góc với DE tại K. Gọi giao điểm của IK v| AD l| M, giao
điểm của FM v| DE l| N. Gọi S l| điểm đối xứng với B qua D. Chứng minh rằng ba điểm A, N, S thẳng hàng.
Phân tích tìm lời giải
Do c{c tứ gi{c FAEH v| ABDE nội tiếp nên HEF  BED. Từ đ}y ta được H  IE  H
 KE nên tam gi{c EIK c}n tại F v| tam gi{c HIK c}n tại H. Từ đó ta được c{c tứ
gi{c FIMH v| HMNK nội tiếp.
Để chứng minh ba điểm N, A, S thẳng h|ng ta cần chứng minh MAN  DAS hay ta AM MN chứng minh A  MN ∽ A
 DS . Muốn vậy ta cần có 
. Chú ý l| từ c{c tứ gi{c nội AD DS AM MF
tiếp trên ta suy ra được tam gi{c HFN c}n nên MF//BD, do đó ta được  . Kết hợp AD BD Nguyễn Công Lợi TÀI LIỆU TOÁN HỌC 14 AM MN
với MF  MN và BD  DS thì ta được 
. Như vậy ta có thể trình bày bài toán AD DS như sau. Lời giải Tứ gi{c FAEH có 0 AFH  AEH  90 nên tứ A
gi{c FAEH nội tiếp. Suy ra HEF  FAH Tứ gi{c ABDE có 0 ADB  AEB  90 nên tứ E
gi{c ABDE nội tiếp. Suy ra BAD  BED I
Từ đó ta được HEF  BED M F N
Xét hai tam giác vuông HIE và HKE có EH H K chung và HEI  HEK nên H  IE  H  KE B D S C
Từ đó suy ra IE  EK và HI  HK . Suy ra
tam gi{c EIK c}n tại F v| tam giác HIK cân tại H 1 Duy ra KIE   0 180  IEK 0 0
 90  IEH  KIE  FAH  90 . Mà ta có 0 NHF  FAH  90 , do 2
đó ta được KIE  MHF nên tứ gi{c FIMH nội tiếp đường tròn. Nên suy ra 0 HMF  HIF  90 Tứ gi{c HMNK có 0
HMN  HKN  90 nên tứ gi{c HMNK nội tiếp đường tròn.
Ta có HFN  HIK, HNM  HKI, HIK  HKI  HFN  HNM
Từ đó suy ra tam gi{c HFN c}n tại H nên HF  HN . Mà ta có HM  FN nên HM l| đường
trung trực của tam gi{c HFN. Ta có FM  AD, BD  AD nên ta được FM//BD. AM MF
Trong tam gi{c ABD có FM//BD nên theo định lí Talets ta có  AD BD AM MN
Lại có MF  MN và BD  DS nên ta được  AD DS AM MN
Xét hai tam giác AMN và ADS có AMN  ADS và  nên A  MN ∽ A  DS AD DS
Từ đó suy ra MAN  DAS , suy ra hai tia AN v| AS trùng nhau. Vậy ta có ba điểm A, N, S thẳng h|ng.
Ví dụ 13. Cho hình thoi ABCD có 0
BAD  60 . Đường thẳng d đi qua C cắt c{c cạnh AB, AD
lần lượt tại M, N kh{c A. Đường thẳng d’ đi qua A v| song song với BD cắt c{c đường
thẳng BN, Dm lần lượt tại E v| F. Gọi P l| giao điểm của BN với DM. Chứng minh rằng
c{c trọng t}m của c{c tam gi{c ABD, AME, AFN thẳng h|ng.
Phân tích tìm lời giải Nguyễn Công Lợi TÀI LIỆU TOÁN HỌC 15
Trên cơ sở hình vẽ v| giả thiết của b|i to{n ta nhận định được tam gi{c AFN đều.
Điều n|y dẫn đến ANB  AFD hay tứ gi{c ADNF nội tiếp v| nếu gọi J l| trọng t}m tam
giác AFN thì J là t}m đường tròn ngoại tiếp tứ gi{c ABNF nên JA  JP . Ta lại thấy tam
gi{c MAE đều nên nếu gọi I l| trọng t}m tam gi{c AME thì I l| t}m đường tròn ngoại tiếp
tứ gi{c EAPM nên suy ra IA  IP . Gọi O l| trọng t}m tam gi{c ABD. Để chứng minh ba
điểm I, J, O thẳng h|ng ta chỉ cần chứng minh được OA  OP , tức l| ta cần chứng minh O
l| t}m đường tròn ngoại tiếp tứ gi{c ABPD. Lời giải Tam giác ABD cân có 0 BAD  60 nên E M' A N' F
tam gi{c ABD đều. Trong tam gi{c BD MB MAF có BD//AF nên ta có  . AF MA O I J K B D
Trong tam giác MAN có BC//NA nên BC MB P ta có  AN MA N BD BC C Suy ra  . Mà ta có BD  BC M AF AN nên ta được AF  AN Ta lại có 0
FAN  ADB  60 và tam giác AFN cân, có 0
AFN  60 nên l| tam gi{c đều.
Từ đó ta được AF  AN  FN và 0 AFN  FNA  60 Do đó ta được A  BN  A
 DF nên suy ra ANB  AFD. Từ đó suy ra tứ gi{c ADNF nội tiếp
đường tròn. Gọi J l| trọng t}m tam gi{c AFN, khi đó J l| t}m đường trong ngoại tiếp tam
gi{c AFN. Do đó J l| t}m đường tròn ngoại tiếp tứ gi{c ABNF, suy ra JA  JP Ta có 0
NPF  NAF  60 nên ta được 0 EPM  NPF  60 M| ta lại có 0
EAM  ABD  60 nên ta được 0
EAM  EPM  60 , do đó tứ gi{c EAPM nội tiếp
đường tròn. Từ đó suy ra 0 AEM  APN  ANF  60 . Trong tam giác MAE có 0
MAE  AEM  60 nên l| tam gi{c đều. Gọi I l| trọng t}m tam gi{c
AME. Khi đó I l| t}m đường tròn ngoại tiếp tứ gi{c EAPM, suy ra IA  IP . Ta có 0
NPD  ABD  60 nên tứ gi{c ABPD nội tiếp đường tròn. Gọi O l| trọng t}m tam
gi{c ABD, khi đó O l| t}m đường tròn ngoại tiếp tứ gi{c ABPD, nên ta được OA  OP
Như vậy ta có JA  JP , IA  IP và OA  OP , suy ra ba điểm J, I, O thuộc đường trung trực
của đoạn thẳng AP. Vậy ba điểm O, I, J thẳng h|ng.
Ví dụ 14. Cho hình vuông ABCD có O l| giao điểm của hai đường chéo. Trên tia đối của 1
tia CD lấy điểm E sao cho CE  CB . Vẽ DM vuông góc với BE tại M. Gọi I l| trung điểm 2
của OB. Chứng minh rằng ba điểm A, I, M thẳng h|ng. Nguyễn Công Lợi TÀI LIỆU TOÁN HỌC 16 Lời giải
Cách 1: Hình vuông ABCD có O l| giao điểm của Q
hai đường chéo nên O l| trung điểm của AC, BD
v| AC vuông góc với BD tại O. Ta có A B OI OI 1   1
, m| ta lại có CE  CB nên ta được OA OB 2 2 I G CE 1 M  OI CE OI OA . Do đó    CB 2 OA CB CE CB O H
Hai tam giác AOI và BCE có AOI  BEC và P OI OA  nên A  OI ∽ B  CE , suy ra AIO  BEC. CE CB D C E
Gọi H l| giao điểm của BC v| DM. Đặt BC  2a a  0
Hai tam giác DCH và BCE có DCH  BCE và CDH  CBE nên D  CH ∽ B  CE DC CH 1 Do đó 
 CD.CE  BC.CH . Mà ta có CD  BC nên CH  CE , suy ra CH  CB. BC CE 2 1
Do H l| trung điểm của CB nên ta được 2 BH  CB  BH.BC  2a 2
Hai tam giác BMH và BCE có BMH  BCE và MBH  CBE nên B  MH ∽ B  CE BM BH Do đó ta được   BM.BE  BH.BC , suy ra 2 BM.BE  2a . BC BE
Trong tam gi{c BCD vuông tại C ta tính được 2 BD  8a 1 1 1 1
Ta có BI  OB và OB  BD nên BI  BD, do dó ta được 2 2 BI.BD  BD  BI.BD  2a 2 2 4 4 BI BM Do đó ta được 2 BM.BE  BI.BD  2a   BE BD BI BM
Hai tam giác BIM và BED có IBM  EBD và  nên ta được B  IM ∽ B  ED BE BD
Do đó ta được BIM  BED , mà ta có AID  BED nên ta được BIM  AID Từ đó suy ra 0
AID  DIM  BIM  DIM  180 nên 0
AIM  180 . Vậy ba điểm A, I, M thẳng hàng.
Cách 2: Vẽ AP vuông góc với MD tại P v| AQ vuông góc với MB tại Q. Từ đó suy ra tứ
gi{c APMQ l| hình chữ nhật nên 0 PAQ  90 . Mà ta có 0
DAB  90 nên ta được DAP  BAQ .
Hai tam giác vuông DAP và BAQ có AD  AB và DAP  BAQ nên D  AP  B  AQ
Do đó AP  AQ , suy ra tứ gi{c APMQ l| hình vuông. Nên MA l| ph}n gi{c của góc BMD .
Gọi H l| giao điểm của BC v| DM. Đặt AB  2a , suy ra CE  a và CD  2a nên DE  3a .
Tam giác BCE và CDH có BCE  DCH và CBE  CDH nên B  CE ∽ D  CH Nguyễn Công Lợi TÀI LIỆU TOÁN HỌC 17 CE BC 1 1 Do đó ta được 
 CE  CH . Mà ta có CE  BC nên CH  CB nên H là trung CH CD 2 2
điểm của BC, suy ra BH  a . Hai tam giác BMH và DME có BMH  DME và MBH  MDE MB BH 1 Suy ra B  MH ∽ D  ME nên ta được   MD DE 3 IB IB IB IB 1 IB MB Ta có     , do đó  ID IO  OD IB  OB IB  2IB 3 ID MD
Vẽ MG l| đường ph}n gi{c của tam gi{c BMD. GB MB IB GB IB ID IB  ID BD Ta có    , suy ra     1  IB  GB GD MD ID GD GB GD GB  GD BD
Từ đó suy ra hai điểm I v| G trùng nhau, nên MI l| ph}n gi{c của góc BMD
Từ đó ta được hai tia MA v| MI trùng nhau, nên ba điểm A, I, M thẳng h|ng.
Ví dụ 15. Cho hình vuông ABCD có hai đường chéo cắt nhau tại E. Đường thẳng qua A
cắt cạnh BC v| cắt đường thẳng CD lần lượt tại M v| N. Gọi giao điểm của EM v| BN l| K,
giao điểm của CK v| AB l| F. Chứng minh rằng ba điểm D, M, F thẳng h|ng. Lời giải
Hình vuông ABCD có hai đường chéo A S B F
cắt nhau tại E nên ta được E l| trung
điểm của hai đường chéo v| AC, BD M E K
vuông góc với nhau tại E.
Ta có EB  EC và EBA  ECB. Trên D C N
cạnh AB lấy điểm S sao cho BS  CM .
Hai tam giác BES và CEM có BS  CM ,
EBS  ECM và EB  EC nên B  ES  C  EM
Do đó ta được ES  EM và SEM  MEC . Suy ra tam gi{c SEM c}n tại E. Lại có 0
SEM  SEB  BEM  MEC  BEM  BEC  90 .Do đó tam gi{c SEM vuông c}n tại E, do đó 0
SME  45 . Tam gi{c AND có MC//AD nên theo định lí Talets ta có AN AD AN AB    MN MC MN SB
Do đó ta được SM//BN, suy ra 0 BKE  SME  45 .
Hai tam giác BKE và BDN có KBE  DBN và BKE  BDN nên B  KE ∽ B  DN BK BE Do đó ta được   BK.BN  BE.BD BD BN
Hai tam giác BEC và BCD có EBC  CBD và BEC  BCD nên B  EC ∽ B  CD Nguyễn Công Lợi TÀI LIỆU TOÁN HỌC 18 BE BC BK BC Do đó ta được 2 
 BC  BE.BD. Từ đó suy ra 2 BK.BN  BC   BC BD BC BN BK BC
Hai tam giác BKC và BCN có KBC  CBN và  nên B  KC ∽ B  CN BC BN
Do đó ta được BKC  BCN nên 0 BKC  90
Hai tam giác BCN và FBC có CBN  BFC và BCN  FBC nên B  CN ∽ F  BC BC CN BC BF AB BF Do đó ta được      BF BC CN BC CN CD MB AB MB BF
Trong tam giác MNC có AB//CN nên ta có  . Do đó ta được  . MC CN MC CD MB BF
Hai tam giác MBF và MCD có MBF  MCD và  nên M  BF ∽ M  CD MC CD
Từ đó suy ra DMF  CMD  BMF  BMD  CMD  BMD hay 0 DMF  90
Vậy ba điểm D, M, F thẳng h|ng.
Ví dụ 16. Cho s{u điểm bất kì A, B, C, D, E, F cùng nằm trên một đường tròn. Gọi G, H, K
lần lượt l| giao điểm của c{c cặp đường thẳng AB v| DE, BC v| EF, CD v| AF. Chứng
minh rằng ba điểm G, H, K thẳng h|ng.
Phân tích tìm lời giải 1 1 Ta nhận thấy BIF   và BHF   nên ta được tứ 2 sdBAF sdCDE 2 sdBAF sdCDE
gi{c BIHF nội tiếp, từ đó suy ra 0
BIH  BFH  180 . Như vậy để chứng minh được G, H, K
thẳng h|ng ta cần chứng minh được 0
BIH  BIG  180 . Muốn vậy ta cần chỉ ra được
BIG  BFH . Ngo|i ra nếu gọi A’, B’, C’ lần lượt l| giao điểm của AB v| EF, AB v| CD, CD
v| EF. Khi đó ta thấy bộ c{c điểm thẳng h|ng, từ đó ta liên tưởng đến định lí Menelaus. Lời giải
Cách 1: Gọi I l| giao điểm của hai đường A
trong ngoại tiếp tam gi{c GBD v| tam gi{c O
KDF(I kh{c D). Khi đó ta có B C
BIF  BID  DIF  BGD  DKF D 1     2   1 sdAFE sdBCD 2 sdABC sdCDF 1     2   1 sdCAE sdBCF 2 sdBAF sdCDE K I H G 1 Mà ta có BHF   2 sdBAF sdCDE
Do đó BIF  BHF , do đó tứ gi{c BIHF nội tiếp đường tròn. Từ đó suy ra 0 BIH  BFH  180 . 1
Mặt kh{c ta lại có BIG  BDG  sdBG . Mà ta có BDG  BFE 2 Nguyễn Công Lợi TÀI LIỆU TOÁN HỌC 19 Nên ta được 0
BIH  BIG  BIH  BFH  180 , suy ra 0 HIG  180
Do đó ba điểm H, I, G thẳng h|ng. Chứng minh tương tự ta được I, H, K thẳng h|ng.
Như vậy bốn điểm H, I, G, K thẳng h|ng. Do đó ba điểm G, H, K thẳng h|ng.
Cách 2: Gọi A’, B’, C’ lần lượt l| giao điểm của AB v| EF, AB v| CD, CD v| EF.
[p dụng định lí Menelaus cho tam gi{c A’B’C’ có ba điểm B, C, H thẳng h|ng ta được BA ' CB ' HC ' . .  1 BB ' CC ' HA '
[p dụng định lí Menelaus cho tam gi{c A’B’C’ với ba điểm D, G, E thẳng h|ng ta được DB ' EC ' GA ' . .  1 DC ' EA ' GB '
[p dụng định lí Menelaus cho tam gi{c A’B’C’ với ba điểm A, F, K thẳng h|ng ta được AA ' KB ' FC ' . .  1 AB ' KC ' FA '
Xét hai tam gi{c A’AF v| A’EB có AA ' F chung và A ' AF  A ' EM AA ' FA ' Do đó ta được A  ’AF ∽ A  ’EB, suy ra   AA '.BA '  EA '.FA ' EA ' BA '
Ho|n to|n tương tự ta được AB'.BB'  CB'.DB' và FC'.EC'  CC'.DC' HC ' GA ' KB ' Từ đó ta được . .  1 . HA ' GB ' KC ' HC ' GA ' KB '
Trong tam gi{c A’B’C’ có c{c điểm G, H, K thỏa mãn . .  1 , nên theo định lí HA ' GB ' KC '
Menelaus ta được ba điểm G, H, K thẳng h|ng.
Ví dụ 17. Cho điểm A nằm ngo|i đường tròn (O). Từ điểm A vẽ hai tiếp tuyến AB v|
AC(B, C l| hai tiếp điểm) v| c{t tuyến ADE với đường tròn (O) sao cho tia AD nằm giữa
hai tia AO v| AB. Đường thẳng qua D v| song song với BE cắt BC, AB lần lượt tại v| Q.
Gọi K đối xứng với B qua E. Chứng minh rằng ba điểm A, P, K thẳng h|ng. Lời giải
Gọi H, I lần lượt l| giao điểm của BC với B E K I
OA, DE. Ta có AB, AC l| hai tiếp tuyến Q D P
với đường tròn (O) nên AB  AC và AO A H O
l| ph}n gi{c của góc BAC . Do đó AO l|
đường cao của tam gi{c ABC. Xét hai x C
tam giác ABD và AEB có ABD  AEB và BAD chung, suy ra A  EB ∽ A  BD. Nguyễn Công Lợi TÀI LIỆU TOÁN HỌC 20 Từ đó 2 AB  AD.AE
Trong tam gi{c ABO vuông có BH l| đường cao nên 2 AB  AH.AO AH AD
Từ đó ta được AD.AE  AH.AO   . Từ đó A  HD ∽ A  EO nên ta được AE AO AHD  AEO
Do đó tứ gi{c OEDH nội tiếp đường tròn, suy ra OHE  ODE
Tam giác ODE có OD  OE nên c}n tại O, suy ra ODE  OED
Từ đó ta được OHE  AHD. Ta có 0
OHE  EHI  AHD  IHD  90 nên ta được EHI  IHD , do
đó HI l| tia ph}n gi{c của góc HED . Gọi Hx l| tia đối của tia HE, khi đó ta có xHA  AHD  OHE ID AD
Do đó HA l| đường ph}n gi{c của HED . Từ đó ta được  ED AE DQ AD
Trong tam gi{c ABE có DQ//BE nên theo định lí Talets ta có  BE AE DP ID DQ DP
Trong tam giác IBE có BE//PD nên ta có  . Từ đó ta được  nên DQ  DP . BE IE BE BE AQ QD AQ 2DQ PQ
Trong tam giác ABE có DQ//BE nên ta có  . Do đó ta được   AB BE AB 2BE BK AQ PQ Hai tam giác APQ và AKB có  và AQP  ABK nên A  PQ ∽ A  KB AB BK
Từ đó ta được QAP  BAK nên hai tia AP v| AK trùng nhau. Vậy ba điểm A, P, K thẳng hàng.
Ví dụ 18. Cho điểm A nằm ngo|i đường tròn t}m O b{n kính R. Vẽ c{c tiếp tuyến AB, AC
với đường tròn (O)(B, C thuộc đường tròn (O)). Vẽ c{t tuyến ADE với đường tròn (O)(D, E
thuộc đường tròn (O)) v| D nằm giữa A, E. Tia AD nằm giữa hai tia AO v| AB. Gọi F l|
điểm đối xứng với D qua AO, H l| giao điểm của EF với BC. Chứng minh rằng A, H, O thẳng h|ng.
Phân tích tìm lời giải
Gọi H’ l| giao điểm của OA với BC. Để chứng minh A, H, O thẳng h|ng ta cần
chứng minh được hai điểm H v| H’ trùng nhau. Muốn vậy ta cần chứng minh 0
EH ' F  AH ' F  AH ' E  180 . Ngo|i ra ta cũng thấy A, O cùng thuộc đường trung trực của
BC nên để chứng minh được A, H, O thẳng h|ng ta cần chứng minh H thuộc đường trung trực AO hay HB  HC . Lời giải Nguyễn Công Lợi TÀI LIỆU TOÁN HỌC 21
Cách 1: Gọi H’ l| giao điểm của AO v| B
BC. Do D v| F đối xứng với nhau qua E
AO nên ta được OF  OD  R . Suy ra F D
thuộc đường tròn (O) v| có A O H' H
AH ' D  AH ' F . Do AB, AC l| hai tiếp F
tuyến của đường tròn (O) nên ta được
AB  AC v| AO l| tia ph}n gi{c của C
góc BAC. Do đó tam gi{c ABC c}n tại
A và AO  BC . Xét hai tam giác ABD
và AEB có ABD  AEB và BAD chung AB AD Do đó ta được A  BD ∽ A  EB nên 2   AB  AD.AE . AE AB
Tam gi{c ABO vuông tại B có BH’ l| đường cao nên 2 AB  AH '.AO . AD AH '
Từ đó ta được AD.AE  AH '.AO   AO AE AD AH '
Xét hai tam gi{c ADH’ v| AOE có  và DAH ' chung nên A  DH' ∽ A  OE AO AE
Từ đó ta được ADH '  AOE nên tứ gi{c DH’OE nội tiếp. Suy ra AH ' D  OED và OH ' E  ODE
Mà ta có OE  OD  R nên tam gi{c ODE c}n tại O, suy ra OED  ODE
Do đó AH ' D  OH ' E , vì vậy ta được AH ' F  OH ' E . Ta có 0
EH ' F  AH ' F  AH ' E  OH ' E  AH ' E  180 . Điều n|y dẫn đến ba điểm E, H’, F
thẳng h|ng, suy ra hai điểm H v| H’ trùng nhau. Vậy ba điểm A, H, O thẳng h|ng. Ta có điều phải chứng minh.
Cách 2: Do AB, AC l| tiếp tuyến với đường tròn (O) nên ta được AB  AC và AO là tia
ph}n gi{c của góc BAC. Do đó tam gi{c ABC c}n tại A v| AO  BC . Do D v| F đối xứng
với nhau qua AO nên ta suy ra OF  OD  R và DF  AO. Ta có DF//BC nên tứ gi{c DBCF
l| hình thang, m| từ gi{c DBCF nội tiếp đường trong (O) nên DBCF l| hình thang c}n. Từ
đó ta được BD  CE, BF  CD.
Xét hai tam giác ABD và AEB có ABD  AEB và BAD chung. BD AB Suy ra A  BD ∽ A  EB nên ta được  . BE AE AC CD
Ho|n to|n tương tự ta được A  CD ∽ A  EC nên suy ra  . AE CE
Xét hai tam giác HBF và HEC có BHF  EHC và BFH  HCE Nguyễn Công Lợi TÀI LIỆU TOÁN HỌC 22 HB BF Do đó ta được H  BF ∽ H  EC nên ta được  HE CE CF HC
Tương tự ta cũng được H  FE ∽ H  BE nên suy ra  BE HE HB BF CD AC AB BD CF HC Từ đó suy ra       
. Do đó ta được HB  HC . HE CE CE AE AE BE BF HE
Ta có HB  HC , AB  AC và OB  OC nên ba điểm H, A, O thuộc đường trung trực của
đoạn thẳng BC. Do đó ba điểm H, A, O thẳng h|ng.
Ví dụ 19. Cho đường tròn (O) nội tiếp tam gi{c ABC. Đường tròn (O) tiếp xúc với c{c cạnh
BC, CA, AB lần lượt tại D, E, F. Trên đoạn OD lấy điểm I, vẽ đường tròn tâm I bán kính
ID. Vẽ BG, CH l| c{c tiếp tuyến của đường tròn (I) với G v| H thuộc đường tròn (I). Gọi M
l| giao điểm của BG v| CH, N l| giao điểm của EF v| BC. Chứng minh rằng ba điểm N, G, H thẳng h|ng.
Phân tích tìm lời gải
Nhận thấy tứ gi{c EFHG nội tiếp đường tròn. Để chứng minh được ba điểm N, H,
G thẳng h|ng ta gọi giao điểm của NG với đường tròn (I) l| H , giao điểm của NG với 1
đường tròn ngoại tiếp tứ gi{c EFHG l| H v| ta đi chứng minh hai điểm H và H trùng 2 1 2 nhau. Lời giải A F M H E O G I N B C D
Từ tính chất hai tiếp tuyến cắt nhau ta có AE  AF, BG  BD  BF,CE  CH  CD, MG  MH
Tam gi{c AEF c}n tại A nên AEF  AFE , suy ra 0 2AEF  BAC  180 1 Trong tam giác ABC có 0
BAC  ABC  ACB  180 . Do đó ta được AEF  ABC  ACB 2 1 Tam gi{c CEH c}n tại C có 0 AEH  90  ECH 2 1 Do đó ta được 0
HEF  AEH  AEF  90  ECH  ABC  ACB 2 Nguyễn Công Lợi TÀI LIỆU TOÁN HỌC 23 1 1
Ho|n to|n tương tự ta được 0
MGH  MBC  MCB; MGF  90  GBF 2 2 1 Do đó 0
HGF  90  MBC  MCB  GBF . Suy ra 0
HEF  HGF  180 nên tứ gi{c EHGF nội 2 tiếp
Gọi giao điểm của NG với đường tròn (I) l| H , giao điểm của NG với đường tròn ngoại 1
tiếp tứ gi{c EHGF l| H . Khi đó ta có N  GD ∽ N  DH nên ta được 2 1 NG ND 2   NG.NH  ND 1 ND NH1 NF NG Ta cũng có N  FG ∽ N  H E nên ta được   NE.NF  NH .NG 2 2 NH NE 2 ND NF Và N  DF ∽ N  ED nên ta có 2   ND  NE.NF NE ND
Do đó ta được NH  NH , suy ra hai điểm H và H trùng nhau. Nên ba điểm H, H và H 1 2 1 2 1 2 trùng nhau.
Vậy ba điểm N, G, H thẳng h|ng.
Ví dụ 20. Cho đường tròn (O; R) có đường kính AB. Lấy điểm C nằm giữa hai điểm O v|
A. Vẽ đường tròn (I) đường kính BC. Vẽ tiếp tuyến AD v| c{t tuyến ACF với đường tròn
(I)(E nằm giữa A v| F) sao cho tia AO nằm giữa hai tia AD v| AE. Đường thẳng vuông góc
với AB vẽ từ C cắt đường tròn (O) tại N v| P(N v| P thuộc nửa mặt phẳng bờ AB). Giao
điểm của DI v| NB l| S. Gọi J l| trung điểm của SD. Gọi L, T lần lượt l| t}m đường tròn
ngoại tiếp c{c tam gi{c SBC v| SEF. Chứng minh rằng ba điểm J, L, K thẳng h|ng. Lời giải
Xét tam giác ADC và ABD có ADC  ABD và P DAC nên A  DC ∽ A  BD. Do đó D AD AC 2   AD  AB.AC AB AD L C J
Tam gi{c ANB vuông tại N có NC l| đường cao A B O I nên 2
AN  AB.AC . Từ đó ta được AN  AD . R E S
Gọi R l| giao điểm của DN v| AS. Hai tam gi{c T F
vuông DAS và NAS có AS chung và AD  AN N nên A  DS  N
 AS. Từ đó suy ra DAS  NAS.
Ta giác AND có AD  AN nên c}n tại A, AR l|
đường ph}n gi{c nên đồng thời l| đường cao của tam giác AND. AC AR
Tam gi{c ADS vuông tại D nên ta có 2
AD  AR.AS. Do đó AR.AS  AB.AC   . AS AB Nguyễn Công Lợi TÀI LIỆU TOÁN HỌC 24 AC AR
Xét hai tam giác ACR và ASB có CAR chung và  nên A  CR ∽ A  SB AS AB
Từ đó ta được ACR  ASB, do đó tứ gi{c CBSR nội tiếp đường tròn. Suy ra L thuộc đường
trung trực của SR. Chứng minh tương tự ta được tứ gi{c RSEF nội tiếp đường tròn, suy ra
T thuộc đường trung trực của đoạn thẳng SR. Mặt kh{c ta có tam gi{c RDS vuông tại R có
RJ l| đường trung tuyến nên ta được RJ  SJ  DJ . Do đó J cũng thuộc đường trung trực
của SR. Vậy ba điểm J, L, T thuộc đường trung trực của SR nên ba điểm J, L, T thẳng h|ng.
Ví dụ 21. Cho tam gi{c nhọn ABC có AB  AC . Đường tròn (I) đường kính BC cắt AB, AC
lần lượt tại F, E. Giao điểm của BE v| CF l| H, giao điểm của AH v| BC l| D, giao điểm
của EF v| BC l| K. Đường thẳng qua D song song với EF cắt AB tại M. Đường tròn ngoại
tiếp tam gi{c IMK cắt AC tại N. Chứng minh rằng ba điểm M, D, N thẳng h|ng. Lời giải Ta có 0
BFC  BEC  90 nên tam giác ABC có A
hai đường cao BE v| CF, do đó ta được AD  BC Tứ gi{c BFHD có 0 BFH  BDH  90 nên nội F
tiếp được đường tròn. Suy ra ra được DBH  DFH M H E 1
Mà ta có DBH  CFE nên DBH  DFE , lại 2 C B 1 I D K
có DBH  CIE , suy ra ta được CIE  DFE N N' 2
Từ đó dẫn đến tứ gi{c IFED nội tiếp. Suy ra
IDF  IEF , mà IEF  IFE nên ta được
IDF  IFK . Hai tam giác IDF và IFK có DIF chung và IDF  IFK nên I  DF ∽ I  FK ID IF Do đó 2   IF  ID.IK nên IF IK 2 ID.IK  IC
Gọi N’ l| giao điểm của MD v| AC. Ta có DN ' C  AEF và AEF  MBD nên DN ' C  MBD DN ' DC Từ đó ta được D  N'C ∽ D  BM, suy ra 
 DB.DC  DM.DN ' . Từ đó suy ra DB DM
DM.DN '  DB.DC  IB  IDIC  ID  IC  IDIC  ID 2 2  IC  ID 2
 ID.IK  ID  IDIK  ID  ID.KD DM DI Do đó 
, kết hợp với MDI  KDN ' ta được D  MI ∽ D  KN' nên DMI  DKN' , DK DN '
suy ra tứ gi{c IMKN’ nội tiếp đường tròn. Suy ra N v| N’ trùng nhau. Vậy ba điểm M, D, N thẳng h|ng. Nguyễn Công Lợi TÀI LIỆU TOÁN HỌC 25
Ví dụ 22. Cho điểm A nằm ngo|i đường tròn (O; R). Vẽ c{c c{t tuyến AEB, ADC với
đường tròn (O)(E, B, C, D thuộc đường tròn (O), E nằm giữa A v| B, D nằm giữa A v| C).
Gọi H l| giao điểm của BD v| CE. Vẽ c{c tiếp tuyến AM, AN với đường trong (O) (M, N l|
c{ tiếp điểm, B v| M nằm cùng nửa mặt phẳng bờ AH). Gọi K l| giao điểm của BN v| CM.
Chứng minh rằng ba điểm A, H, S thẳng h|ng. Lời giải
Gọi H’ l| giao điểm của MN v| BD, E’ l| giao điểm A
của CH’ với đường tròn (O). Ta có 0 AMO  ANO  90
, nên tứ gi{c AMON nội tiếp đường tròn (I) đường D
kính OA. Gọi S l| giao điểm của AH’ với dường tròn N E' E
(I). Khi đó dễ d|ng chứng minh được H' H F
H' E '.H' C  H' M.H' N  H' A.H' S . Từ đó suy ra tứ M
gi{c AE’SC nọi tiếp đường tròn, nên ta được K O S
AE ' C  ASC . Gọi F l| giao điểm của OA với MN. Ta B C cũng chứng minh được 2
AH '.H ' C  AF.AO  AN  AC.AD . Do đó tứ gi{c H’SCD nội tiếp, suy ra 0
H ' SC  H ' DC  180 . M| ta lại có
BE ' C  H ' DC , từ đó ta suy ra được 0
AE ' C  BE ' C  180 . Do đó ba điểm A, E’, B thẳng
h|ng, suy ra hai điểm E v| E’ trùng nhau, H v| H’ trùng nhau.
Vậy ba điểm M, H, N thẳng h|ng. Gọi K’ l| giao điểm của AH v| CM.
Xét đường tròn (I) có ASN  MNA v| trong đường tròn (O) có MCN  MNA
Do đó ta được ASN  MCN , suy ra tứ gi{c K’SNC nội tiếp. Nên ta có 0 K' SC  K' NC  180
Ta cũng chứng minh được HB.HD  HM.HN  HA.HS, nên tứ gi{c ABSD nội tiếp đường tròn.
Suy ra ta có ABD  ASD. Trong đường tròn (O) ta có ABD  HCD nên ta được ASD  HCD
, do đó tứ gi{c SHDC nội tiếp. Suy ra 0
HSC  HDC  180 . M| ta lại có HDC  BNC , do đó K ' NC  BNC
Suy ra hai tia NK’ v| NB trùng nhau, do đó ba điểm B, K, N thẳng h|ng. Ta có hai điểm K
v| K’ trùng nhau nên A, K, S thẳng h|ng. Vậy ba điểm A, H, S thẳng h|ng. Nguyễn Công Lợi TÀI LIỆU TOÁN HỌC 26
Ví dụ 23. Cho đường tròn (I) nội tiếp tam gi{c ABC. Đường tròn (I) tiếp xúc với BC, AB,
CA lần lượt tại D, E, F. Gọi AM l| đường trung tuyến của tam gi{c ABC. Gọi giao điểm
của EF v| DI l| K. Chứng minh rằng ba điểm A, K, M thẳng h|ng.
Phân tích tìm lời giải
Trên cơ sở hình vẽ v| giả thiết của b|i to{n ta có hai nghĩa đến hai hướng chứng
minh ba điểm A, K, M thẳng h|ng là.
+ Qua K vẽ đường thẳng song song với BC cắt AB, AC lần lượt tại X v| Y. Gọi N l|
giao điểm của AK v| BC. Sử dụng tính chất hai tiếp tuyến cắt nhau, định lí Talets v| tính
chất đường ph}m gi{c ta đi chứng minh NB  NC . Điều n|y sẽ dẫn đến M v| N trùng nhau.
+ Qua K vẽ đường thẳng song song với BC cắt AB, AC lần lượt tại X v| Y. Sử dụng
tính chất hai tiếp tuyến cắt nhau, định lí Talets v| tính chất đường ph}n gi{c ta đi chứng
minh hai tia AK và AM trùng nhau. Lời giải
Cách 1: Gọi N l| giao điểm của AK A L S và BC.
Ta có AE v| AF l| hai tiếp tuyến F
của đường tròn (I) nên ta được K X Y
EAI  FAI . Qua A vẽ đường thẳng E I
song song với BC cắt DI tại L v| cắt
EF tại S. Qua K vẽ đường thẳng B D N M C
song song với BC cắt AB, AC lần lượt tại X v| Y.
Ta có DI  BC và AS//BC nên
AL  LI . Do đó ta được 0 ALI  AEI  AFI  90
Từ đó suy ra c{c điểm A, E, I, F, L
cùng nằm trên một đường tròn.
Do đó ta được ELI  EAI  IAF  ILF , suy ra LK là ph}n gi{c của tam gi{c LEF.
Do đó LK v| LS l| đường ph}n gi{c trong v| ph}n gi{c ngo|i của tam gi{c LEF. KE SE KE KF Nên ta được    KF SF SE SF XK KE KY KF
Ta có XK//AS và KY//AS nên ta có  và  . Từ đó suy ra KX  KY AS SE AS SF Nguyễn Công Lợi TÀI LIỆU TOÁN HỌC 27 XK AK
Trong tam gi{c ABN có XK//BN nên theo định lí Talets ta có  BN AN KY AK
Trong tam gi{c ACN có KY//CN nên theo định lí Talets ta có  CN AN
Kết hợp với KX  KY ta suy ra được BN  CN hay N l| trung điểm của BC.
Từ đó suy ra M v| N trùng nhau. Vậy ba điểm A, K, M thẳng h|ng.
Cách 2: Qua K vẽ đường thẳng song song với BC cắt AB, AC lần lượt tại X v| Y.
Ta có DI  BC v| XY//BC nên ta được DI  XY Ta có 0
XEI  XKI  90 nên tứ gi{c EXKI nội tiếp đường tròn, suy ra IEK  IXK
Ho|n to|n tương tự ta được tứ gi{c IKFY nội tiếp nên IFK  IYK
M| tam gi{c IEF c}n tại I nên IEK  IFK , do đó ta được IXK  IYK
Tam gi{c IXY c}n tại Y có IK l| đường cao nên IK cũng l| đường trug tuyến. XY XK 2XK XY
Do đó ta được XK  YK  . Suy ra   2 BM 2BM BC AX XY
Trong tam gi{c ABC có XY//BC nên theo định lí Talets ta có  AB BC AX XK Hai tam giác AXK và ABM có  và AXK  ABM nên A  XK ∽ A  BM AB BM
Điều n|y dẫn đến XAK  BAM , suy ra hai tia AK v| AM trùng nhau. Vậy ba điểm A, K, M thẳng h|ng
Ví dụ 24. Cho tam gi{c ABC nội tiếp đường tròn (O). Lấy điểm D trên cạnh BC. Đường
tròn t}m P tiếp xúc với AD, DC v| tiếp xúc trong với đường tròn (O). Đường tròn t}m Q
tiếp xúc với AD, BD v| tiếp xúc trong với đường tròn (O). Gọi I l| t}m đường tròn nội tiếp
tam gi{c ABC. Chứng minh rằng ba điểm P, I, Q thẳng h|ng. Lời giải
Trước hết ta ph{t biểu v| chứng minh hai bổ để sau:
Bổ đề 1: Cho đường tròn (I) tiếp xúc trong với đường tròn C
(O) tại C. AB là dây bất kì của đường tròn (O) và tiếp xúc I
với đường tròn (I) tại T. Khi đó CT đi qua điểm chính giữa A B T
của cung AB không chứa C. O Chứng minh
Gọi M l| giao điểm của CT với đường tròn (O)(M kh{c C). M
Do hai đường tròn (I) v| (O) tiếp xúc nhau nên ba
điểm C, O, I thẳng h|ng. Tam gi{c CIT có ICT  ITC Nguyễn Công Lợi TÀI LIỆU TOÁN HỌC 28
và tam giác OCM có OCM  OMC
Do đó ta được ITC  OMC nên suy ra IT//OM
Mà ta có IT  AB nên suy ra OM  AB . Do đó M l| điểm chính giữa cung AB không chứa C.
Bổ đề 2: Cho tam giác ABC nội tiếp đường tròn (O). Lấy điểm M thuộc BC. Đường tròn (O’) tiếp
xúc với hai đoạn thẳng MA, MC lần lượt tại E, F đồng thời tiếp xúc với đường tròn (O) tại K. Khi
đó tâm I của đường tròn nội tiếp tam giác ABC nằm trên đoạn thẳng EF. Chứng minh
Vẽ tiếp tuyến Kx của đường tròn (O). Gọi N l| giao A
điểm của KF với đường tròn (O). Khi đó theo bổ đề
1 thì điểm N nằm chính giữa cung BC của đường K E
tròn (O). Gọi I’ l| gao điểm của AN v| EF, khi đó ta x I' O I
có BN  CN nên ta được BAN  CAN và NB  NC .
Do đó AN l| ph}n gi{c của góc BAC B M F C
Ta có I ' EK  FKx và I ' AK  NHx suy ra N
I ' EK  I ' AK nên tứ gi{c AKI’e nội tiếp đường
tròn. Do đó suy ra ta được AI ' K  AEK , mà ta có
AEK  EFK nên AI ' K  EFK Ta có NI ' K  NFI ' .
Xét hai tam gi{c NI’K v| NFI’ có NI ' K  NFI ' và I ' NK nên ta được N  I ' K ∽ N  FI ' NI ' NK Do đó ta được ' 2   NI  NK.NF NF NI '
Xét hai tam giác NCF và NKC có NCF  NKC và CNF chung nên ta được N  CF ∽ N  KC NC NF Do đó ta được 2   NC  NF.NK NK NC
Từ đó ta được NC  NI ' hay tam gi{c NCI’c}n tại N, do đó ta được NI ' C  NCI '
Từ đó suy ra I ' CA  I ' AC  BCN  BCI '  I ' CA  BCI ' hay CI’ l| ph}n gi{c của góc ACB
Do đó I’ l| t}m đường tròn nội tiếp tam gi{c ABC, suy ra hai điểm I v| I’ trùng nhau.
Vậy I nằm trên đoạn thẳng EF. Nguyễn Công Lợi TÀI LIỆU TOÁN HỌC 29
Trở lại b|i to{n: Gọi E, F lần lượt l| tiếp điểm A
của đường tròn (P) với BC, AD. Gọi G, H lần
lượt l| tiếp điểm của đường tròn (Q) với BC,
AD. Gọi I l| giao điểm của EF với GH. Từ bổ
đề 2 suy ra I l| t}m đường tròn nội tiếp tam P F I
gi{c ABC. Gọi M l| giao điểm của GH v| QD, H Q M N
N l| giao điểm của EF v| PD. Dễ thấy MDNI B G D E C
l| hình chữ nhật. Hai tam gi{c QDG v| DPE
có QGD  PEF và QDG  DPE . Nên QD QG Q  DG ∽ D  PE   , PD DE
Xét hai tam giác QMG và DNE có
QMG  DNE và MQG  NDE . Nên ta được Q  MG ∽ D  E , QG QM QM QG QM QD QG Suy ra  . Mà ta có DN  MI nên  Từ đó   , suy ra DE DN MI DE MI DP DE QM MI  QD DP QM MI
Xét hai tam giác QMI và QDP có 
và QMI  QDP . Nên ta được Q  MI ∽ Q  DP , QD DP
do đó ta được IQM  PQD . Từ đó suy ra hai tia QI v| QP trùng nhau hay ba điểm Q, I, P thẳng h|ng.
Nhận xét: Bổ đề 2 trong ví dụ 23 chính là nội dung định lí Lyness. Ba điểm P, I, Q thẳng hàng
trong ví dụ 23 nằm trên một đường thẳng có tên là đường thẳng The’bault.
Ví dụ 25. Cho tam gi{c nhọn ABC nội tiếp đường tròn (O; R). C{c tia AO, BO, CO cắt cạnh
BC, CA, AB lần lượt tại D, E, F. Gọi M l| giao điểm của hai tiếp tuyến tại A, B của đường 9
tròn (O). Chứng minh rằng nếu có DE  BE  CF  R thì ba điểm C, F, M thẳng h|ng. 2 Lời giải S AD S AD Ta có ABD  và ACD  . Do đó ta M A S AO S AO AOB AOC được F E AD S S S  S ABD ACD ABD ACD    AO S S S  S O AOB AOC AOB AOC S  S  S S AOB AOC BOC BOC   1  B D C S  S S  S AOB AOC AOB AOC Nguyễn Công Lợi TÀI LIỆU TOÁN HỌC 30 AD S Từ đó ta được BOC  1  . R S  S AOB AOC BE S CF S
Ho|n to|n tương tự ta được AOC AOB  1  ;  1  R S  S R S  S BOC AOB AOC BOC AD  BE  CF S S S Từ đó ta được BOC AOC AOB  3    R S  S S  S S  S AOB AOC BOC AOB AOC BOC S S S 3
Theo bất đẳng thức Neibitz ta có BOC AOC AOB    S  S S  S S  S 2 AOB AOC BOC AOB AOC BOC AD  BE  CF 9 9 Từ đó suy ra  hay AD  BE  CF  R R 2 2 9
M| ta lại có DE  BE  CF  R , tức l| dấu bằng của bất đẳng thức trên xẩy ra, 2 Do đó ta được S  S  S
, điều n|y xẩy ra khi v| chỉ khi AOB AOC BOC 1 OA.OB.sin 1 1 0
180  AOB  OB.OC.sin 0
180  BOC  OC.OA.sin 0 180  COA 2 2 2
Hay AOB  BOC  COA . Do đó ba tam gi{c AOB, BOC, COA bằng nhau nên AB  BC  AC
Vậy tam gi{c ABC đều. M| ta lại có OA  OB và MA  MB nên C, O, M thuộc đường
trung tực của đoạn thẳng AB. Do đó ba điểm C, O, M thẳng h|ng. M| ba điểm C, O, F
thẳng h|ng. Từ đó ba điểm C, F, M thẳng h|ng.
Ví dụ 26. Cho tam giác ABC có AB  AC nội tiếp đường tròn t}m O. Trên c{c tia đối của
tia BA, CA lấy c{c điểm M, N sao cho BM  AC;CN  AB . Gọi I l| điểm chính giữa cung
lớn BC. Gọi K l| điểm đối xứng với I qua BC. Chứng minh ba điểm M, N, K thẳng h|ng.
Phân tích tìm lời giải
Đầu tiên ta nhận thấy tam gi{c AMN c}n tại A, tam gi{c BIC c}n tại I v| tam gi{c
BKC c}n tại K. Lấy điểm H sao cho tứ gi{c ABHC l| hình bình h|nh thì ta được
BHC  BAC nên BHC  BKC , điều n|y dẫn đến BCHK nội tiếp đường tròn. Từ đó để
chứng minh ba điểm M, N, K thẳng h|ng ta đi chứng minh ba điểm H, N, H thẳng h|ng
v| ba điểm M, N, H thẳng h|ng. Tất nhiên ta cũng có thể lấy điểm H trên MN thỏa mãn
CH//AB rồi chứng minh tứ gi{c ABHC l| hình bình h|nh cũng được. Lời giải Nguyễn Công Lợi TÀI LIỆU TOÁN HỌC 31
Ta có BM  AC;CN  AB nên ta suy ra I A
được AM  AN , do đó tam gi{c AMN c}n
tại A. Do I v| K đối xứng với nhau qua O BC nên BIC  BKC . B C
Mà ta có BIC  BAC nên ta được BKC  BAC N Tam gi{c BIC c}n tại I. H K M| ta lại có B  KC  B  IC nên tam giác M BKC c}n tại K.
Trên đoạn MN lấy điểm H sao cho
CH//AB. Ta có CHN  AMN . M| ta lại có
AMN  CNH nên ta được CHN  CNH
Do đó tam gi{c CHN c}n tại C, suy ra CH  CN. Lại có CN  AB nên ta được CH  AB
Tứ gi{c ABHC có CH//AB v| CH  AB nên là hình bình hành.
Do đó ta được BHC  BAC nên suy ra BHC  BKC . Từ đó suy ra tứ gi{c BCHK nội tiếp đường tròn.
Ta có HCN  BAC và BKC  BAC nên ta được HCN  BKC
Do hai tam gi{c HCN v| BKC c}n nên ta được H  CN ∽ B  KC
Do đó ta được CHN  KBC nên suy ra CHN  CHK  KBC  CHK M| ta lại có 0
CHN  CHK  NHK; KBC  CHK  180 nên ta được 0 NHK  180 .
Do đó ba điểm H, N, H thẳng h|ng. M| ta lại có M, N, H thẳng h|ng.
Vậy ba điểm M, N, K thẳng h|ng.
Ví dụ 27. Cho tam gi{c ABC vuông tại A có AB  AC, AH l| đường cao. Lấy điểm M nằm
giữa A v| H. Trên c{c đoạn MC, MB lấy c{c điểm E v| F sao cho BE  BA và CF  CA. Gọi
I l| giao điểm của BE v| CF. Gọi K l| giao điểm thứ hai của hai đường tròn ngoại tiếp tam
gi{c BHF v| CHE. Chứng minh rằng ba điểm I, H, K thẳng h|ng.
Phân tích tìm lời giải
Để chứng minh ba điểm H, I, K thẳng h|ng ta gọi G l| giao điểm thứ hai của IH với
đường tròn ngoại tiếp tam gi{c BFH v| đi chứng minh hai điểm G v| K trùng nhau. Muốn
vậy ta đi chứng tứ gi{c GHEC nội tiếp đường tròn. Vẽ BP vuông góc với CM tại P, BP cắt
AH tại S, BM cắt SC tại Q. Khi đó tam gi{c nhọn SBC có A l| trực t}m. Từ đó ta chứng
minh được SE  SF nên dẫn đến IE  IF . Ta lại nhận thấy B  EH ∽ B  CE nên suy ra Nguyễn Công Lợi TÀI LIỆU TOÁN HỌC 32
BEH  BCE và tương tự ta được CFH  CBF nên từ đó ta suy ra được CFH  IGF . Điều IE IH n|y dẫn đến I  FH ∽ I  GF nên 2 2
IF  IH.IG  IE  IH.IG   . IG IE
Từ đó suy ra hai tam gi{c IEG v| EIH đồng dạng với nhau, nên ta được IEH  IGE nên
HGE  HCE , tức l| tứ gi{c GHEC nội tiếp. Vậy b|i to{n được chứng minh. Lời giải
Vẽ BP vuông góc với CM tại P, BP cắt AH tại S, S
BM cắt SC tại Q. Gọi O l| t}m của đường tròn
ngoại tiếp tam gi{c BHF. HI cắt đường tròn (O)
tại điểm thứ hai l| G. Trong tam gi{c SBC có
SH, CP l| c{c đường cao cắt nhau tại M nên M A
l| trực t}m của tam gi{c SBC, do đó BM  SC . Q P
Hai tam giác BHS và BPC có HBS chung và M F E BHS  BPC nên B  HS ∽ B  PC. Suy ra ta được I BH BS 
nên suy ra BH.BC  BP.BS . Tam giác B H C BP BC O
ABC vuông tại A có đường cao AH nên ta được K G 2
AB  BH.BC . Từ đó suy ra 2 BE  BH.BC , mà ta BE BS có BH.BC  BP.BS nên 2 BE  BP.BS   BP BE
. Hai tam giác BES và BPE có EBS chung và BE BS  nên B  ES ∽ B  PE BP BE
Do đó ta được BES  BPE , mà ta có 0 BPE  90 nên 0
BES  90 hay tam gi{c BES vuông tại E.
Trong tam gi{c BES vuông tại E có EP l| đường cao nên 2 SE  SP.SB
Ho|n to|n tương tự ta được 2 SF  SQ.SC .
Hai tam giác SPC và SQ có PSC chung và SPC  SQB nên S  PC ∽ S  QB SP SC Do đó ta được 
 SQ.SC  SP.SB. Từ đó ta được SE  SF . Từ đó suy ra hai tam SQ SB
gi{c IES v| IFS bằng nhau nên IE  IF . BE BH
Hai tam giác BEH và BCE có EBH chung và  nên ta được B  EH ∽ B  CE BC BE
Do đó ta được BEH  BCE . Ho|n to|n tương tự ta được CFH  CBF
Mà ta có CBF  IGF nên suy ra CFH  IGF Nguyễn Công Lợi TÀI LIỆU TOÁN HỌC 33
Hai tam giác IFH và IGF có FIH chung và IFH  IGF nên I  FH ∽ I  GF IF IH IE IH Do đó ta được 2 2 
 IF  IH.IG  IE  IH.IG   . IG IF IG IE
Từ đó suy ra hai tam gi{c IEG v| EIH đồng dạng với nhau, nên ta được IEH  IGE
Do đó IGE  HCE hay HGE  HCE , suy ra tứ gi{c ECGH nội tiếp đường tròn hay đường
tròn ngoại tiếp tam gi{c CHE đi qua điểm G. Do đó G l| giao điểm thứ hai của hai đường
tròn ngoại tiếp c{c tam gi{c BHF v| CHE. Suy ra hai điểm G v| K trùng nhau. Vậy ba điểm I, H, K thẳng h|ng.
Ví dụ 28. Cho tam gi{c nhọn ABC có AB  AC , đường cao AH. Đường tròn (O; R) nội tiếp
tam gi{c ABC tiếp xúc với BC tại D. Gọi E l| trung điểm của AH, tia DE cắt đường trog (O)
tại F. Gọi L l| t}m đường tròn b|ng tiếp góc F của tam gi{c FBC. Chứng minh rằng ba điểm F, D, L thẳng h|ng.
Phân tích tìm lời giải
Gọi M, N lần lượt l| tiếp điểm của đường tròn (O) trên c{c cạnh AB, AC. Đường
thẳng AD cắt đường tròn (O) tại T kh{c D. DO cắt đường tròn (O) tại điểm thứ hai l| G. Vẽ
OK vuông góc với DT tại K, OK cắt BC tại S. Qua A kẻ đường thẳng song song với BC cắt
MN tại J, BP vuông góc với GF tại P, CQ vuông góc với GF tại Q v| I l| giao điểm của OA
với MN. Ta có FL l| ph}n gi{c của góc BFC . Nếu ta chứng minh được FD cũng l| ph}n
gi{c của góc BFC thì ba điểm F, D, L thẳng h|ng. Như vậy ta đi chứng minh FD l| ph}n
gi{c của góc BFC . Muốn vậy ta cần chứng minh được BFD  CFD . Để ý l| 0
BFP  BFD  CFQ  CFD  90 nên ta cần chứng minh được BFP  CFQ . Như vậy ta cần chứng minh được B  PF ∽ C
 QF . Lại chu ý l| BPF  CQF nên ta sẽ đi chứng minh BP PF  . CQ QF Lời giải Nguyễn Công Lợi TÀI LIỆU TOÁN HỌC 34
Gọi M, N lần lượt l| tiếp điểm J P A
của đường tròn (O) trên c{c cạnh G M T
AB, AC. Đường thẳng AD cắt F I Q E N
đường tròn (O) tại T kh{c D. DO K O
cắt đường tròn (O) tại điểm thứ B
hai l| G. Vẽ OK vuông góc với D C S
DT tại K, OK cắt BC tại S. Qua A
kẻ đường thẳng song song với
BC cắt MN tại J, BP vuông góc
với GF tại P, CQ vuông góc với
GF tại Q v| I l| giao điểm của L OA với MN.
Khi đó dễ d|ng chứng minh
được OA l| đường trung trực của đoạn thẳng MN nên
MN  OA tại I. Ta có OK vuông
góc với DT tại K nên K l| trung điểm của DT.
Tam gi{c ODS vuông tại D có DK l| đường cao nên 2 OD  OK.OS
Tam gi{c OMA vuông tại M có MI l| đường cao nên 2 OM  OI.OA OK OA
Mà ta có OM  OD nên ta được OK.OS  OI.OA   OI AS Từ đó suy ra O  IS ∽ O  KA nên 0 OIS  OKA  90
Do IS v| MN cùng vuông góc với OA tại I nên IS v| MN trùng nhau, suy ra ba điểm M, N, S thẳng h|ng.
Hai tam giác DTG và AHD có DTG  AHD và GDT  DAH nên D  TG ∽ A  HD TG TD 2TK TK Từ đó ta được    , suy ra G  KT ∽ D  EH HD HA 2HE HE
Do đó suy ra GKT  DEH , mà ta có 0 GKT  OKG  90 và 0 DEH  EDH  90 nên OKG  EDH
Măt kh{c ta có EDH  OGF nên ta được OKG  OGF Ta có 2
OK.OS  OD và OD  OG nên ta được O  KG ∽ O  GS Nguyễn Công Lợi TÀI LIỆU TOÁN HỌC 35
Từ đó suy ra OKG  OGS , do đó ta được OGF  OGS nên GF  GS
Do đó ba điểm G, F, S thẳng h|ng nên BP//DF//CQ. [p dụng định lí Talets v| tính chất hai BP BS BS AJ BM AN BM BD PF
tiếp tuyến cắt nhau ta được   .  .    CQ CS AJ CS AM CN CN CD QF
Kết hợp với BPF  CQF nên B  PF ∽ C
 QF , từ đó ta được BFP  CFQ Mà ta có 0
BFP  BFD  CFQ  CFD  90 nên ta được BFD  CFD , suy ra FD là phân giác
của góc BFC . Mặt kh{c ta có FL cũng l| ph}n gi{c của góc BFC . Do đó FD v| FL trùng
nhau hay ba điểm F, D, L thẳng h|ng.
Ví dụ 29. Cho tam gi{c đều ABC nội tiếp đường tròn (O). Lấy điểm M bất lì trên cung AB
koong chứa điểm C. Đường thẳng AM cắt tiếp tuyến tại B, tiếp tuyến tại C với đường tròn
(O) v| đường thẳng BC lần lượt tại E, F, H. Chứng minh rằng giao điểm của c{c đường
tròn ngoại tiếp c{c tam gi{c MAO, MBF, MCF v| O, H l| c{c điểm thẳng h|ng Lời giải
Cách 1: Gọi N l| giao điểm F của BF v| CE. Kéo dại MN
cắt BC tại S. Khi đó ta có A ABE  ACB M và ACB  BAC E nên ta suy ra được N O ABE  BAC . Từ đó C H B S BE//AC. Mặt kh{c ta có EBC  ABE  ABC 0  BAC  ABC  120
Ho|n to|n tương tự ta được CF//AB v| 0 BCF  120
Hai tam giác ABE và FCA có B  AE ∽ C  FA và BEA  CAF nên A  BE ∽ F  CA AB BE BC BE Do đó ta được 2 
 AB.AC  BE.CF  BC  BE.CF   CF AC CF BC BC BE
Hai tam giác EBC và BCF có EBC  BCF và  nên E  BC ∽ B  CF CF BC
Do đó CEB  FBC hay NBC  CEB . Hai tam giác CBN và CEB có NBC  CEB và CBN chung nên C  BN ∽ C  EB . Suy ra 0
BNC  EBC  BNC  120 nên 0 0 BNE  180  BNC  60 Mặt kh{c ta lại có 0
BME  ACB  60 nên BNE  BME Nguyễn Công Lợi TÀI LIỆU TOÁN HỌC 36
Từ đó suy ra tứ gi{c BEMN nội tiếp đường tròn. Chứng minh tương tự ta được tứ gi{c
CFMN nội tiếp đường tròn. Từ đó ta có NEB  NMB và NBS  NEB nên NBS  NMB hay SBN  SMB
Hai tam giác SBN và SMB có BSN  MSB và SBN  SMB nên S  BN ∽ S  MB SB SN Từ đó ta được 2 
 SB  SM.SN . Ho|n to|n tương tự ta cũng được 2 SC  SM.SN SM SB
Từ đó ta được SB  SC hay S l| trung điểm của BC.
Dễ d|ng chứng minh được AB  BC  CA  R 3 . Tam gi{c OBC c}n tại O có OS l| đường R
OS cũng l| đường cao v| đường ph}n gi{c của tam gi{c BOC. Từ đó ta suy ra SO  và 2 3R SA  2 Từ đó ta được 2 SB  SO.SA , mà ta có 2
SB  SM.SN nên ta được SO.SA  SM.SN hay SN SO  . SA SM SN SO
Xét hai tam giác SON và SMA có  và MSA chung nên S  ON ∽ S  MA SA SM
Do đó ta được SON  SMA . Suy ra tứ gi{c AONM nội tiếp đường tròn. Mặt kh{c tứ gi{c
BNOC cũng nội tiếp đường tròn. Gọi N l| giao điểm của OH với đường tròn ngoại tiếp 1
tam gi{c OBC, gọi N l| giao điểm của DH với đường tròn ngoại tiếp tam gi{c OAM. Khi 2
đó HN .HO  BH.HC và HN .HO  HM.HA 1 2
Mà ta có HB.HC  HM.HA nên ta được HN .HO  HN .HO  HN  HN  N  N 1 2 1 2 1 2
Khi đó giao điểm của hai đường tròn ngoại tiếp hai tam gi{c OBC v| OAM l| N  N 1 2
M| N l| giao điểm của hai đường tròn ngoại tiếp hai tam gi{c OBC v| OAM nên N  N  N . 1 2
Mà N thuộc OH nên N cũng thuộc OH. Ta có N thuộc c{c đường tròn ngoại tiếp c{c tam 1
gi{c OBC, OAM, MCF nên N l| giao điểm của ba đường tròn đó. Từ đó ta được giao điểm
của c{c đường tròn ngoại tiếp c{c tam gi{c MAO, MBF, MCF v| O, H l| c{c điểm thẳng hàng. Nguyễn Công Lợi TÀI LIỆU TOÁN HỌC 37
Cách 2: Gọi N l| giao điểm F của BF v| CE. Kéo d|i MN
cắt BC tại S. Hai tiếp tuyến A
với đường tròn (O) tại B, C M E
cắt nhau ở P. Từ P kẻ đường N O
thẳng song song với BC cắt
FB tại Q. Chứng minh tương B S C H tự c{ch 1 ta được AB  E ∽ FC  A E  BC ∽ B  CF Q P C  BN ∽ C  EB S  BN ∽ S  MB
Từ đó ta được S l| trung điểm của BC. Ta có BCP  ABC nên CP//AB.
Tương tự ta cũng có AC//BP. Nên tứ gi{c ABPC l| hình bình h|nh. M| ta có AB  AC nên
tứ gi{c ABPC l| hình thoi. Do đó S l| trung điểm của AP. Điều n|y dẫn đến c{c điểm A, O, S, P thẳng h|ng. Ta có 0
OBP  OCP  90 nên c{c điểm B, C, O, P thuộc đường tròn đường kính OP.
Ta có EPQ  PBC và PBC  ACB nên 0 EPQ  ACB  60 BC FC AC FC
[p dụng định lí Talets cho c{c FPQ v| FPE ta được  ;  nên ta được PQ FP FE FP BC AC  PQ FE
Mà ta có AC  BC nên PQ  PE . Mặt kh{c ta có 0
EPQ  60 nên tam gi{c EPQ đều.
Ta có NBC  NQP và NBC  NEP nên NQP  NEP , do đó tứ gi{c ENPQ nội tiếp đường tròn. Từ đó ta được 0
BNP  BCP  60 , suy ra tứ gi{c BNCP nội tiếp đường tròn, nên N thuộc
đường tròn ngoại tiếp tam gi{c BCP. M| O cũng thuộc đường tròn ngoại tiếp tam gi{c
BCP nên năm điểm O, N, B, C, P cùng thuộc đường tròn đường kính OP. Do đó ta được 0 0
ONP  90  SNP  SNO  90 .
Tam giác HAP có HS l| đường cao cũng l| đường trung tuyến nên c}n tại H.
Ta có HPS  HAS và HAS  SNO nên HPS  SNO Nguyễn Công Lợi TÀI LIỆU TOÁN HỌC 38 Ta có 0 0
SHP  HPS  90 ;SNP  SNO  90 ; HPS  SNO nên ta được SHP  SNP
Từ đó suy ra tứ gi{c BNSP nội tiếp đường tròn. Suy ra 0 HNP  90 . Ta có 0
OHN  OHP  HNP  180 nên ba điểm O, H, N thẳng h|ng.
Tứ gi{c BNOC nội tiếp đường tròn nên 0 BNC  BOC  120 Mặt kh{c ta có 0 0
BNE  180  BNC  60 , m| ta lại có 0 BME  ACB  60
Từ đó ta được BNE  BME nên tứ gi{c BEMN nội tiếp đường tròn. Chứng minh tương tự
ta được tứ gi{c CFMN nội tiếp. Như vậy N thuộc c{c đường tròn ngoại tiếp c{c tam gi{c
MAO, MBE, MCF. Vậy giao điểm của c{c đường tròn ngoại tiếp c{c tam gi{c MAO, MBF,
MCF v| O, H l| c{c điểm thẳng h|ng.
Ví dụ 30. Cho tam gi{c ABC v| đường tròn (I) nội tiếp tam gi{c tiếp xúc với c{c cạnh AB,
BC, CA lần lượt tại F, D, E. Gọi M l| giao điểm của BC với đường ph}n gi{c trong của góc
BIC v| N l| giao điểm của EF với đường ph}n gi{c trong của góc EDF . Chứng minh rằng
ba điểm A, M, N thẳng h|ng.
Phân tích tìm lời giải
Nếu ABC c}n tại A thì hiển nhiên ba điểm A
A, M, N thẳng h|ng. Như vậy ta cần chứng minh
trong trường hợp tam gi{c ABC không c}n tại A P E
thì ba điểm A, M, N thẳng h|ng. Chú ý đến t}m I Q N F I
của đường tròn nội tiếp tam gi{c ABC ta có 0 ABC ACB 0 ABC ACB IMC  90   ; NDC  90   4 4 4 4 B M D C
nên ta suy ra được IM//DN hay IM//PN. Như vậy
để chứng minh được ba điểm A, M, N thẳng IM AP AN
h|ng thì ta cần chứng minh được   PN IA AM . Lời giải
+ Trường hợp tam gi{c ABC c}n tại A, khi đó hiển nhiên ba điểm A, M, N thẳng h|ng
+ Xét trường hợp tam gi{c ABC không c}n tại A, khi đó không mất tính tổng qu{t ta giả sử AB  AC.
Gọi P, Q theo thứ tự l| giao điểm của AI với (I) v| EF. Dễ thấy ba điểm D, N, P thẳng h|ng nên ta có Nguyễn Công Lợi TÀI LIỆU TOÁN HỌC 39 1 1
 IMC  MIB  MBI  BIC  MBI   0 180  IBC  ICB  MBI 2 2 1   0 ABC ACB ABC 0 ABC ACB  180      90   2  2 2  2 4 4   1 1  0 NDC NDE EDC FDE EDC 180            2 2 
 090 IBD  090 ICD   1   0 180  FDB  EDC  EDC 2 0 IBD ICD 0 ABC ACB  90    90   2 2 4 4
Do đó ta được IMC  NDC do đó ta được IM//ND. Do đó ta suy ra IM//PN (*). Để ý l|
ID  IP nên ta được MID  IDP  QPN . Do BC tiếp xúc với (I) tại D v| P l| điểm chính
giữa cung EF nên ta được 1 1 1 IDM  PDM  IDP  sdPED  IDP  sdPE  sdED  IDP 2 2 2 1 1
 sdPF  sdED  IDP  PNF  QPN  PQN 2 2 IM DI IM PI Do đó ta được I  DM ∽ P  QN, suy ra 
m| ta lại có ID  IP do đó  PN QP PN QP Mặt kh{c ta có 0
IAE  90 , EQ  IP và IE  IP nên ta được 2 2 IQ.IA  IE  IP QP IQ IP PA IP IA IM AP Do đó ta có  1   1     , nên ta được  (**) IP IP IA IA PQ AP PN IA
Từ (*) v| (**) ta được A, M, N thẳng h|ng.
Ví dụ 31. Cho tam gi{c nhọn ABC nội tiếp đường tròn (O). Gọi I l| điểm chính giữa cung
BC không chứa A. Trên AC lấy K kh{c C sao cho IK  IC. Đường thẳng BK cắt đường
tròn (O) tại D kh{c B. Trên DI lấy M sao cho CM song song với AD. Đường thẳng KM cắt
đường thẳng BC tại N. Đường tròn ngoại tiếp tam gi{c BKN cắt đường tròn (O) tại P kh{c
B. Chứng minh rằng PK đi qua trung điểm của đoạn thẳng AD.
Phân tích và lời giải
Do I l| điểm chính giữa cung BC v| theo giả J A
thiết ta được IB  IC  IK . Do đó ta được 0 0
IKD  180  IKB  180  IBK  CID
Cũng do I l| điểm chính giữa cung BC nên
DI l| tia ph}n gi{c của góc BDC . Do đó ta O K D được KID  CID M N C B
Xét hai tam gi{c KID v| CID có ID l| cạnh P
chung, KID  CID và IK  IC nên K  ID  C
 ID nên ta được DK  DC . Từ đó I Nguyễn Công Lợi TÀI LIỆU TOÁN HỌC 40
ta được AI l| đường trung trực của đoạn
thẳng BK. Gọi IJ l| đường kính của (O), ta có
AJ//DH v| DJ//AK. Nên tứ gi{c AJDK l|
hình bình h|nh. Do đó KJ đi qua trung điển của AD.
Như vậy để chứng minh PK đi qua trung điểm của AD ta chỉ cần chứng minh ba điểm J,
K, P thẳng h|ng l| được. Thật vậy, ta có IPK  IPB  BPK . Do DBC  DIC  DIK nên tứ
gi{c BKNP nội tiếp, suy ra BPK  BNK . Lại có BPI  BAI . Do đó ta được
IPK  BAI  BNK  BAI  NKC  NCK
Do ID l| đường trung trực của CK nên ta được
IPK  BAI  NKC  NCK  BAI  MCK  BCK
Lại có CM//AD nên ta được IPK  BAI  MCK  BCK  BAI  CAD  BCK
Do tứ gi{c ABCD nội tiếp nên IPK  BAI  CAD  BCK  BAI  CBD  BCK  IAK  AKB Do BK  AI nên ta được 0
IPK  90  IPJ . Từ đó ta được ba điểm P, K, J thẳng h|ng. Vậy
PK đi qua trung điểm của đoạn thẳng AD.
Ví dụ 32. Cho tứ gi{c ABCD có c{c cặp cạnh đối diện không song song với nhau, ngoại
tiếp đường tròn t}m O. Gọi M, N lần lượt l| trung điểm của AB, CD. Chứng minh rằng ba
điểm M, O, N thẳng h|ng khi v| chỉ khi OA.OC  OB.OD .
Phân tích tìm lời giải
Giả sử ba điểm M, O, N thẳng h|ng. Khi đó gọi R v| S lần lượt l| trung điểm của
AD v| BC thì ta được tứ gi{c MRNS l| hình bình h|nh nên MN v| SR cắt điểm O. Gọi giao
điểm của AB v| CD l| P khi đó tam gi{c MPN c}n tại P. Để chứng minh được OA OB
OA.OC  OB.OD ta cần chứng minh 
. Chú ý là NC  ND ta cần chứng minh được OD OC BO BM MO   OA MA OM và  
hay ta cần chứng minh được hai tam gi{c BMO v| OC ON NC OD NO DN
ONC đồng dạng, hai tam gi{c OMA v| DNO đồng dạng. Lời giải Nguyễn Công Lợi TÀI LIỆU TOÁN HỌC 41
+ Điều kiện cần: Giả sử ba điểm M, O, N P
thẳng h|ng. Khi đó gọi R v| S lần lượt l|
trung điểm của AD v| BC. Dễ d|ng C
chứng minh được tứ gi{c MRNS l| hình B
bình h|nh nên MN v| SR cắt nhau tại H N
trung điểm của mỗi đường, điểm đó M O K
chính l| điểm O. Gọi giao điểm của AB
v| CD l| P. Do đường tròn (O) nội tiếp A D
tứ gi{c ABCD nên đường tròn (O) nội
tiếp ta gi{c APD. Từ đó suy ra PO l|
ph}n gi{c của góc MPN , đồng thời PO l|
đường trung tuyến của tam gi{c MPN.
Do đó tam gi{c MPN c}n tại P. Mặt kh{c ta có 0
MBC  BPC  BCP  BPC  180  BCN hay ta được 0 MBC  BCN  BPC  180 1 1
Suy ra MBC  BCN   0 MPC  180  0  MBO  NCO  BPO  90 2 2 MBO  BPO  BOP Ta lại có 0   MBO  MOB  BPO  90 0 MOB  90  BOP 
Kết hợp c{c kết quả trên ta được MBO  NCO  MBO  MOB  NCO  MOB
Điều n|y dẫn đến hai tam gi{c BMO v| ONC đồng dạng với nhau nên ta được BO BM MO   OC ON NC
Chứng minh tương tự ta cũng có hai tam gi{c OMA v| DNO đồng dạng với nhau nên OA MA OM   OD NO DN OA OB
Để ý l| NC  ND nên ta được 
hay ta được OA.OC  OB.OD . OD OC
+ Điều kiện đủ: Giả sử ta có OA.OC  OB.OD . Khi đó để ý l| AOB  COD   0
180  OAB  OBA   0 180  OCD  ODC 1 0
 360  DAB  ABC  BCD  CDA 0  180 2 OA OA OD
Vẽ c{c hình bình h|nh OAKB v| OCHD. Khi đó ta có   AK OB OD Nguyễn Công Lợi TÀI LIỆU TOÁN HỌC 42
Từ đó hai tam gi{c OMB v| CNO đồng dạng nên MOB  OCN  OCB và CON  MBO  OBC Suy ra ta được 0
MOB  BOC  CON  OCB  BOC  OBC  180 .
Điều n|y có nghĩa l| điểm O nằm trên đường thẳng MN. Hay ta được ba điểm M, O, N thẳng h|ng.
Ví dụ 33. Cho tứ gi{c ABCD nội tiếp đường tròn (O). Lấy điểm P nằm trên đường thẳng
AC sao cho PB v| PD l| hai tiếp tuyến của đường tròn. Tiếp tuyến tại C cắt PD v| AD lần
lượt tại Q v| R. Gọi E l| giao điểm thứ hai cả AQ với đường tròn (O). Chứng minh rẳng ba điểm B, E, R thẳng h|ng.
Phân tích và lời giải
Gọi giao điểm của BE với AD l| R’, ta cần R R'
chứng minh R v| R’ trùng nhau.
Thật vậy, ta có APD  PAC và DPC chung nên P  AD ∽ P  DC. Lại có A
PAB  PBC và BPC chung nên ta được Q P  AB ∽ P  BC E O AD PA PA AB Từ đó ta được    , do CD PD PB BC P D C đó suy ra AD.BC  AB.CD.
Tương tự ta được AC.DE  AD.CE B
[p dụng định lí Ptoleme cho tứ gi{c ABCD nội tiếp ta được AB.CD  AD.BC  AC.BD 1
Từ đó suy ra AD.BC  AB.CD  AC.BD 2
[p dụng định lí Ptoleme cho tứ gi{c ACED nội tiếp ta được AC.DE  AD.CE  AE.CD 1 BD 2CD CD 2ED
Suy ra ta được AC.DE  AD.CE  AE.CD . Từ đó ta được  và  2 AB CA CA EA
Ho|n to|n tương tự như trên ta chứng minh được R  DC ∽ R  CA nên suy ra RD CD RC   . RC CA RA 2 2 2 RD RD.RA  RC   CD   2ED  Từ đó ta được           2 RA RA RA CA AE       R ' D ED R ' A EA Ta cũng có A  BR ' ∽ D  R ' và D  BR ' ∽ E  AR ' nên ta được  và  R ' B AB R ' B BD Nguyễn Công Lợi TÀI LIỆU TOÁN HỌC 43 2 R ' D ED.BD  2ED  RD R ' D Suy ra     . Từ đó ta được 
nên hai điểm R v| R’ trùng R ' A EA.AB AE   RA R ' A nhau.
Vậy ba điểm B, E, R thẳng h|ng
Ví dụ 34. Cho hai đường tròn O và O cắt nhau tại A v| B. C{c tiếp tuyến tại A v| B 2  1 
của đường tròn O cắt nhau tại K. Lấy điểm M trên đường tròn O không trùng với A 1  1 
v| B. Gọi P l| giao điểm thứ hai của MA với đường tròn O . Gọi C l| giao điểm thứ hai 2 
của MK với đường tròn O . Gọi Q l| giao điểm thứ hai của đường thẳng CA với đườn 1 
tròn O . Chứng minh rằng trung điểm của đoạn thẳng PQ nằm trên đường thẳng CM. 2  Lời giải
Không mất tính tổng qu{t, giả sử M thuộc
cung lớn AB , trường hợp M thuộc cung A
nhỏ AB được chứng minh tương tự. Gọi P
H l| giao điểm của PQ v| BC. Ta cần O1 K O2 C H
chứng minh được PH  QH . B
Do BK l| tiếp tuyến của đường tròn O Q 1 
v| KCM l| c{t tuyến của tương ứng nên M
ta dễ d|ng chứng minh được BC CK B  CK ∽ M  BK suy ra  . Hoàn BM BK AC CK to|n tương tự ta được  AM AK AC BC
M| ta lại có KA  KB , nên từ c{c tỉ số trên ta được 
. Ta lại có tứ gi{c AMBC nội AM BM
tiếp đường tròn O nên ta được AMB  BCQ  PMB  BCQ 1 
Mặt kh{c tứ gi{c AQB nội tiếp đường tròn O nên ta được BPM  AQB  BPM  CQB 2  MP BM Do đó ta được B  MP ∽ B  CQ   CQ BC
[p dụng định lí Menelaus cho tam gi{c APQ với ba điểm C, M, H thẳng h|ng ta được CA HQ MP HP AC MP BC BM . .  1   .  .  1  HP  HQ CQ HP MA HQ AM CQ BM BC
Hay H l| trung điểm của PQ. Ta có điều phải chứng minh.
Ví dụ 35. Cho tam gi{c ABC có trực t}m H. Đường ph}n gi{c ngo|i của góc BHC cắt cạnh
AB, AC lần lượt tại D v| E. Đường ph}n gi{c trong của góc BAC cắt đường tròn ngoại tiếp
tam gi{c ADE tại K. Chứng minh rằng K, H v| trung điểm của BC thẳng h|ng.
Phân tích tìm lời giải Nguyễn Công Lợi TÀI LIỆU TOÁN HỌC 44
Nhận thấy tam gi{c ADE c}n tại A. Gọi P l| giao điểm của KD v| HB, Q l| giao
điểm của KE v| HC. Khi đó tứ gi{c HPKQ l| hình bình h|nh. Để chứng minh H, K v|
trung điểm của BC thẳng h|ng ta cần chỉ ra được PQ song song với BC. Muốn vậy ta đi PB QC chứng minh  PH QH Lời giải
Trước hết ta chứng minh tam gi{c ADE c}n tại A. A
Thật vậy, vì HD l| ph}n gi{c ngo|i của góc BHC nên ta có C' 1   1 DHB HBC HCB   B' E  0 90 ABC  0 90 ACB       2 2  H 1 D Q  BAC P 2 K B C Do đó ta được 1 1 0 0
ADE  DBH  DHB  90  BAC  BAC  90  BAC 2 2 1 Tương tự ta cũng có 0
AED  90  BAC , từ đó suy ra ADE  AED hay tam giác ADE cân 2 tại E.
Mặt kh{c ta có AK l| ph}n gi{c của góc DAE nên cũng l| đường trung trực của đoạn DE,
từ đó suy ra AK l| đường kính của đường tròn ngoại tiếp tam gi{c ADE. Từ đó KD  AB ,
tương tự ta cũng có KE  AC .
Gọi P l| giao điểm của KD v| HB, Q l| giao điểm của KE v| HC. Ta có KP  AB và
HQ  AB , từ đó suy ra KP//HQ. Tương tự ta cũng được KQ//HP, do đo tứ gi{c KPKQ l| hình bình hành.
Gọi BB’ v| CC’ l| c{c đường cao của tam gi{c ABC, ta có DP//HC’ v| QE//HB’ PB DB QC EC
Nên theo định lí Talet ta có  ;  PH DC ' QH EB ' DB HB EC DB
Theo tính chất đường ph}n gi{c của tam gi{c ta có  ;  DC ' HC ' EB ' HB '
Vì B, C, B’, C’ cùng thuộc đường tròn đường kính BC nên ta được B  HC' ∽ C  HB' HB HC PB QC Từ đó ta được 
. Kết hợp với c{c kết quả trên ta được  , nên theo định HC ' HB ' PH QH
Talet ta được PQ song song với BC. Do HK đi qua trung điểm của PQ nên HK cũng đi qua trung điểm của BC.
Ví dụ 36. Cho tam gi{c ABC nhọn nội tiếp đường tròn (O) có trực t}m H, đường cao AF v|
M l| trung điểm của BC. Đường tròn đường kính AH cắt đường tròn O) tại Q kh{c A. Nguyễn Công Lợi TÀI LIỆU TOÁN HỌC 45
Đường tròn đường kính HQ cắt đường tròn (O) tại K kh{c Q. Đường thẳng KQ cắt dường
tròn ngoại tiếp tam gi{c KFM tại N kh{c K. Chứng minh rằng MN đi qua trung điểm của AQ.
Phân tích và lời giải
Gọi L v| R lần lượt l| t}m đường tròn N A
ngoại tiếp c{c tam gi{c KHQ v| KMF. Ta
cần chứng minh ba điểm K, L, R thẳng Q hàng. R
Thật vậy, gọi AE l| đường kính của đường L K
tròn (O) v| D đối xứng với H qua BC. Khi H O
đó dễ thấy tứ gi{c HBEC l| hình hình
h|nh nên ba điểm H, M, E thẳng h|ng. Lại X C B F M
có Q thuộc đường tròn đường kính AE nên ta được 0
AQF  90 . M| Q lại thuộc D E
đường tròn đường kính AH nên ta được 0 QAH  90 Từ đó 0
AQH  AQF  90 nên ba điểm Q, H,
E thẳng h|ng. Do vậy bốn điểm Q, H, M. E thẳng h|ng.
Gọi tiếp tuyến tại K v| H của đường tròn
ngoại tiếp tam gi{c KHQ cắt nhau tại X.
Khi đó ta có XKH  XHK  KQH (góc nội tiếp v| góc tạo bởi tia tiếp tuyến với d}y cung
cùng chắn một cung). M| ta lại có KQH  KAE ( hai góc nội tiếp cùng chắn cung KE của
đường tròn (O)). Từ đó ta có 0 0       0     0 KXH 180 2KHX 180 2KQH 2 90 KQH
2 90  KAE  2AEK  2KDH
Lại có XH  XK nên X l| t}m đường tròn ngoại tiếp tam gi{c KHD.
Do D v| H đối xứng nhau qua BC nên BC l| đường trung trực của DH, từ đó suy ra X
thuộc đường thẳng BC. Do XH l| tiếp tuyến của đường tròn đường kính QH nên ta có XH  QH
Xét tam gi{c XHM vuông tại H có đường cao HF nên ta có 2 2 XK  XH  XF.XM
Từ đó suy ra XK l| tiếp tuyến chung của hai đường tròn ngại tiếp tam gi{c KQH v| KFM
với tiếp điểm K l| giao điểm của hai đường tròn. Do đó ta suy ra hai đường tròn ngại tiếp
tam gi{c KQH v| KFM với tiếp xúc nhau K. Từ đó suy ra hai t}m đường tròn v| tiếp điểm Nguyễn Công Lợi TÀI LIỆU TOÁN HỌC 46
cùng nằm trên một đường thẳng hay ba điểm L, K, R thẳng h|ng. Gọi P l| trung điểm của
AQ, ta cần chứng minh ba điểm M, N, P thẳng h|ng.
Thật vậy, ta có bốn điểm Q, H, M, E thẳng h|ng, từ đó suy ra KQH  KAE nên hai tam
gi{c vuông KQH v| KAE đồng dạng. Từ đó suy ra hai tam gi{c KQA v| KHF đồng dạng
M| ta lại có KP v| KM lần lượt l| hai trung tuyến của hai tam gi{c KQA v| KHE nên ta
được hai tam gi{c KQP v| KHM đồng dạng, suy ra QPK  QMK và QKP  HKM .
Từ đó suy ra tứ gi{c QPMK nội tiếp đường tròn. Ta có biến đổi góc sau 0
CMN  QKF  QMK  LKM  MKF  KPQ  RMK  90  RMF 0 0 0
 90  PMK  RMK  90  RMF  180  BMP  CMP
Do đó ba điểm M, P, N thẳng h|ng hay MN đi qua trung điểm của AQ. Vậy ta có điều phải chứng minh.
Ví dụ 37. Cho hai đường tròn O và O cắt nhau tại hai điểm A v| B. Lấy điểm Q nằm 2  1 
trên đường tròn O . C{c đường thẳng AQ, BQ cắt đường tròn O lần lượt tại C, D. C{c 1  2 
tiếp tuyến tại A, B với đường tròn O cắt nhau tại P. Chứng minh rằng PQ đi qua trung 2  điểm của CD
Phân tích và lời giải
Gọi M l| giao điểm của AB v| O O , khi 1 2 Q
đó M l| trung điểm của AB. Để chứng A
minh PQ đi qua trung điểm của CD ta cần N M P chứng minh MQB  PQC . O2 O D 1 K C
Thật vậy, giả sử ta có MQB  PQC . Khi B
đó ta gọi K l| giao điểm của PQ v| CD.
Tứ gi{c ABCD nội tiếp đường tròn nên ta
có QBM  KCQ , kết hợp với MQB  PQC
ta suy ra được hai tam gi{c MQB v| KQC đồng dạng. MB QB Từ đó ta được 
. Cũng do tứ gi{c ABCD nội tiếp đường tròn nên ta có KC QC AB QB MA AB 1 Q  CD ∽ Q  BA do đó  . Từ đó ta được 
, m| ta lại có MA  AB nên ta CD QC KC CD 2 1
thu được KC  DC hay K l| trung điểm của DC. Do đó PQ đi qua trung điểm của CD. 2
Để b|i to{n sẽ được chứng minh ho|n tất ta cần chứng minh được MQB  PQC . Thật vậy,
gọi N l| giao điểm của PQ với đường tròn O . 2  Nguyễn Công Lợi TÀI LIỆU TOÁN HỌC 47
Do PA l| tiếp tuyến của đường tròn O nên ta suy ra được P  AN ∽ P  QA 2  PA AN PN PB BN PN Từ đó suy ra   . Tương tự ta cũng có   . PQ AQ PA PQ QB PB BN AN
M| ta lại có PA  PB nên từ c{c kết quả trên ta thu được   AQ.BN  BQ.AN . QB AQ
[p dụng định lí Ptoleme cho tứ ANBQ nội tiếp ta được AN.BQ  BN.AQ  AB.NQ 1 BQ QN
Do đó ta được NB.AQ  AN.BQ  AB.QB  BM.QN . Do đó  , kết hợp với 2 BM AN QBM  QNA ta được Q  BM ∽ Q
 AN . Từ đó suy ra MQB  PQC . Vậy b|i to{n được chứng minh.
B. CÁC BÀI TOÁN VỀ BA ĐƯỜNG ĐỒNG QUY
I. Một số phương pháp chứng minh ba đường đồng quy
Phương pháp 1: Chuyển bài toán chứng minh ba đường thẳng đồng quy về bài toán chứng
minh ba điểm thẳng hàng.
Phương pháp 2: Chứng minh ba đường thẳng là đường trung tuyến, ba đường phân giác, ba
đường cao, ba đường trung trực trong tam giác.
Phương pháp 3: Gọi giao điểm của hai đường thẳng là M và chứng minh đường thẳng còn
lại cũng đi qua điểm M.
Phương pháp 4: Sử dụng định lí Ceva.
Cho tam giác ABC. Các điểm A’, B’, C’ lần lượt thuộc các đường thẳng BC, CA, AB. Khi đó
A' B B 'C C ' A
ba đường thẳng AA’, BB’, CC’ đồng quy khi và chỉ khi . .  1 .
A'C B ' A C ' B Chứng minh
+ Điều kiện cần: Qua A kẻ đường thẳng song song với BC cắt đường thẳng BB’, CC’ tại M, N. B ' C BC C'A AN A ' B AM A'B B ' C C ' A AM BC AN Ta có  ;  ;  . Vậy ta có . .  . .  1 B ' A AM C ' B BC A ' C AN A ' C B ' A C ' B AN AM BC
+ Điều kiện đủ: Gọi I l| giao của BB’ v| CC’. Giải sử AI cắt BC tại A’’, suy ra A’’ cũng thuộc BC. A''B B ' C C ' A A'B B ' C C ' A
Theo định lý Ceva (phần thuận) ta có . .  1 mà . .  1 A '' C B ' A C ' B A ' C B ' A C ' B A'B A ' B Nên 
. Từ đó suy ra A '  A ' . Do đó AA’, BB’, CC’ đồng quy A ' C A ' C
II. Một số ví dụ minh họa Nguyễn Công Lợi TÀI LIỆU TOÁN HỌC 48
Ví dụ 1. Cho tam gi{c ABC có trực t}m H. Đường tròn (O) đi qua hai điểm B, C cắt AB,
AC lần lượt tại D, E D  B;E  C . Gọi K l| trực t}m tam gi{c ADE. Chứng minh rằng c{c
đường thẳng BE, CD, HK đồng quy.
Phân tích tìm lời giải
Để chứng minh BE, CD, HK đồng quy ta sẽ đi chứng minh ba điểm H, I, K thẳng
h|ng. Muốn vậy ta cần phải chứng minh được 0 BIK  BIH  180 Lời giải
Vẽ c{c hình bình h|nh HBMC v| IHCN. Khi đó ta có A
ADE  ECB và ADE  ABH  ACH . Từ đó ta được KDE  HCB K E
Ho|n to|n tương tự ta chứng minh được KED  HBC D
Xét hai tam giác KED và HCB có KDE  HCB và I KE DE N H
KED  HBC nên KED ∽ HCB , từ đó  . Lại O BH BC B C DE IE M
chứng minh được IDE ∽ IBC nên ta được  . BC IC IE KE KE Suy ra   IC KH CM Lại có
IEK  IED  KED  ICH  HBC  ICH  BCM  ICM IE KE
Xét hai tam giác IEK và ICM có IEK  ICM và 
nên ta được IEK ∽ ICM IC CM
Từ đó suy ra KIE  MIC . Ta có NMC  IBH  ABH  ABI  ACH  ACI  NIC
Do đó tứ gi{c MINC nội tiếp đường tròn nên ta được BIH  MNC  MIC  KIE Nên suy ra     0 BIK BIH
BIK KIE 180 . Từ đó ta được ba điểm H, I, K thẳng hang hay BE, CD, KH đồng quy.
Ví dụ 2. Cho hai đường tròn O và O không bằng nhau v| tiếp xúc ngo|i tại T. Kẻ 2  1 
O A tiếp xúc với O tại A v| O B tiếp xúc với O tại B sao cho c{c điểm A v| B cùng 1  2  1 2
nằm trên nửa mặt phẳng có bờ l| O O . Lấy H thuộc O A v| K thuộc O B sao cho BH và 1 2 1 2
AK cùng vuông góc với O O . Đường thẳng TK cắt đường tròn O tai điểm thứ hai l| F 2  1 2
v| TH cắt đường tròn O tại điểm thứ hai l| E. Biết EF cắt AB tại S. Chứng minh rằng ba 1 
đường thẳng O A, O B v| TS đồng quy tại một điểm. 1 2
Phân tích tìm lời giải Nguyễn Công Lợi TÀI LIỆU TOÁN HỌC 49
Nhận thấy tứ gi{c O O AB nội tiếp đường tròn nên ta suy ra được IT l| đường ph}n 1 2
gi{c của góc O IO . Để chứng minh được ba đường thẳng O A, O B v| TS đồng quy tại 1 2 1 2
một điểm.Ta đi chứng minh ba điểm T, I, S thẳng h|ng. Muốn vậy ta cần chứng minh SB IB được  . SA IA Lời giải
Gọi I l| giao điểm của O A và O B . Xét tứ 1 2 A F giác O O AB có 0
O AO  O BO  90 nên nội S 1 2 1 2 1 2 E B I
tiếp đường tròn. Do đó O I IB H K 1 O I.IA  O I.IB   nên ta có O1 T O2 1 2 O I IA 2 O T O B O I A  IO ∽ B  IO . Từ đó 1 1 1   , do 2 1 O T O A O I 2 2 2
đó IT l| đường ph}n gi{c của góc O IO 1 2
Dễ thấy O BH  O O B v| từ tứ gi{c O O AB nội tiếp nên ta có O O B  O AB 1 1 2 1 2 1 2 1 O B O A O T O A Nên ta được O  BH ∽ O  AB suy ra 1 1 1 1    1 1 O H O B O H O T 1 1 1 1 Từ đó ta được O  TH ∽ O
 AT nên ta được O AT  O TH  O ET . Do vậy tứ gi{c O TAE 1 1 1 1 1 1 nội tiếp được. Suy ra 0 0 0
EO A  ETA  180  O TH  O TA  180  O AT  O AT  90 1 1 2 1 2
Từ đó ta được EO // AO . Chứng minh tương tự ta được BO // FO . Tứ đó 1 2 1 2 EB O E O T IB O  BE ∽ O  FA và có A  IO ∽ B  IO nên ta suy ra 1 1    và 1 2 2 1 FA O A O T IA 2 2 O BE  O FA 1 2 SB EB IB
Mà ta có BO // FO nên ta được BE // FA. [p dụng định lí Talet ta được   , 1 2 SA FA IA
suy ra IS l| đường ph}n gi{c trong của góc AIB .Mà hai góc O IO và AIB đối đỉnh với 1 2
nhau nên ba điểm T, I, S thẳng h|ng. Do đó ta được ba đường thẳng O A, O B và TS 1 2 đồng quy tại điểm I.
Ví dụ 3. Cho tam gi{c ABC, gọi I l| t}m đường tròn nội tiếp tam gi{c ABC. C{c đường
thẳng AI, BI, CI cắt đường tròn ngoại tiếp tam gi{c ABC lần lượt tại D, E, K. Trên cạnh BC
lấy điểm M. Hai điểm P v| Q thỏa mãn MP song song với BE, CP vuông góc với CK, MQ
song song với CK v| BQ vuông góc với BE. Chứng minh PE, QK, DM đồng quy. Lời giải Nguyễn Công Lợi TÀI LIỆU TOÁN HỌC 50
Từ M kẻ đường thẳng song song với BE cắt JB
đường ph}n gi{c ngo|i của góc C , đó chính A E
l| điểm P, thỏa mãn CP  CK và MP song N
song với BE. Tương tự điểm Q thỏa mãn K Q O P
nằm trên đường ph}n gi{c ngo|i góc B nên I
BQ  BE v| MQ song song với CK. Đường B M C
thẳng AD v| BE cắt đường ph}n gi{c ngo|i
tại hai điểm J , J . Giả sử DM cắt đường A B D tròn ngoại tiếp A
 BC tại N. Khi đó ta có 1 DAE  A  B 1 0  90  C v| cũng có 2 2 1 1 JA 0
BCJ  90  C  BJ c  A Lại có MP B B 2 2 1
song song với BJ nên ta được MPC  A . B 2 1 1
Mặt kh{c ta có DNC  A nên ta được MPC  MNC  A suy ra tứ gi{c MNPC l| tứ gi{c 2 2
nội tiếp đường tròn. Do đó suy ra 0 BCJ  MNP  180 . B 1 1
Tứ gi{c BEND nội tiếp nên ta được 0 0
END  180  EBD  180  A  B 0  90  C 2 2 1 Từ 0 0
BCJ  90  C  END  MNP  BCJ  MNP  180 suy ra ba điểm P, N , E thẳng B B 2 hàng.
Chứng minh ho|n to|n tương tự ta cũng được Q, K, N thẳng h|ng. Vậy PE, QK, DM đồng quy.
Ví dụ 4. Cho đường tròn t}m O. Từ điểm M nằm ngo|i đường tròn vẽ tiếp tuyến MA,
MC(A, C l| tiếp điểm), B thuộc cung lớn AC sao cho MB nằm giữa MO v| MC. Tia MB cắt
đường tròn tại Q(Q kh{c B), cắt CA tại N. Gọi K l| điểm đối xứng với C qua B. Qua Q kẻ
đường thẳng song song với BC cắt CM tại H. Chứng minh rằng QH, AC, MK đồng quy. Bài giải
Gọi giao điểm của MO với AC C B
l| E. Gọi giao điểm của QH với N K H Q
BC l| I. Vì MA, MC l| hai tiếp I M O
tuyến tại A, C của đường tròn E tâm O nên MA  MC nên M
nằm trên đường trung trực của A
AC. Mặt kh{c OA  OCnên O Nguyễn Công Lợi TÀI LIỆU TOÁN HỌC 51
nằm trên đường trung trực của
AC. Vì vậy OM l| đường trung
trực của AC nên OM vuông góc với AC tại E.
Vì MC vuông góc với OC nên tam gi{c MCO vuông tại C v| OM  EC nên 2 MC  ME.MO
Mặt kh{c xét tam gi{c MQC v| MCB có CMQ là góc chung và MCQ  MBQ MQ MC Nên ta được M  QC ∽ M  CB do đó suy ra 2   MQ.MB  MC MC MB MQ ME Từ đó MQ.MB  ME.MO   . Suy ra M  QE ∽ M
 OB nên ta được MEQ  MBO MO MB
hay MEQ  QBO . Lại có tứ gi{c QEOB có MEQ  MBO nên tứ gi{c QEOB l| tứ gi{c nội tiếp
Do đó ta được BEO  BQO . M| tam gi{c BQO c}n tại O nên QBO  BQO  BEO  BQO
Từ đó ta được MEQ  BEO. Mà 0 0
QEC  MEQ  90 , BEO  CEB  90 nên QEC  CEB do đó
EC l| tia ph}n gi{c của QEB . Mà HA là phân giác ngoài QEB nên theo tính chất ph}n giác suy ra NQ EQ MQ EQ NQ MQ  ,    NB EB MB EB NB MB MQ MH HQ QI NQ Mặt kh{c QH//BC nên ta có   và QI//CB nên ta có  MB MC BC CB NB HQ QI Từ đó ta được 
nên Q l| trung điểm của HI. Trong tam gi{c MCB có HQ//BC nên BC CB MH QH  MH 2QH HI MH HI hay   nên 
. Kết hợp với MHI  MCK ta được MC BC MC 2BC CK MC CK M  HI ∽ M
 CK suy ra HMI  CMKnên hai tia MI, MK trùng nhau. Do đó M, I, K thẳng
h|ng hay QH, AC, MK đồng quy.
Ví dụ 5. Cho nửa đường tròn (O; R) đường kính AB. Trên nửa mặt phẳng bờ AB chứa nửa
đường tròn vẽ c{c tia Ax v| By vuông góc với AB. Lấy điểm M nằm trên nửa đường
tròn(M kh{c A, B). Tiếp tuyến tại M cắt Ax v| By lần lượt tại C v| D. Đường trong ngoại
tiếp tam gi{c OCD cắt nửa đường tròn (O) tại E v| F. Chứng minh rằng AD, BC v| EF đồng tại một điểm.
Phân tích và lời giải
Gọi N l| giao điểm của AD v| BC. Ta đi chứng minh rằng ba điểm E, F, N thẳng h|ng. Nguyễn Công Lợi TÀI LIỆU TOÁN HỌC 52
Dễ thấy AC//BD, AC  CM, BD  DM và 0 COD  90 . J
Gọi I l| trung điểm của CD, H l| giao điểm của MN
với AB, K v| S lần lượt l| giao điểm của EF với OM D
và OI. Trong tam giác NBO có AC//BD nên ta có I M CN AC CN CM    . Từ đó suy ra MN//BD//AC. C MN BD MB MD N K S E F MN DM BN HN Ta có    nên ta được MN  NH . AC CD BC AC A H O B
Vẽ đường kính OJ của đường tròn (I) đường kính
CD. Hai đường tròn (O) v| (I) cắt nhau tại E v| F.
Suy ra OI l| đường trung trực của EF, nên ta được OI  EF , do đó EF//AB.
Tam gi{c EOJ vuông tại E có ES l| đường cao nên 2 OE  OS.OJ . 1 Từ đó ta được 2 2
2OS.OI  OM  OS.OI  OM . 2 OS OK Mà ta có O  SK ∽ O  MI nên ta được   OS.OI  OM.OK . OM OI 1 1 Do đó ta được 2
OM.OK  OM  OK  OM hay K l| trung điểm của MO. 2 2
M| ta lại có EF//AB, do đó EF đi qua N. Vậy ba điểm E, F, N thẳng h|ng.
Ví dụ 6. Cho tam gi{c ABC nội tiếp đường tròn (O). Đường tròn t}m A cắt BC tại D, E v|
cắt đường tròn (O) tại G, H sao cho D nằm giữa hai điểm B, E v| tia AB nằm giữa hai tia
AC, AG. Đường tròn ngoại tiếp c{c tam gi{c BDG v| CEH cắt lần lượt AB, AC tại K, L
kh{c B, C. Chứng minh rằng GK v| HL cắt nhau tại một điểm trên OA.
Trích đề thi Olympic Toán Quốc tế - IMO 2015
Phân tích và lời giải
Gọi giao điểm của GK với LH l| X. Ta cần A chứng minh X thuộc OA.
Thật vậy, dễ thấy AG v| AH l| b{n kính đường X L H
tròn t}m A nên ta được AG  AH , đồng thời G
v| H thuộc đường tròn (O) nên ta có OG  OH K O G
từ đó suy ra AO l| đường trung trực của GH. B D E C
Ta chỉ cần chứng minh X thuộc đường trung
trực của GH l| b|i to{n được chứng minh.
Thật vậy, dễ thấy tứ gi{c HCBG nội tiếp đường tròn (O) nên ta được 0 GHC  180  GBD v| tứ Nguyễn Công Lợi TÀI LIỆU TOÁN HỌC 53
gi{c GHED nội tiếp đường tròn tâm A nên ta được 0 GDE  180  GHE
Khi đó ta có biến đổi sau 0
EHC  GHC  GHE  180  GBD  GDB  BGD
Do tứ gi{c KGBD nội tiếp nên ta có KGD  KBD v| do tứ gi{c HGDE nội tiếp đường tròn
tâm A nên ta có HGD  HEC . Lại có GBA  HCA (hai go{c nội tiếp chắn hai cung bằng nhau) và EHC  BGD
Lại do tứ gi{c GDEH nội tiếp nên EHG  GDB .Khi đó ta có biến đổi sau 0
XGH  XGD  HGD  KBD  HEC  180  GBA  BGD  BDG  HEC 0
 180  HCA  EHC  EHG  HEC  ACB  GHE  XHE  GHE  XHG
Do đó ta được tam gi{c XGH c}n tại X hay X tuộc đường trung trực của GH. Vậy X thuộc
AO nên GK v| HL cắt nhau tại một điểm trên OA.
Ví dụ 7. Cho tam gi{c nhọn ABC có c{c đường cao BD v| CE cắt nhau tại H. Gọi M, N lần
lượt l| trung điểm của c{c đoạn thẳng AH, BC. Chứng minh rằng MN v| c{c đường ph}n
gi{c của c{c góc ABD; ACE đồng quy tại một điểm.
Phân tích tìm lời giải
Gọi K l| giao điểm của c{c đường ph}n gi{c của c{c góc ABD; ACE khi đó KBD  KCD . Lại thấy 0
BEC  BKC  BDC  90 . Nên c{c điểm B, E, K, D, C cùng nằm trên
đường tròn t}m N. M| ta lại có ECK  KCD nên điểm K nằm chính giữa cung DE của
đường tròn t}m N đường kính BC. M| ta lại thấy c{c điểm A, E, H, D nằm trên đường
tròn t}m M. Nên hai đường tròn t}m M v| t}m N cắt nhau tại D v| E. Như vậy MN đi qua
điểm chính giữa cung DE của đường tròn t}m N hay MN đi qua điểm K. Vậy MN v| c{c
đường ph}n gi{c của c{c góc ABD; ACE đồng quy tại một điểm. Lời giải
Gọi K l| giao điểm của c{c đường ph}n gi{c của A
ABD và ACE . Dễ thấy ABH  ACH do cùng phụ với góc BAC. M K D
M| BK v| CK l| ph}n gi{c của c{c góc ABD; ACE ABH ACH E nên ta được KBD    KCD . Từ đó ta 2 2 được B N C
KBC  KCB  DBC  KBD  KCB 0  DBC  KCD  KCB  90 Nguyễn Công Lợi TÀI LIỆU TOÁN HỌC 54 Suy ra 0 BKC  90 nên ta được 0
BEC  BKC  BDC  90 . Từ đó suy ra c{c điểm B, E, K, D, C
cùng nằm trên đường tròn t}m N. M| ta lại có ECK  KCD nên điểm K nằm chính giữa
cung DE của đường tròn t}m N đường kính BC. Mặt kh{c ta lại có 0 AEH  ADH  90 nên
c{c điểm A, E, H, D nằm trên đường tròn t}m M. Như vậy hai đường tròn t}m M v| t}m
N cắt nhau tại D v| E. Như vậy MN đi qua điểm chính giữa cung DE của đường tròn t}m
N hay MN đi qua điểm K. Vậy MN v| c{c đường ph}n gi{c của c{c góc ABD; ACE đồng quy tại một điểm.
Ví dụ 8. Chọn s{u điểm trên c{c cạnh của một tam gi{c đều ABC sao cho A ; A trên BC, 1 2
B ; B trên CA, C ; C trên AB. Những điểm n|y l| đỉnh của một lục gi{c lồi 1 2 1 2
A A B B C C với c{c cạnh bằng nhau. 1 2 1 2 1 2
Chứng minh rằng c{c đường thẳng A B , B C và C A đồng qui. 1 2 1 2 1 2 Lời giải
Bên trong tam gi{c đều ABC lấy điểm M sao A
cho tứ gi{c C C A M l| hình thoi. Khi đó 1 2 1
C A  A M  MC . Điều n|y dẫn đến tam gi{c 2 1 1 1 B2
A MA đều. Nên A M  A M. Suy ra 1 2 1 2 C1
MA B B cũng l| hình thoi. Do đó MA  MB . 2 1 2 2 2
Từ đó MA  MA  MB  MC . Như vậy tứ gi{c M B 1 2 2 1 1 1 M A A B C C
l| tứ gi{c nội tiếp. Suy ra 2 M2 1 2 2 1 1 0
A C A  A MA  30 . Ho|n to|n tương tự ta 1 1 2 1 2 2 B A A C 1 1 2 được 0 C A B  C MB  30 . 1 1 2 1 2 2
Trong tam gi{c đều ABC lấy c{c điểm M và M sao cho c{c tứ gi{c B M C C và B M B C 1 2 2 1 1 2 1 2 2 1
l| c{c hình thoi. Chứng minh ho|n to|n tương tự ta được hai tứ gi{c A B B C và A B C C 1 1 2 2 2 1 1 2 nội tiếp. Từ đó ta được 0
A B C  B A B  B C A  C B C  30 . Xét tam 1 1 2 1 1 2 1 1 2 1 1 2
giác A B C có A B , B C , C A l| c{c đường ph}n gi{c trong nên chúng đồng quy. Vậy ta 1 1 1 1 2 1 2 1 2
có điều phải chứng minh.
Ví dụ 9. Cho điểm C thay đổi trên nửa đường tròn đường kính AB  2R A  C, B  C .
Gọi H l| hình chiếu vuông góc của C lên AB. Gọi I v| J lần lượt l| t}m đường tròn nội tiếp
tam gi{c ACH v| BCH. C{c đường thẳng CI v| CJ cắt AB lần lượt tại M v| N. Chứng minh
rằng c{c đường thẳng MJ, NI, CH đồng quy.
Phân tích tìm lời giải Nguyễn Công Lợi TÀI LIỆU TOÁN HỌC 55
Để chứng minh ba đường thẳng MJ, NI, CH đồng quy ta đi chứng minh MJ, NI,
CH l| ba đường cao của tam gi{c CMN. Muốn vậy ta cần chứng minh được 0
CIN  CJM  90 . Điều n|y sẽ được khẳng định khi ta chứng minh được c{c tứ gi{c CIHN
v| CMHJ nội tiếp đường tròn Lời giải
Vì I, J l| t}m đường tròn nội tiếp tam gi{c C 1
ACH và BCH nên ICH  ACH và 2 1 JCH  BCH . Ta có 2 J I 1
MCN  MCH  NCH  ACH  BCH 0  45 2 A M H N B
Lại có HI l| đường ph}n gi{c của góc AHC 1 1 nên suy ra 0 0 MHI  AHC  90  45 2 2
Từ đó ta được MCN  MHI nên tứ gi{c CIHN nội tiếp nên suy ra được 0 CIN  CHN  90 .
Do đó ta được NI vuông góc với CM. Chứng minh tương tự ta được tứ gi{c CJHM nội tiếp suy ra 0
CJM  CHM  90 nên MJ vuông góc với CN. Trong tam gi{c CMN có c{c đường
cao CH, MJ, NI chúng đồng quy tại một điểm.
Ví dụ 10. Cho hai đường tròn O và O ' cắt nhau tại A v| B. Giả sử CD v| EF l| hai tiếp
tuyến chung ngo|i của hai đường tròn n|y C,E  O;D,F  O' v| điểm A gần CD hơn.
Gọi  l| đường thẳng qua A tiếp xúc với đường tròn ngoại tiếp tam gi{c AEF v|  là 1 2
đường thẳng qua B tiếp xúc với đường tròn ngoại tiếp tam gi{c BCD. Chứng minh rằng
c{c đường thẳng  ,  , CD, EF đồng quy. 1 2
Phân tích tìm lời giải
Gọi M l| điểm đối xứng với B qua EF. Khi đó ta có EMF  EBF nên 0
EMF  EAF  180 nên tứ gi{c MEAF nội tiếp đường tròn. Gọi N l| giao điểm của c{c tiếp
tuyến tại A, M của đường tròn ngoại tiếp tứ gi{c MEAF. Ta sẽ chứng minh c{c đường
thẳng  ,  , CD, EF. Muốn vậy ta cần chứng minh được ba điểm E, F, N thẳng h|ng. Khi 1 2
đó N thuộc đường tròn ngoại tiếp tam gi{c ABF. Do đó N thuộc đường trung trực của
đoạn thẳng AB nên N thuộc đường thẳng OO’. Chứng minh tương tự ta có  v| CD cắt 2
nhau tại một điểm N’ thuộc đường thẳng OO’. Do tính chất đối xứng CD v| EF cắt nhau
tại một điểm thuộc đường thẳng OO’, do đó hai điểm N v| N’ trùng nhau. Vậy c{c đường
thẳng CD, EF,  và  đồng quy tại N. 1 2 Nguyễn Công Lợi TÀI LIỆU TOÁN HỌC 56 Lời giải
Gọi M l| điểm đối xứng với B C
qua EF. Khi đó ta có EMF  EBF . 1 D A Mà ta có
EBF  EAF  EBF  EAB  FAB O O' N 0
 EBF  BEF  BFE  180 B F Do đó ta được 0 EMF  EAF  180 E M
nên tứ gi{c MEAF nội tiếp
đường tròn. Gọi N l| giao điểm
của c{c tiếp tuyến tại A, M của
đường tròn ngoại tiếp tứ gi{c MEAF.
Ta cần chứng minh ba điểm N, E, F thẳng h|ng. Thật vậy, gọi F’ l| giao điểm thứ hai của
đường tròn ngoại tiếp tứ gi{c MEAF với NE. Khi đó dễ thấy hai tam gi{c NAE v| NF’A AE NA đồng dạng nên  AF ' NF ' ME MN NA AE ME Ho|n to|n tương tự ta có  
. Từ kết quả trên ta được  nên MF ' NF ' NF ' AF ' MF ' AF ' AE  . MF ' ME
Gọi I l| giao điểm của AB v| EF, ta có 2 2
IE  IA.IB  IF nên IE  IF EB IF IE BF IF EB BF Ta lại có I  EB ∽ I  AE nên ta được   . Tương tự ta có  nên  . EA IA IA AF IA EA AF ME MF AE AF AF ' AE Do vậy ta có  hay  . Kết hợp với 
ta được hai điểm F v| F’ AE AF ME MF MF ' ME
trùng nhau. Do đó ba điểm N, E, F thẳng h|ng. Ta có N thuộc đường tròn ngoại tiếp tam
gi{c ABF. Do đó N thuộc đường trung trực của đoạn thẳng AB, suy ra N thuộc đường
thẳng OO’. Chứng minh tương tự ta có  và CD cắt nhau tại một điểm N’ thuộc đường 2
thẳng OO’. Do tính chất đối xứng CD v| EF cắt nhau tại một điểm thuộc đường thẳng
OO’, do đó hai điểm N v| N’ trùng nhau. Vậy c{c đường thẳng CD, EF,  và  đồng 1 2 quy tại N.
Ví dụ 11. Cho lục gi{c ABCDEF có c{c cặp cạnh đối diện song song với nhau. Gọi M, N, P,
Q, R, S lần lượt l| trung điểm của AB, BC, CD, DE, EF, FA. Chứng minh rằng c{c đường
thẳng MQ, NR, PS đồng quy.
Phân tích tìm lời giải Nguyễn Công Lợi TÀI LIỆU TOÁN HỌC 57
Nhận thấy nếu lục gi{c ABCDEF có AD, BE, CF đồng quy tai một điểm O khi đó
hiển nhiên đường thẳng MQ, NR, PS đồng quy. Để ý l| khi AD, BE, CF không đồng quy
thì chúng cắt nhau theo đôi một tạo th|nh một tam gi{c nằm trong lục gi{c. Khi đó gọi O
l| giao điểm của MQ v| NR(điểm O nằm trong tam gi{c XYZ) v| ta cần chứng minh PS đi
qua điểm O. Chú ý đến c{c trung điểm ta được S  S và S  S nên S  S . OQD OQE XQD XQE OXE OXD Tương tự ta được S  S nên S  S
. Cũng tương tự như trên thì ta được OXA OXB OAD OBE S  S . Do đó nên ta được S  S
. Lại để ý l| khi điểm O không thuộc PS thì ta OBE OCF OAD OCF thấy S  S
. Nên ta sử dụng phép phản chứng để chứng minh tiếp bài toán. OAD OCF Lời giải
Ta xét c{c trường hợp có thể xẩy ra như sau: A M B
+ Trường hợp 1: Nếu AD, BE, CF đồng quy S N
tại một điểm O. Khi đó theo định lí Talets ta F C có O OB AB 2BM BM R P    OE ED 2EQ EQ E Q D
Mặt kh{c ta có BM//EQ v| B, O, E thẳng
h|ng nên suy ra M, O, Q hay MQ đi qua
điểm O. Ho|n to|n tương tự ta cũng có NR,
SP đi qua điểm O. Vậy MQ, NR, PS đồng quy tại O.
+ Trường hợp 2: Nếu AD, BE, CF không đồng quy. Khi đó chứng cắt nhau tạo th|nh tam giác XYZ.
Lập luận tương tự trường hợp 1 ta MQ, NR, PS A M B
lần lượt đi qua X, Y, Z. Gọi O l| giao điểm của N S
MQ v| NR(điểm O nằm trong tam gi{c XYZ). C Z Y F O Ta có S  S và S  S nên ta suy ra X OQD OQE XQD XQE P được S  S . Tương tự ta được S  S R OXE OXD OXA OXB E Q D nên ta được S  S . Cũng tương tự ta OAD OBE chứng minh được S  S nên ta được OBE OCF S  S OAD OCF
Giả sử điểm O không thuộc đường thẳng SZP, không mất tính tổng qu{t ta xem điểm A,
O, C cùng nằm một phía so với SP. Khi đó ta được S  S m| ta lại có S  S nên OASX OFSZ ZAS ZSF suy ra S  S OAZ OFZ Tương tự ta cũng có S  S mà S  S nên ta được S  S PDZ OPCZ OPD OCP ODZ OCZ Nguyễn Công Lợi TÀI LIỆU TOÁN HỌC 58 Do đó ta được S  S
, điều nay m}u thuẫn với S  S
. Suy ra điểm O phải thuộc OAD OCF OAD OCF
đường thẳng SP hay SP đi qua O. Vậy ba đường thẳng MQ, NR, PS đồng quy tại O.
Ví dụ 12. Cho tam gi{c nhọn ABC nội tiếp đường tròn O . Gọi A , B ,C và A , B ,C lần 1 1 1 2 2 2
lượt l| ch}n đường cao của tam gi{c ABC kẻ từ A, B, C v| c{c điểm đối xứng với A , B ,C 1 1 1
qua trung điểm của BC, CA, AB. Gọi A , B ,C lần lượt l| giao điểm thứ hai của c{c đường 3 3 3 tròn ngoại tiếp A  B C , B  A C và C
 A B với đường tròn (O). Chứng minh rằng 2 2 2 2 2 2 A A , B B ,C C đồng quy. 1 3 1 3 1 3
Phân tích và lời giải A A I A' A3 C1 B1 B2 C1 C G 2 O B2 C A1 B A2 B A C 1 M B3 C3
Ta sẽ chứng minh A A , B B ,C C cùng đi qua trọng t}m G của tam gi{c ABC. Gọi M l| 1 3 1 3 1 3
trung điểm của BC, A’ l| điểm đối xứng với A qua trung trực của BC. Ta sẽ chứng minh
A’ trùng với A . Thật vậy, gọi I l| giao điểm của AA’ v| trung trực của BC. 3
Ta có A, A đối xứng nhau qua trung trực của BC nên
CA  BA ,CA  BA  C  AA  B  A A
 . Suy ra ACA  ABA nên ta được A’ thuộc đường AC AC tròn (O). Mà A  BB ∽ A  CC nên suy ra 1 
. Do A, B và C ,C cùng đối xứng nhau 1 1 AB AB 1 2 1
qua trung điểm của AB nên BC  AC . Chứng minh ho|n to|n tương tự ta được 2 1 B C  AB . 2 1 BC AC AC BA Ta có 2 1   
C BA  B CA nên ta được C  BA ∽ B  CA CB AB AB CA . M| ta lại có 2 2 2 2 2 1 Từ đó suy ra A C  B  A B
 C  AC A  AB A nên tứ gi{c AC B A l| tứ gi{c nội tiếp. 2 2 2 2 2 2
Suy ra A  A . Gọi G l| giao điểm của AM v| A A . Lại có AA // MI và AI // MA từ đó 3 1 3 1 1 1 MA 1
ta được tứ gi{c AIMA l| hình bình h|nh. Do đó 1 AI  MA  AA   . Vì MA // 1 1 3 2 AA 2 1 3 Nguyễn Công Lợi TÀI LIỆU TOÁN HỌC 59 MG MA 1
AA nên theo định lí Talets ta được 1 
 . Suy ra G l| trọng t}m tam gi{c ABC. 3 GA AA 2 3
Vậy A A đi qua G. Tương tự B B ,C C đi qua G. Suy ra A A , B B ,C C đồng quy tại G. 1 3 1 3 1 3 1 3 1 3 1 3
Ví dụ 13. Cho tam gi{c ABC nội tiếp đường tròn (O). Một đường tròn K tiếp xúc với 1 
c{c cạnh AB, AC v| tiếp xúc trong với đường tròn (O) lần lượt tại M , N , P . C{c điểm 1 1 1
M , N , P và M , N , P đượcx{c định một c{ch tương tự. Chứng minh rằng c{c đoạn thẳng 2 2 2 3 3 3
M N , M N , M N cắt nhau tại trung điểm của mỗi đoạn. 1 1 2 2 3 3
Phân tích và lời giải
Gọi E, F lần lượt l| giao điểm của AP , CP với 1 1 A
đường tròn K . Gọi D l| giao điểm của N P với 1  1 1 D
đường tròn (O). Ta sẽ chứng minh D l| điểm chính E giữa cung nhỏ AC N M 1 1
Thật vậy, gọi P x l| tiếp tuyến của đườngtròn (O) I 1 K1 tại P , khi đó ta có 1 B C
NAE  CAP  CAx  FP X  FN P F 1 1 1 1 P
Ta lại có tứ gi{c EN FP nội tiếp nên N EA  N FP . 1 x 1 1 1 1 1 Từ đó ta được A  EN ∽ N  FP nên 1 1 1 FP N  AN E  AP N 1 1 1 1 1
Do đó P N l| ph}n gi{c của APC nên D l| điểm 1 1 1 chính giữa cung nhỏ AC
Từ đó ta suy ra BD l| ph}n gi{c của góc ABC . Gọi I l| giao diểm của M N v| BD. Khi đó 1 1
ta suy ra được IM P  N M P  N P x  DP x  DBP  IBP . Suy ra tứ gi{c IM BP nên ta 1 1 1 1 1 1 1 1 1 1 1 1
được P IB  P M B m| ta lại có P M B  P N M . Do đó P IB  P N M  DIP  DN I nên 1 1 1 1 1 1 1 1 1 1 1 1 1 1 DI DP D  IP ∽ D  N I suy ra 1 2 
 DI  DN .DP . Ta cũng có DAN  CPN  DPN nên 1 1 1 1 DN DI 1 1 1 1 1 1 DA DN D  AN ∽ D  P A suy ra 1 2 
 DA  DN .DP . Do đó ta được AD  DI nên tam giác 1 1 1 1 DP DA 1 ADI c}n tại D ABC ABC
Suy ra DIA  DAI  ABI  IAB  DAN  N AI   IAB   N AI 1 1 1 2 2
Do đó ta được IAB  N AI nên AI l| ph}n gi{c của góc BAC . Vậy I l| giao điểm hai đường 1
ph}n gi{c trong của tam gi{c ABC nên I l| t}m đường tròn nội tiếp tam gi{c ABC. Dễ thấy Nguyễn Công Lợi TÀI LIỆU TOÁN HỌC 60 A
 M N c}n tại A v| AI l| ph}n gi{c của góc BAC nên AI l| đường trung tuyến của M N . 1 1 1 1
Nên I l| trung điểm của M N . Ho|n to|n tương tự ta cũng được I l| trung điểm của 1 1
M N , M N . Vậy c{c đoạn thẳng M N , M N , M N cắt nhau tại trung điểm của mỗi đoạn. 2 2 3 3 1 1 2 2 3 3
Ví dụ 14. Cho tam gi{c ABC v| c{c điểm A ; B ;C lần lượt nằm trên cạnh BC, CA, AB. Gọi 1 1 1
G l| trọng t}m tam gi{c ABC. Gọi G ;G ;G lần lượt l| trọng t}m c{c tam gi{c a b c
AB C ; BC A ; CA B . Chứng minh rằng c{c đường thẳng AG ; BG ;CG đồng quy khi và 1 1 1 1 1 1 a b c
chỉ khi AA ; BB ; CC đồng quy. 1 1 1
Phân tích và lời giải
Theo định lí Ceva thì ba đường thẳng A
AA ; BB ; CC đồng quy khi v| chỉ khi 1 1 1 AC BA CB 1 1 1 . .  1. BC CA AB 1 1 1 Ga
Gọi giao điểm tương ứng của AG ; BG ;CG với B a b c C 2 2 A B 3 1
BC, CA, AB lần lượt l| A ; B ;C . Khi đó ba đường C 2 2 2 1
thẳng AG ; BG ;CG đồng quy khi v| chỉ khi G G c a b c b CA BC AB 2 2 2 . .  1 . B A A C 1 2 BA AC CB 2 2 2
Như vậy ta cần chứng minh được CA BC AB CA BC AB 2 2 2 2 2 2 . .  . .  1 BA AC CB BA AC CB 2 2 2 2 2 2
Thật vậy, gọi A l| trung điểm của B C , gọi h ; h lần lượt l| đường cao của c{c tam gi{c 3 1 1 1 2 S h .AC AA .AC
AA C và AA B tương ứng hạ từ A ; A . Khi đó ta có AA C 2 1 2   . Tương tự 2 3 1 2 3 S h .AB AA .AB AA B 2 1 3 1 3 1 S AA .AB S S AA .AC AA .AB ta cũng có AA B AA C AA B 2 2  . Từ đó 2 2 2 2 :  : hay S AA .AC S S AA .AB AA .AC AA C 3 1 AA B AA C 3 1 3 1 3 1 3 1 3 1 S S AA C AA C AC AC 2 3 1 1 .  . S S AB AB AA B AA B 1 3 1 2 S AC AC
Mặt kh{c do A l| trung điểm của B C nên S  S .Từ đó suy ra AA C 2 1  . . 3 1 1 AA B AA C 3 1 3 1 S AB AB AA B 1 2 S A C A C AC AC M| ta cũng có AA C 2 2  . Do đó ta được 2 1  . . S A B A B AB AB AA B 2 2 1 2 BC CB CB AB BA BA
Chứng minh ho|n to|n tương tự thì ta được 2 1 2 1  . ;  . AC CA CA CB BC BC 2 1 2 1 CA BC AB CA BC AB
Nh}n theo vế c{c đẳng thức trên thì ta được 2 2 2 2 2 2 . .  . .  1. BA AC CB BA AC CB 2 2 2 2 2 2 Nguyễn Công Lợi TÀI LIỆU TOÁN HỌC 61
Suy ra c{c đường thẳng AA ; BB ;CC đồng quy, điều n|y dẫn đến ba đường thẳng 2 2 2
AG ; BG ;CG đồng quy. Vậy c{c đường thẳng AG ; BG ;CG đồng quy khi v| chỉ khi a b c a b c AA ; BB ; CC đồng quy 1 1 1
Ví dụ 15. Cho tam gi{c ABC v| D l| một điểm bất kì không thuộc c{c đường thẳng AB v|
AC. Trên đoan thẳng AD lấy điểm M(M kh{c A, D). Gọi I một điểm thuộc đoạn AD(I kh{c
A, D v| kh{c giao điểm của AD với BC). Gọi giao điểm của BI, CI lần lượt với AC, AB l| E,
F. Gọi giao điểm của DE, DF lần lượt với CM, BM l| K, H. Chứng minh rằng c{c đường
thẳng CM, AD, BK đồng quy.
Phân tích tìm lời giải
Trên cơ sở hình vẽ v| giả thiết ta thấy có nhiều bộ ba điểm thẳng h|ng nên theo định DM FA HB DM EA KC
lí Menelaus ta sẽ lập được c{c đẳng thức về tỉ số như . .  1 và . .  1 DA FB HM DA EC KM EC FA KC HM . Từ đó ta thu được .  .
. Ta lại có ba đường thẳng AG, BE, CF đồng quy EA FB KM HB GB EC FA GC EC FA GC KM HM nên . .
 1 . Từ đó ta rút ra được  . nên  . hay GC EA FB GB EA FB GB KC HB GB KC HM . .
 1. M| theo định lí Ceva thì hiển nhiên c{c đường thẳng CM, AD, BK đồng GC KM HB quy. Lời giải
Gọi G l| giao điểm của AD v| BC. [p dụng định lí A
Menelaus cho tam gi{c AMB với bộ ba điểm F, H, D DM FA HB M E thẳng h|ng ta được . .  1. F DA FB HM I K H
[p dụng định lí Menelaus cho tam gi{c AMC với bộ
ba điểm K, K, D thẳng h|ng ta được G B C DM EA KC . .  1. Từ đó ta được DA EC KM DM FA HB . . EC FA KC HM D DA FB HM  1  .  . . DM EA KC EA FB KM HB . . DA EC KM
Ta lại có ba đường thẳng AG, BE, CF đồng quy nên theo định lí Ceva ta được GB EC FA . .  1 GC EA FB GC EC FA EC FA KC HM Từ đó ta được  .
, kết hợp với hệ thức .  . GB EA FB EA FB KM HB GC KM HM GB KC HM Suy ra  . hay ta được . .  1. GB KC HB GC KM HB
Như vậy theo định lí Ceva thì c{c đường thẳng CH, BK, MG đồng quy. Nguyễn Công Lợi TÀI LIỆU TOÁN HỌC 62
Ví dụ 16. Cho tam giác ABC có AB  AC . Đường ph}n gi{c của góc BAC cắt BC tại V. Gọi
D l| ch}n đường vuông góc hạ từ A xuống BC. Đường tròn ngoại tiếp tam gi{c DAV cắt
AB, AC lần lượt tại F, E. Chứng minh rằng ba đường thẳng AD, BE, CF đồng quy tại một điểm.
Phân tích tìm lời giải
Để chứng minh AD, BE, CF đồng quy tại một điểm ta cần chỉ ra được DB EC FA DB EC EA . .
 1 hay ta cần chứng minh . 
. Chú ý là EA  FA nên để kết thúc DC EA FB DC FB FA DB FB
chứng minh ta cần phải chỉ ra được  DC EC Lời giải Ta có 0
ADV  90 nên tam gi{c ADV nội tiếp A
đường tròn đường kính AV. Từ đó suy ra 0
AFV  AEV  90 nên ta được 0 BFV  90 . Xét hai
tam giác VFB và ABD có ABD là góc chung và E VFB  ADB nên A  DB ∽ V
 FB . Từ đó ta được F DB AB FB VB    . Chứng minh ho|n to|n B D V C FB VB DB AB EC VC tương tự ta được  . DC AB VB VC
Lại có AV l| đường ph}n gi{c của tam gi{c ABC nên ta được  . Kết hợp c{c kết AB AC FB EC DB FB quả trên ta được  hay ta được 
. Xét hai tam giác AFV và AEV có DB DC DC EC 0
AFV  AEV  90 , AV chung và FAV  EAV . Từ đó A  FV  A
 EV nên suy ra FA  EA . DB FB DB EC Do đó   .  1. DC EC DC FB DB EC FA Từ đó ta được . .
 1 . Như vậy theo định lí Ceva thì ba đường thẳng AD, BE, CF DC EA FB đồng quy.
Ví dụ 17. Cho tam gi{c nhọn ABC nội tiếp đường tròn (O; R) có H l| trực t}m. Chứng
minh rằng tồn tại c{c điểm D, E , F lần lượt trên c{c cạnh BC, CA, AB sao cho
OD  HD  OE  HE  OF  HF v| ba đường thẳng AD, BE, CF đồng quy. Lời giải Nguyễn Công Lợi TÀI LIỆU TOÁN HỌC 63
Gọi L l| giao điểm của AH với đường tròn (O). Gọi A
D l| giao điểm của OL v| BC. Nối D với H, khi đó ta P
chứng minh được HD  DL . Từ đó ta được Q F E
OD  DL  OD  DH  OL  R O
Tương tự ta được OE  EH  OF  HF  R . Nối OB, H
OC v| BL thì ta được BOC  2BAC và B K D C 0 BOC  180  2OBC . Do đó ta được L 0 0
2OBC  180  2BAC  OBC  90  BAC V| ta lại có 0
CBL  CAL  90  ACB. Từ đó ta được     0   0 OBL OBC CBL 90 BAC 90  ACB  ABC .
Vì OB  L  R nên ta được OLB  OBL  ABC . Do đó ta được 0 0
BLO  180  2ABC  BOD  180  2ABC .
Như vậy trong tam gi{c BOD có 0
BOD  180  2ABC . Ho|n to|n tương tự ta được trong BD OD tam giác COD có 0
COD  180  2ACB . Theo định lí sin ta có  và sin BOD sin OBD CD OD  sin COD sin OCD OD OD BD CD
Mà ta có OCD  OBD nên ta được  . Từ đó ta được  sin OBD sin OCD sin BOD sin COD sin  0 180  2ABC BD sin BOD  Hay ta được   CD sin COD sin  0 180  2ACB sin  0 180  2ACB sin  0 180  2BAC EC FA 
Chứng minh ho|n to|n tương tự ta được   ;  . EA 0   FB sin 180 2BAC sin  0 180  2ABC sin  0 180  2ABC sin 0 180  2ACB sin  0 180  2BAC BD EC FA  Do đó ta được . .   . sin  . . 1 CD EA FB 0 180  2ACB sin  0 180  2BAC sin  0 180  2ABC
Từ đó theo định lí Ceva ta được AD, BE, CF đồng quy.
Ví dụ 18. Cho tam gi{c nhọn ABC có I, O l|n lượt l| t}m đường tròn nội tiếp v| ngoại tiếp
tam gi{c. Đường thẳng AI cắt đường tròn (O) tại điểm thứ hai l| D kh{c A. Gọi E v| F l| 1
hai điểm lấn lượt nằm trên cạnh BC v| cung BDC sao cho BAF  CFE  BAC . Gọi G l| 2
trung điểm của EF. Chứng minh rằng GD, EI cắt nhau tại một điểm nằm trên đường tròn (O). Nguyễn Công Lợi TÀI LIỆU TOÁN HỌC 64 Lời giải
Gọi giao điểm thứ hai của EI với đường tròn (O) l| A X
X, giao điểm của AD v| BC l| L. Gọi G’ v| T lần lượt
l| giao điểm của DX cới IF v| AF. Đường thẳng AF
cắt đường tròn (O) tại điểm thứ hai l| K. Ta có T O I
BAK  CAE nên ta được BK  CE suy ra KE//BC. G G' B
Lại có BAD  DAC suy ra BAD  BAK  DAC  CAE F L C
hay ta được KAD  DAE nên IAT  IXT . Từ đó suy K E D
ra tứ gi{c ITXA nội tiếp đường tròn. Do đó ITA  IXA  EXA  EKA . TF IL
Từ đó ta được IT//KE//BC, nên suy ra  . AT AI IL CL
Vì CI l| đường ph}n gi{c của tam gi{c ACL nên  . AI AC CL DC Chứng minh được C  LD ∽ A  CD nên ta được  . AC AD 1 1 1
Ta có ICD  ICB  BCD  ACB  BAC  sdABC; IDC  sdAC 2 4 2 1 1 1 Từ đó ta được 0 0
CID  180  ICD  IDC  180  sdABC  sdAC  sdACB 4 2 4 CD ID
Từ đó suy ra ICD  DIC nên tam gi{c ICD c}n tại D, suy ra DI  DC   AD AD TF ID TF AD
Kết hợp c{c kết quả trên ta thu được   .  1. TA AD TA ID
[p dung định lí Menelaus cho tam gi{c AIF với ba điểm T, D, G’ thẳng h|ng ta được TF AD IG ' . .  1 . TA DI G ' F TF AD G ' I Kết hợp với .  1 ta được
 1  G ' I  G ' F nên G’ l| trung điểm của IF. Đo đó TA ID G ' F
hai điểm G v| G’ trùng nhau. Vậy GD, EI cắt nhau tại một điểm nằm trên đường tròn (O). TF ID
Nhận xét: Bài toán này có đường giải quyết theo một hướng khác. Để chứng minh ta TA AD TF  TA ID  AD TA AF
có thể quy về chứng minh    . TA AD AD ID  AD
Ví dụ 19. Cho tam gi{c ABC nhọn nội tiếp đường tròn (O), có đường cao AD. Đường tròn
(A) bất kì có t}m A. Gọi E v| F l| hai điểm trên đường tròn (A) sao cho E, F đối xứng với
nhau qua AD v| tia AE nằm giữa hai tia AB, AF. Đường tròn (A) cắt đường tròn (O) tai G
v| H sao cho tia AB nằm giữa hai tia AG v| AC. Giao điểm của CE, BF với đường tròn (O) Nguyễn Công Lợi TÀI LIỆU TOÁN HỌC 65
lần lượt l| P, Q kh{c E, F. Đường tròn ngoại tiếp tam gi{c BPG v| CQH lần lượt cắt BA,
CA tại K, L kh{c B, C. Chứng minh rằng GK v| HL cắt nhau tại một điểm trên AO.
Phân tích tìm lời giải
Gọi X l| giao điểm của GK v| HL. Dễ thấy AG v| AH l| b{n kính đường tròn t}m
A nên ta được AG  AH , đồng thời G v| H thuộc đường tròn (O) nên ta có OG  OH từ đó
suy ra AO l| đường trung trực của GH. Ta có EF//BC nên ta được EFB  FBC . Lại có
EPQ  EFB do đó ta được QPE  FBC nên tứ gi{c PQCB nội tiếp. Lại có 0 GHC  180  GBC
và GHE  GFE nên ECH  BGF . Do tứ gi{c GPEH nội tiếp nên HGP  HEC, mà PGK  PBK từ đó
HGX  HEC  GBF  GBA  PBF . Tương tự
GHX  GFB  HCE  HCA  QCE . Như vậy ta cần chỉ ra HEC  GBF  GFB  ECH thì ta sẽ
có HGX  GHX v| như vậy HX  GX hay X thuộc đừng trung trực của HG hay X thuộc
OA, điều n|y có nghĩa l| GK v| HL cắt nhau tại X trên AO. Lời giải
Gọi X l| giao điểm của GK v| HL. Ta cần A chứng minh X thuộc OA.
Thật vậy, dễ thấy AG v| AH l| b{n kính H
đường tròn t}m A nên ta được AG  AH , L
đồng thời G v| H thuộc đường tròn (O) O K G Q
nên ta có OG  OH từ đó suy ra AO l| P E F X
đường trung trực của GH. Ta có EF//BC B C
nên ta được EFB  FBC . M| ta có tứ gi{c
PEFQ nội tiếp đường tròn t}m A nên ta
lại có EPQ  EFB . Do đó ta được
QPE  FBC , từ đó suy ra tứ gi{c PQCB nội tiếp đường tròn.
Do tứ gi{c GBCH nội tiếp nên ta lại có 0
GHC  180  GBC . Lại có GHE  GFE (hai góc nội
tiếp của đường tròn t}m A). Ta có biến đổi góc sau 0 0
ECH  GHC  GHE  180  GBC  GHE  180  GBP  PBC  GFE  BGP  GPB   0 180  BPC  BCP 0
 180  GPE  BGP  FEC  BGP  PGF  BGF
Do tứ gi{c GPEH nội tiếp nên HGP  HEC, m| lại có PGK  PBK  GBF  GBA  PBF Nguyễn Công Lợi TÀI LIỆU TOÁN HỌC 66
Từ đó ta được HGX  HGP  PGK  HEC  PBK  HEC  GBF  GBA  PBF (1)
Ho|n to|n tương tự ta được GHX  GFB  HCE  HCA  QCE (2)
Lại có GBA  HCA, PBF  QCE và BGF  CHE nên ta được GBF  GFB  HEC  EHC
Hay ta được HEC  GBF  GFB  ECH (3)
Từ (1), (2) v| (3) nên ta được HGX  GHX nên ta được HX  GX . Từ đó suy ra X thuộc
đừng trung trực của HG hay X thuộc OA. Vậy GK v| HL cắt nhau tại một điểm trên AO.
Ví dụ 20. Cho một đường tròn với hai d}y AB v| CD không song song. Đường vuông góc
với AB kẻ từ A cắt đường vuông góc với CD kẻ từ C v| từ D lần lượt tại M v| P. Đường
vuông góc với AB kẻ từ B cắt đường vuông góc với CD kẻ từ C v| từ D lần lượt tại Q v|
N. Chứng minh rằng c{c đường thẳng AD, BC, MN đồng quy v| c{c đường thẳng AC, BD, PQ đồng quy. Lời giải
Gọi giao điểm của MN với BC v| AD lần lượt D l| I v| J. Ta có: N B 1 Q MC.BC. sin MCB MI S MC. cos BCD I MBC 2    IN S 1 C NBC NB. cos ABC NB.BC. sin NBC 2 M A P MJ AM. cos BAD Tương tự ta có  JN ND. cos ADC AM MC MI MJ
Nhưng do BCD  BAD, ABC  ADC  A  MC ∽ D  NB nên  suy ra  ND NB IN JN
hay I v| J trùng nhau. Do đó ta được AD, BC, MN đồng quy. Tương tự ta được AC, BD, PQ đồng quy
Ví dụ 21. Cho tam gi{c ABC. Đường tròn nội tiếp t}m I của tam gi{c tiếp xúc với c{c cạnh
BC, CA, AB lần lượt tại D, E, F. Gọi M l| trung điểm của cạnh BC. Chứng minh rằng c{c
đường thẳng AM, DI, EF đồng quy tại một điểm.
Phân tích và lời giải Nguyễn Công Lợi TÀI LIỆU TOÁN HỌC 67
Để chứng minh AM, EF, ID đồng quy ta gọi L, N A
lần lượt l| giao điểm của ID với EF v| của AL với
BC v| chứng minh hai điểm M v| N trùng nhau,
tức N l| trung điểm BC. Trong tam gi{c ABC ta E L có F 1 I S .AB.AN.sin BAN sin BAN NB AB ABN 2    . NC S 1 AC sin CAN ACN .AC.AN.sin CAN B N≡M C 2 D Trong tam giác AEF ta có sin BAN 2S 2S S FL AFL AEL AFL  :   AF.AL AE.AL S EL sin CAN AEL 1 S .IF.IL.sin FIL sin FIL sin B FL AC Trong tam giác IEF ta có FIL 2      EL S 1 AB sin EIL sin C EIL .IE.IL.sin EIL 2 NB Từ đó ta có được
 1 nên suy ra N l| trung điểm của BC hay hai điểm M v| N trùng NC
nhau. Vậy c{c đường thẳng AM, EF, ID đồng quy.
Ví dụ 22. B|i to{n . Cho ADBC l| tứ gi{c nội tiếp được được. Gọi P, Q, R lần lượt l| hình
chiếu của D trên c{c đường thẳng BC, CA v| AB. Chứng minh rằng PQ  QR khi v| chỉ
khi c{c đường ph}n gi{c của c{c góc ABC và ADC cắt nhau tại một điểm nằm trên đường thẳng AC
Đề thi Olympic Toán Quốc tế - IMO 2003 Lời giải
Trước hết ta chứng minh AQR  CQP. B
Thật vậy, dễ d|ng nhận thấy c{c tứ gi{c AQRC,
CDPQ v| BPDR nội tiếp được. Do đó ta có
AQR  ADR và CQP  CDP. M| ta lại có P Q
RDP  ADC . Do đó ta được ADR  CDP . Từ đó ta A E C được AQR  CQP R
+ Điều kiện cần:Khi c{c đường ph}n gi{c của c{c góc D
ABC và ADC cắt nhau tại một điểm nằm trên đường
thẳng AC. Ta cần chứng minh thì PQ  QR . M
Thật vậy, gọi E l| giao điểm trên đường thẳng AC Nguyễn Công Lợi TÀI LIỆU TOÁN HỌC 68
của hai đường ph}n gi{c của c{c góc ABC và ADC .
Trên tia đối của tia DA l}y điểm M sao cho AM  AD. [p dụng tính chất đường ph}n gi{c AB DA EA AB DM
cho hai tam gi{c ABC v| ADC ta được   . Từ đó ta được  . BC DC EC BC DC
Mặt kh{c do tứ gi{c ABCD nội tiếp nên ta có ABC  MDC . Từ đó suy ra A  BC ∽ M  DC
nên ta được ABC  MCD và CAB  CMD . Lại có tứ gi{c AQDR nội tiếp nên DQR  DAQ QR DR DR
và RDQ  CMD  CAB . Từ đó D  RQ ∽ M  AC suy ra   AC MA 2AD RP DR Dễ thấy A  DC ∽ A  PR nên  . Từ đó ta được AC DA QR PR 1   QR  PR  PQ  QR AC 2AC 2
+ Điều kiện đủ: Khi PQ  QR , ta cần chứng minh c{c đường ph}n gi{c của c{c góc ABC
và ADC cắt nhau tại một điểm nằm trên đường thẳng AC.
Thật vậy, gọi E l| giao điểm của đường ph}n gi{c của góc ADC với AC, ta đi chứng minh
DE cũng l| đường ph}n gi{c của góc ABC. Theo tính chất đường ph}n gi{c trong tam AD EA DM giác ta có   . CD EC DC RP DR Dễ thấy A  DC ∽ A  PR nên ta được  . AC DA 2QR 2DR QR DR DR
Mà PQ  QR nên ta được     suy ra D  RQ ∽ M  AC AC 2AD AC 2AD MA
Suy ra RDQ  CMD  CAB , lại có ABC  CDM nên A  BC ∽ M  DC AB DM AB EA Do đó ta được  . Từ đó suy ra 
. Do đó BE l| ph}n gi{c của góc ABC . BC DC BC EC
Ví dụ 23. Cho tam gi{c nhọn ABC v| lấy c{c điểm P, Q nằm trên cạnh BC sao cho
PAB  BCA và CAQ  ABC . C{c điểm M v| N lần lượt trên AP v| AQ sao cho P l| trung
điểm của AM v| Q l| trung điểm của AN. Chứng minh rằng giao điểm của BM v| CN
nằm trên đường tròn ngoại tiếp tam gi{c ABC.
Trích đề thi Olympic Toán Quốc tế - IMO 2014
Phân tích tìm lời giải
Gọi giao điểm của MB v| CN l| D. Để chứng minh giao điểm của BM v| CN nằm
trên đường tròn ngoại tiếp tam gi{c ABC ta đi chứng minh tứ gi{c ABDC nội tiếp đường
tròn. Muốn vậy ta cần chứng minh được tứ gi{c BQDN nội tiếp đường tròn để có BAC  BQN  BDN hoặc 0
BDC  A  180 . Với hai kết quả trên ta đều có tứ gi{c ABDC nội tiếp đường tròn. Nguyễn Công Lợi TÀI LIỆU TOÁN HỌC 69 Lời giải
Cách 1: Gọi giao điểm của MB v| CN l| D. ta cần A
chứng minh tứ gi{c ABDC nội tiếp đường tròn. Thật
vậy, theo giả thiết ta có AP  PM và AQ  QN do đó
ta được MN//PQ. Cũng theo giả thiết PAB  BCA và CAQ  ABC Q ta có APQ  AQP  A P B C
Mặt kh{c dễ d|ng chứng minh được A  BP ∽ C  AQ PB QA PB QA Suy ra    , do đó B  PM ∽ N  QC D PA QC PM QC N M
nên CNQ  MBP . Do đó tứ gi{c BQDN nội tiếp
đường tròn, suy ra BAC  BQN  BDN . Vậy tứ gi{c
ABDC nội tiếp đường tròn, ta có điều phải chứng minh. AB BC
Cách 2: Do CAQ  ABC nên ta được A  BC ∽ Q  AC , suy ra  . AQ AC
Gọi E v| F lần lượt l| điểm nằm trên tia đối của tia AB v| AC sao cho AE  AB, AF  AC. BC AB 2AB EB Do    nên ta được E  BC ∽ N
 AC , do đó ta được BEC  ANC AC AQ 2AQ NA
Tương tự ta được CFB  AMB .Mặt kh{c ta thấy tứ gi{c BFEC l| hình bình h|nh. Gọi I l|
trung điểm của BC thì ta có AI//CE//FB. Suy ra BAC  BAI  IAC  BEC  CFB.
Do đó ta được DNM  ANM  ANC  A  BEC . Tương tự ta được DMN  A  CFB .
Ta có DNM  DMN  2A  BEC  BFC  A , suy ra 0 BDC  A  180
Do đó tứ gi{c ABDC nội tiếp đường tròn.
Ví dụ 24. Cho tam giác ABC có AB  AC v| H l| trực t}m. Ta kí hiệu (O) l| đường trong
ngoại tiếp tam gi{c ABC. Gọi M l| trung điểm của BC. Đường thẳng AM cắt đường trong
(O) tại điểm thứ hai l| N. Đường tròn đường kính AM cắt đường tròn (O) tại P. Chướng
minh rẳng c{c đường thẳng AP, BC, OH đồng quy khi v| chỉ khi HA  HN
Trích đề thi chọn đội tuyển IMO Pháp 2012 Lời giải Nguyễn Công Lợi TÀI LIỆU TOÁN HỌC 70
Trước hết ta sẽ chứng minh ba điểm P, H, M A thẳng h|ng.
Thật vậy, Gọi V l| điểm đối xứng với A qua
O. Khi đó ta có BH  AC và VC  AC. Từ đó U O P
suy ra BH//VC. Ho|n to|n tương tự ta được H
CH//VB nên tứ gi{c BHCV l| hình bình hành. J C B M
Do M l| trung điểm của BC nên M l| ta có HV nhận M l| trung điểm. V N
Do P thuộc đường tròn (O) nên ta được 0 APV  90
Gọi U l| giao điểm thứ hai của HV với đường tròn (O) khi đó ta có 0 AUV  ABV  90 . Suy ra
hai điểm U v| P trùng nhau. Do đó ba điểm P, M, H thẳng hàng.
Gọi J l| giao điểm của AP v| BC. Do AH v| MP l| hai đường cao của tam gi{c AIM, do đó
H l| trực t}m của tam gi{c AJM. Từ đó suy ra OH đi qua J khi v| chỉ khi OH vuông góc với AN hay OA  ON .
Điều n|y chỉ ra rằng OH  AN khi v| chỉ khi OH l| đường trung trực của AN, có nghĩa l| HA  AN .
Từ đó ta có điều khải chứng minh.
Ví dụ 25. Cho đường tròn (O) ngoại tiếp tam gi{c ABC. Một đường tròn đi qua A v| C cắt
cạnh BC v| BA lần lượt tại D v| E. C{c đường thẳng AD v| CE cắt đường tròn lần thứ hai
tại G v| H tương ứng. C{c tiếp tuyến với đường tròn (O) tại A v| C cắt DE lần lượt tại L v|
M. Chứng minh rằng c{c đường thẳng LH v| MG cắt nhau tại một điểm trên đường tròn (O).
Phân tích tìm lời giải
Gọi giao điểm của đường thẳng MG với đường tròn (O) l| P kh{c G. Chú ý rằng
tứ gi{c BPED nội tiếp đường tròn nên P l| giao điểm thứ hai của hai đường tròn ngoại tiếp
tam gi{c ABC v| BDE. Do đó để chứng minh c{c đường thẳng LH v| MG cắt nhau tại một
điểm trên đường tròn (O) ta sẽ chứng minh LH cũng đi qua P. Muốn vậy ta gọi Q l| giao
điểm của đường thẳng LH với đường tròn (O) v| ta cần chứng minh Q l| giao điểm tứ hai
của hai đường tròn ngoại tiếp hai tam gi{c ABC v| BDE. Lời giải Nguyễn Công Lợi TÀI LIỆU TOÁN HỌC 71
Gọi giao điểm của đường thẳng MG với đường tròn (O) l| P kh{c G. Do MCD  CAE và
MDC  CAE nên ta được MCD  MDC , suy ra tam gi{c MCD c}n tại M. Do đó ta được MC  MD .
Vì MC l| tiếp tuyến tại C của đường tròn (O) nên M ta được 2 2 MC  MD  MG.MP C
Suy ra MD l| tiếp tuyến của đường tròn ngoại G
tiếp tam gi{c DGP. Từ đó ta được DGP  EDP . D
Gọi (O’)l| đường tròn ngoại tiếp tam gi{c BDE. O Khi đó B
+ Nếu B v| P trùng nhau thì với BGD  BDE ta A E P
được hai đường tròn (O) v| (O’) tiếp xúc trong tại H L B.
+ Nếu B v| P kh{c nhau, khi đó ta có hai trường hợp sau
 Trường hợp P nằm cùng phía với G so với
đường thẳng BC, khi đó ta được 0
EDP  ABP  180 Mà ta có 0 DGP  ABP  180 nên
tứ gi{c BPED nội tiếp đường tròn.
Từ đó suy ra P l| giao điểm của hai đường tròn (O) v| (O’)(kh{c D v| kh{c E)
 Trường hợp P nằm kh{c phía với G so với đường thẳng BC, khi đó ta có
EDP  DPG  AGP  ABP  EBP
Suy ra tứ gi{c PBDE nội tiếp đường tròn nên P cũng l| giao điểm của hai đường tròn (O) v| (O’)
Ho|n to|n tương tự ta gọi Q l| giao điểm của đường thẳng LH với đường tròn (O). ta
cũng chứng minh được Q l| giao điểm của hai đường tròn (O) v| (O’) ( kh{c D v| kh{c E).
Do đó hai điểm P v| Q trùng nhau,
Vậy hai đường thẳng LH v| MG cắt nhau tại một điểm trên đường tròn (O).
Ví dụ 26. Cho tam giác ABC với I l| t}m nội tiếp v| (O) l| đường tròn ngoại tiếp tam gi{c.
Đường thẳng AI cắt (O) tại điểm thứ hai l| D (kh{c A). Gọi E l| một điểm trên cung BDC 1
của đường tròn (O) v| F l| một điểm nằm trên đoạn BC sao cho BAF  CAE  BAC . 2
Chứng minh rằng đường thẳng EI v| DG cùng đi qua một điểm trên nằm trên (O).
Trích đề thi Olympic Toán Quốc tế - IMO 2010
Phân tích và lời giải Nguyễn Công Lợi TÀI LIỆU TOÁN HỌC 72
Gọi K l| giao điểm của tia AD với đường tròn t}m D, A
b{n kính DI. Do D l| giao điểm của ph}n gi{c góc
BAC với (O) nên D chính l| trung điểm cung BC . Dễ I O
thấy khi đó DI  DB  DC. G
Suy ra bốn điểm B, I, C, K cùng thuộc (D) v| D l| C B F
trung điểm IK. Theo giả thiết vì G l| trung điểm IF nên E D
DG l| đường trung bình của tam gi{c IFK. Nên ta
được GD // FK  IDG  IKF . Ta sẽ chứng minh K A  EI ∽ A  KF . 1
Thật vậy, do BAF  CAE  BAC và ABF  AEC nên 2 AB AF suy ra A  BF ∽ A  EC   . AE AC AB AI
Hơn nữa lại có ABI  IBC  AKC và IAB  IAC nên A  BI ∽ A  KC   . AK AC AE AK
So s{nh hai đẳng thức ở trên ta được AE.AF  AB.AC  AI.AK   . AI AF
Mặt kh{c IAE  IAC  CAE  IAB  BAF  IAF nên A  EI ∽ A  KF  IEA  IKF (2)
Từ (1) v| (2) ta được IDG  IEA  ADG  AEI , tức l| c{c góc n|y cùng chắn một cung trên
đường tròn (O) đã cho. Từ đó suy ra c{c đường thẳng EI v| DG cùng đi qua một điểm nằm trên (O).
Ví dụ 27. Cho tam gi{c ABC với c{c góc nhọn v| AB  AC. Đường tròn đường kính BC
t}m O cắt AB, AC lần lượt tại M v| N. Ph}n gi{c trong của góc BAC và MON gặp nhau tại
R. Chứng minh rằng hai đường tròn ngoại tiếp của hai tam gi{c BMR v| CNR có một điểm chung nằm trên BC
Trích đề thi Olympic Toán Quốc tế - IMO 2004 Lời giải
Gọi J, K lần lượt l| t}m đường tròn ngoại tiếp c{c tam gi{c BMR, CNR. Với đường tròn
tâm O ta có MNA  MBC (cùng chắn cung MC). Tam gi{c CON c}n tại O nên ta được
ONC  OCN , nên ta được ONM  BAC . Điều n|y chứng tỏ ON l| tiếp tuyến của đường
tròn t}m I ngoại tiếp tam gi{c AMN. Ho|n to|n tương tự ta được OM l| tiếp tuyến với
đường tròn t}m I ngoại tiếp tam gi{c AMN Nguyễn Công Lợi TÀI LIỆU TOÁN HỌC 73
Từ đó suy ra OM  ON nên tam giác MON cân. Theo giả thiết OR l| ph}n gi{c MON nên
nó l| trung trực của MN. Do đó ta được tam gi{c MRN c}n, suy ra MR, NR l| hai đường
ph}n gi{c kh{c của MON.
Điều n|y dẫn đến R l| t}m đường tròn nội A
tiếp tam gi{c MON v| R cũng l| điểm chính
giữa cung MN thuộc đường tròn t}m I. Hai
đường tròn t}m J v| K ngoại tiếp với hai M I
tam gi{c BMR v| CNR có điểm chung thứ N hai là P. R
Ta có AMB l| c{t tuyến chung của hai J
đường tròn t}m J v| O, ANC l| c{t tuyến K
chung của hai đường tròn t}m K v| O, O B P C
nên ARP l| c{t tuyến chung của hai đường
tròn tâm J và K. Vậy A, R, P thẳng h|ng. Do
đó c{c tứ gi{c PRMB v| PRNC nội tiếp. Từ đó ta được 1 0
RPB  RMB  RPB  ABC  BAC  180 2 1 Và 0
RPC  RNC  RPC  ACB  BAC  180 . Từ đó ta được o
BPC  RPB  RPC  180 . Do 2
đó P phải nằm trên BC. Vậy hai đường tròn ngoại tiếp hai tam gi{c BMR v| CNR có một điểm chung trên BC
Ví dụ 28. Gọi I l| t}m đường tròn nội tiếp của tam gi{c ABC v|  l| đường tròn ngoại tiếp
của nó. Đường thẳng AI lại cắt  tại D. Lấy điểm E trên cung BDC v| F trên cạnh BC sao 1
cho BAF  CAE  BAC . Gọi G l| trung điểm của IF. Chứng minh rằng c{c đường thẳng 2
DG v| EI cắt nhau tại một điểm trên 
Trích đề thi Olympic Toán Quốc tế - IMO 2010 Lời giải Nguyễn Công Lợi TÀI LIỆU TOÁN HỌC 74
Gọi giao điểm của EI với đường tròn  là P. Ta có P A
thể chứng minh DP đi qua trung điểm G của FI.
Do IPX  IAX nên tứ gi{c XPAI nội tiếp, suy ra
AXI  API  ALE  AFC . Từ đó suy ra XI song I X song với BC. Ta có G
DIC  DAC  ACI  DCB  BCI  DCI nên ta được C B F Y
DI  DC  DB. Nên D l| t}m đường tròn ngoại L E
tiếp tam gi{c BIC. Từ đó D  BP ∽ D  YB nên suy ra D DB DP  DY DB DI DP Từ đó suy ra  , nên ta được D  IP ∽ D  YI DY DI
Suy ra DIY  DPI  DAF nên IY song song với XP. Từ c{c điều trên suy ra FXIY l| hình
bình h|nh nên hai đường chéo FI v| XY cắt nhau tại trung điểm G. Vậy DG v| EI cắt nhau
tại một điểm trên  .
Ví dụ 29. Cho lục gi{c ABCDEF có BC  EF v| c{c đỉnh nằm trên đường tròn đường kính
AD. Gọi H l| giao điểm của AC với BD, K l| giao điểm của AE với DE. Gọi P, Q lần lượt l|
hình chiếu vuông góc của H trên AF, DE; R, S lần lượt l| hình chiếu vuông góc của K trên
AB, CD. Chứng minh rằng RS, PQ, HK đồng quy. Lời giải
Gọi M l| giao điểm của AB v| CD. Vẽ KI vuông góc với AD tại I. Do BC  EF và AD là
đường kính của đường tròn ngoại tiếp đa gi{c ABCDEF nên được BHC  AKD. M| ta lại có 0
BHC  BMC  180 nên suy ra 0
AKD  BMC  180 . Do đó tứ gi{c MAKD nội tiếp đường tròn.
Giả sử MAK  MDK. Do c{c tứ gi{c M
AIKR, DSIK. AMDK nội tiếp đường C tròn nên ta được 0 AIR  AKM  MAK  90 K1 B H 0  90  MDK  DKS  DIS K2 P S D A
Từ đó suy ra ba điểm R, I, S thẳng J I Q R hang. K
Gọi K ; K lần lượt l| điểm đối E 1 2 F
xứng với K qua AB, AD. Vì AK M  AKM  ADM và 1 Nguyễn Công Lợi TÀI LIỆU TOÁN HỌC 75 0
ADM  AHM  180 nên suy ra 0 AK M  AHM  180 . 1
Do đó tứ gi{c AHMK nội tiếp đường tròn. Suy ra AHK  AMK  AMK . 1 1 1
Ho|n to|n tương tự ta có tứ gi{c AHK D nội tiếp nên DHK  DAK  DAK  DMK 2 2 2 Từ đó ta được 0
AHK  DHK  AHD  AMK  DMK  AKD  AMD  AHD  180 1 2
Từ đó suy ra ba điểm K ; H; K thẳng h|ng. M| RI l| đường trung bình của tam gi{c KK K 1 2 1 2
nên RS đi qua trung điểm J của HK. Tương tự PQ cũng đi qua trung điểm J. Do vậy RS, PQ, HK đồng quy.
Ví dụ 30. Cho nửa đường tròn (O; R) đường kính AB v| một điểm M trên nửa đường
tròn(M kh{c A, B). Trên nửa mặt phẳng bờ AB chứa điểm M vẽ c{c tiếp tuyến Ax v| By
với nửa đường tròn. Tiếp tuyến tại M với nửa đường tròn cắt Ax, By lần lượt tại C, D.
Đường tròn ngoại tiếp tam gi{c COD cắt đường tròn (O) tại E v| F. Chứng minh rằng c{c
đường thẳng AD, BC, EF đồng quy. Lời giải
Dễ thấy AC//BD. Theo tính chất hai tiếp tuyến cắt J
nhau ta có AC  CM; BD  DM và 0 COD  90 . Gọi I
l| trung điểm của CD, N l| giao điểm của AC v| D
BC, H l| giao điểm của MN v| AB. Gọi K, S lần I
lượt l| giao điểm của EF với OM, OI. M CN AC CN CM
Theo định lí Talets ta có    C BM BD NB MD N K E F CN CM Trong tam giác CBD có  nên theo định lí NB MD A H O B
Talets đảo ta suy ra MN//BD. Do đó ta được
MN//AC//BD. Từ đó ta được MN DM BN NH    , suy ra MN  NH AC DC BC AC
Đường tròn ngoại tiếp tam gi{c COD l| đường tròn (I) đường kính CD. Vẽ đường kính OJ
của đường tròn (I). Vì đường tròn (I) cắt đường tròn (O) tại E, F nên OI l| đường trung
trực của EF. Từ đó ta được OI vuông góc với EF nên EF//AB. Tam gi{c EOJ vuông tại E có 1 ES l| đường cao nên 2
OS.OJ  OE . Do đó ta được 2 2
2OS.OI  OM  OS.OI  OM . Dễ thấy 2 OS OK 1 1 O  SK ∽ O  MI nên 
 OS.OI  OM.OK . Từ đó 2 OM.OK  OM  OK  OM hay OM OI 2 2 Nguyễn Công Lợi TÀI LIỆU TOÁN HỌC 76
K l| trung điểm của OM. Do đó NK l| đường trung bình của tam gi{c MHO nên KN//AB.
M| ta lại có EF//AB nên EF đi qua N. Vậy c{c đường thẳng AD, BC, EF đồng quy tại N.
Ví dụ 31. Cho điểm A nằm ngo|i đường tròn(O; R). Vẽ c{c c{t tuyến AEB, ADC với đường
tròn (O)(c{c điểm B, C, D, E thuộc đường tròn). Gọi H l| giao điểm của BD v| CE. Vẽ c{c
tiếp tuyến AM, AN với đường tròn (O)(M, N l| c{c tiếp điểm) trong đó B v| M nằm cung
nửa mặt phẳng bờ AH. Chứng minh rằng c{c đường thẳng AH, BN, CM đồng quy. Lời giải
Gọi H’ l| giao điểm của giao điểm của MN v| BD, A
E’ l| giao điểm của CD’ v| đường trong (O). Ta có 0
AMO  ANO  90 nên tứ gi{c AMON nội tiếp D
đường tròn đường kính AH. Gọi I l| t}m của I N E' E H' H
đường tròn đó. Gọi S l| giao điểm của AH’ với F M K' K đường tròn (I). Khi đó S O
H' E.H' C  H' M.H' N  H' A.H' S B C
Từ đó suy ra tứ gi{c AE’SC nội tiếp đường tròn, do
đó ta được AE ' C  ASC . Gọi F l| giao điểm của
AO v| MN. Khi đó dễ d|ng chứng minh được 2
AH '.AS  AF.AO  AN  AC.AD
Từ đó suy ra tứ gi{c H’SCD nội tiếp đường tròn, do đó ta được 0
H ' SC  H ' DC  180 . Mà ta có
BE ' C  H ' DC . Từ đó ta được 0 AE ' C  BE ' C  180
Từ đó suy ra ba điểm A, E’, B thẳng hang nên hai điểm E v| E’ trùng nhau, hai điểm H v| H’ trùng nhau.
Như vậy ba điểm M, H, N thẳng hang. Gọi K’ l| giao điểm của AH v| CM.
Xét đường tròn (I) có ASN  MNA v| trong đường tròn (O) có MCN  MNA
Do đó ta được ASN  MCN suy ra tứ gi{c K’SCN nội tiếp. Từ đó ta được 0 K' CN  K' SN  180
Ta chứng minh được HB.HD  HM.HN  HA.HS nên suy ra tứ gi{c ABSD nội tiếp đường tròn.
Do đó suy ra ABD  ASD. Trong đường tròn (O) có ABD  HCD .
Suy ra ASD  HCD nên tứ gi{c SHDC nội tiếp đường tròn. Do đó 0 HSC  HDC  180 . Nguyễn Công Lợi TÀI LIỆU TOÁN HỌC 77
M| ta lại có HDC  BNC . Kết hợp c{c kết quả trên ta được K ' NC  BNC , suy ra hai tia NK’
v| NB trùng nhau hay ba điểm B, K’, N thẳng hang. Từ đó suy ra hai điểm K v| K’ trùng nhau.
Như vậy ba điểm A, K, S thẳng hang nên suy ra AH, BN, CM đồng quy tại một điểm.
Ví dụ 32. Cho tam gi{c ABC có AH, BM, CD lần lượt l| đường cao, đường trung tuyến,
đường ph}n gi{c của tam gi{c. Biết sin BAC  cosABC. tan ACB . Chứng minh rằng AH, BM, CD đồng quy. Lời giải 1 Ta có S
 AH.BC. Trong tam giác vuông ABH có AH  AB.sin ABC nên ta được ABC 2 1 1 AH.BC S
 AB.BC.sin ABC . Ta lại có S
 AB.AC.sin BAC . Do đó sin BAC  . ABC 2 ABC 2 AB.AC BH
Trong tam giác vuông AHB có cos ABC 
và trong tam giác vuông ACH có AB AH tan ACB  . CH
Từ giả thiết sin BAC  cosABC. tan ACB ta suy ra được A AH.BC AH BH BC BH  .   . AB.AC AB CH AC CH
Gọi I l| giao điểm của AH v| BM. Vẽ MK vuông góc D K M
với AH tại K.. Trong tam gi{c AHC có MA  MC và I I'
MK//HC nên ta được AK  KH nên KM l| đường B H C
trung bình của tam gi{c AHC, do đó HC  2MK . BC BH BC BH Từ đó ta được    . 2CM 2MK CM MK BH BI BC BI
Cũng do MK//HC nên ta được  . Do đó ta được  . MK IM CM MI
Gọi I’ l| giao điểm của BM v| CD. [p dụng tính chất đường ph}n gi{c cho tam gi{c BCM ta được BC BI ' BI BI ' BI BI ' BI BI '         BI  BI ' MC MI ' MI MI ' BI  MI BI ' MI ' BM BM
Từ đó suy ra hai điểm I v| I’ trùng nhau, do đó AH, BM, CD đồng quy.
Ví dụ 33. Cho tam gi{c nhọn ABC. Đường tròn (O) đi qua hai điểm B, C cắt AB, AC lần
lượt tại F v| E. Gọi H l| giao điểm của BE v| CF. Chứng minh rằng c{c tiếp tuyến tại E, F
với đường tròng (O) v| AH đồng quy tại một điểm. Lời giải Nguyễn Công Lợi TÀI LIỆU TOÁN HỌC 78
Gọi giao điểm của BC v| EF l| P. A
Đường thẳng HP cắt đường tròn
(O) tại X, Y(X nằm giữa P v| H).
Gọi giao điểm của đường tròn T E
ngoại tiếp tam gi{c HEF với OA l| Y F
V. Khi đó ta chứng minh được H V X
PH.PV  PE.PF  PB.PC. Từ đó Q O
suy ra tứ gi{c BCVH nội tiếp S B D C đường tròn. Do đó P J R
BVF  BCF  BEF  BOF nên tứ
gi{c BFOV nội tiếp đương tròn.
Ho|n to|n tương tự ta được tứ
gi{c VECO nội tiếp đường tròn.
M| hai đường tròn giao nhau tại
O v| V nên BF v| CE cắt nhau tại
một điểm trên đường thẳng VO.
M| ta lại có AF.AB  AE.AC .
Do đó ba điểm O, V, A thẳng h|ng. Từ đó BVO  AVF nên 0 OVX  90 hay PH vuông góc với OA.
Chứng minh tương tự ta được AH vuông góc với OP. Gọi S l| giao điểm của AH v| OP. OV OF Khi đó ta được O  VF ∽ O  FA nên ta có 2   OF  OV.OA OF OA Từ đó ta được 2 OX  OV.OA , suy ra 0
OXA  90 . Do đó AX, AY l| tiếp tuyến của đường tròn (O).
Từ đó ta được PD.PJ  PS.PO  PH.PV  PE.PE suy ra tứ gi{c EFDJ nội tiếp đường tròn.
Do đó ta thu được PB.PC  PE.PF  PD.PJ
Qua D kẻ đường thẳng song song với EF cắt AB, AC lần lượt tại R, Q. DB DR Từ đó suy ra D  BR ∽ D  QC nên ta được 
 DB.DC  DQ.DR . Do đó ta được DQ DC
DQ.DR  DB.DC  BJ  JD CJ  JD  BJ  JD BJ  JD 2 2  BJ  DJ   2 2 PJ  DJ    2 2
PJ  BJ   PDPJ  JD   2 2 PJ  CJ 
 PDPJ  JD  PB.PC  PD.PJ  PD.JD  PB.PC  PD.DJ Từ đó suy ra D  PR ∽ D
 QJ nên ta được DPR  DQJ , suy ra tứ gi{c RPQJ nội tiếp đường tròn.
Vậy đường tròn ngoại tiếp tam gi{c PQR đi qua trung điểm J của BC. Nguyễn Công Lợi TÀI LIỆU TOÁN HỌC 79
Ta có BEC  BFC nên ta được AFH  AEH , suy ra HSB  HEA
Từ đó ta chứng minh được HC.HF  HE.HB  HA.HS , suy ra tứ gi{c AFSC nội tiếp đường tròn.
Gọi T l| giao điểm của hai tiếp tuyến tại E, F với đường tròn (O). Khi đó ta có 0
ESF  ETF  CSF  CSE  ETF  180  BAC  CHE  ETF 1 1 1 0
 180  sdBC  sdEF  sdBF  sdCE  sdBF  sdBC  sdCE  sdEF 0  180 2 2 2
Từ đó suy ra tứ gi{c SETF nội tiếp đường tròn, suy ra FST  FET . M| ta lại có
FSA  FCA  FET , do đó suy ra FST  FSA . Suy ra hai tia SA, ST trùng nhau hay bốn
điểm S, T, A, H thẳng h|ng.
Vậy tiếp tuyến tại E, F với đường tròn (O) v| AH đồng quy tại một điểm.
Ví dụ 34. Cho tứ gi{c ABCD không phải l| hình thang nội tiếp đường tròn (O). Gọi giao
điểm của AB v| CD l| E, giao điểm của AD v| BC l| F, giao điểm của AC v| BD l| H.
Đường tròn (O) cắt đường thẳng EH tại M v| N. Gọi I l| trung điểm của MN. Chứng minh
rằng hai tiếp tuyến tại M, N với đường tròn (O) v| đường thẳng OI đồng quy tại một điểm. Lời giải
Gọi I l| giao điểm của đường tròn 1 F
ngoại tiếp tam gi{c HAB v| MN. Ta sẽ
chứng minh ba điểm O, I v| F thẳng 1 A N
h|ng, từ đó suy ra được hai điểm I v| I1 B trùng nhau. I M H
Thật vậy, do tứ gi{c ABHI nội tiếp 1 O
đường tròn nên ta được BI H  BAH . Lại 1 E C D
có BAH  BDE nên ta được BI H  BDE , 1
từ đó suy ra tứ gi{c I BED nội tiếp 1
đường tròn. Từ đó ta được I BH  I EC . 1 1
Mà ta có I AC  I BH nên ta được 1 1 I AC  I EC . 1 1
Điều n|y dẫn đến tứ gi{c I AEC nội tiếp đường tròn. Ta có HDC  HAB và HAB  HI C 1 1
nên ta được HDC  HI C , do đó tứ gi{c I DCH nội tiếp đường tròn. Sử dụng tính chất góc 1 1 Nguyễn Công Lợi TÀI LIỆU TOÁN HỌC 80
ngo|i của một tứ gi{c nội tiếp ta được AI N  ABH; NI D  HCD . M| ta lại có ABH  HCD 1 1
nên ta được AI D  2ABD . 1
Mặt kh{c ta có AOD  2ABD , do đó suy ra AI D  AOD nên tứ gi{c I ODA nội tiếp đường 1 1 tròn.
Chứng minh ho|n to|n tương tự ta được I OCB nội tiếp đường tròn. 1
Khi đó I v| O l| hai giao điểm của hai đường trong ngoại tiếp c{c tứ gi{c I ODA và 1 1 I ODA . 1
M| ta lại có FA.FD  FB.FC nên suy ra F thuộc đường thẳng OI hay ba điểm O, I và F 1 1 thẳng hang. AI F  ODA 1    AI F  OI D Ta có 1 1 OA  D  ODA    OI  D  ODF 1 OAD OI D    1 
Mà ta có AI N  NI D nên kết hợp với kết quả trên ta suy ra được MN vuông góc với OI . 1 1 1
M| ta lại có MN vuông góc với OI. Từ đó suy ra được hai điểm I v| I trùng nhau hay suy 1 ra OI đi qua F.
Cũng từ kết quả trên ta được O  ID ∽ O  DF nên ta được 2 OI.OF  OD . Do đó suy ra 2
OI.OF  ON nên hai tam gi{c OIN v| ONF đồng dạng với nhau. Từ đó suy ra 0
ONF  OIN  90 nen NF l| tiếp tuyến của đường tròn (O) tại N. Chứng minh
tương tự ta cũng được MF l| tiếp tuyến của đường trong (O) tại M.
Vậy hai tiếp tuyến tại M, N với đường tròn (O) v| đường thẳng OI đồng quy tại một điểm.
Ví dụ 35. Cho tam gi{c ABC không c}n. Đường tròn nội tiếp (I) của tam gi{c tiếp xúc với
BC, CA, AB lần lượt tại A ; B ;C IA ; IB ; IC 0 0 0 . 1 1
1 theo thứ tự l| ph}n gi{c cảu tam gi{c IBC,
ICA, IAB. C{c đường ph}n gi{c A A ; B B ;C C A B C I 0 2 0 2 0 2 của tam gi{c 0 0 0 đồng quy tại 0 .
Chứng minh rằng c{c đường thẳng A A ; B B ;C C II 1 2 1 2
1 2 đồng quy tại một điểm trên 0 . Lời giải Nguyễn Công Lợi TÀI LIỆU TOÁN HỌC 81
Không mất tính tổng qu{t ta có thể giả sử A
AB  BC  CA . Gọi K ; K ; K a b c lần lượt l|
giao điểm của c{c đường thẳng A A ; B B ;C C II A ; B ;C C0 1 2 1 2 1 2 với 0 . Gọi 3 3 3 A2
theo thứ tự l| hình chiếu của I A3 0 trên K B a 0 I I0 B C ;C A ; A B r; r 0 0 0 0 0 0 tương ứng. Gọi 0 lần B2 C2
lượt l| b{n kính đường tròn nội tiếp tam giác ABC và A B C B A
0 0 0 . Chú ý l| c{c tứ gi{c 1 A0 C AB IC ; BC IA ;CA IB 0 0 0 0 0 0 nội tiếp đường tròn nên ta được 1 1 1 B A C  ABC  ACB ; A B C  BAC  ACB ; A C B  BAC  ABC 0 0 0   0 0 0   0 0 0   2 2 2
Theo tính chất góc ngo|i của tam gi{c v| chú ý đến tam gi{c CA B 0 0 c}n tại C. Lại có 0
BAC  ABC  ACB  180 nên ta thu được 1 1   0 ABC  ACB
IA A  IBA  BIA  ABC  180   1 0 1 1 2 2  2    ABC  ACB 1    0
180  ACB  A A B  B A C  A A C 2 0 0 0 0 2 0 4 2 1 1 I A A  A C A  A A C  BAC  ABC  ABC  ACB 0 2 3 2 0 0 2 0 0     2 4 1   0   1 180
ACB  ABC  ACB  IA A 1 0 2 4
Từ c{c kết quả trên ta thu được IA I A I  A A ∽ I  A A 1 song song với 0 2 và 1 0 0 2 3 IA IA r K I r Từ đó 1 0  
. Chú ý là A ; A thuộc nửa mặt phẳng có bờ II nên ta có a  I A I A r 1 2 0 K I r 0 2 0 3 0 a 0 0 . K I r K I r
Ho|n to|n tườn tự ta cũng có b  và c 
. Từ đó suy ra ba điểm K ; K ; K K I r K I r a b c b 0 0 c 0 0
trùng nhau hay c{c đường thẳng A A ; B B ;C C II 1 2 1 2
1 2 đồng quy tại một điểm trên 0 . Nguyễn Công Lợi TÀI LIỆU TOÁN HỌC